Fawwaz Solution

Download as pdf or txt
Download as pdf or txt
You are on page 1of 339
SOLUTIONS WENUAL FAWWAZ T. ULADY FUNDAMENTALS OF APF Utd ELECTROMAGHETICS Chapter 1 Section 1-3: Traveling Waves Problem Li A 4-KElz sound wave traveling in the a-direction in air was observed tohave a differential pressure p(x,t) = SNim? at x= 0 and = 25 ys. Ifthe reference phase of p(x,7) is 42°, find a complete expression for p(x,t). The velocity of sound in air is 330 mvs. Solution: The general form is given by Eq. (1.17), ee 2m rashwten( 2 2E 4), where it is given that 9 = 42°, From Eq. (1.26), T = 1/f = 1/(4x 10°) = 0.25 ms. from Eq. (1.27), upf 330 3 Trig = 825x107 m. Also, since (x= 0, t= 25 us) = 5 (Nim?) = Acos { Spa +42" Taos = Acos(1.36,rad) = 0.2084, eS a) it follows that A = 5/0.208 = 24 N/m, So, with r in (8) and x in (mm), 1 (x,t) =24e08 (2 x 108. = 2Acos(8n x 1051-24.24nx-+42°) (N/m). — 2) anx IO ges +42) im) (i Problem 12 Forthe pressure wave described in Example 1-1, plot (@) p(x) versus.xatr=0, () (x,t) versus? atx=0. Be sure to use appropriate scales for x and so that each of your plots covers at least two cycles. Solution: Refer to Fig. P1.2(a) and Fig. P1.2(b). 2 CHAPTER ! Figure P12: (a) Pressure wave as a function of distance at r = 0 and (b) pressure wave as a function of time at x = 0. Problem 13 A harmonic wave traveling along. string is generated by an oscillator that completes 120 vibrations per minute. If it is observed that a given crest, or maximum, travels 250 em in 10 s, what is the wavelength? Solution: Problem 14 Two waves, y1(¢) and yo(¢), have identical amplitudes and oscillate at the same frequency, but y2(?) leads y(t) by a phase angle of 60°. If vilt) = 4cos(2x x 102), ‘write down the expression appropriate for y2(1) and plot both functions ever the time span from 0 t0 2 ms. Solution: ya(t) = 4cos(2x x 107 +60"). Figure P1.4: Plots of yi(t) and yo(7). i Problem 1.5 The height of an ocean wave is described by the function yar) = 1Ssin(0.5¢—0.62) (mn). Determine the phase velocity and the wavelength and then sketch y(t) at ¢= 25 over the range from x= 010 x= 22. Solution: The given wave may be rewritten as a cosine function: ylxt) = 1.Seos(0-S¢—0.6x—n/2). By comparison of this wave with Eq. (1.32), x,t) =A cos(a —Br+90), ‘we deduce that Figure P15: Plot of y(x,2) versus x Atr=25, (2,2) = 1.Ssin(1 ~0.6x) (mm), with the argument of the cosine function given in radians. Plot is shown in Fig. P15. Problem 1.6 A wave traveling along a string in the +-x-direction is given by yalet) = Acos(t— Bx), where x= 0 is the end of the string, which is tied rigidly to a wall, as shown in Fig. 1-21 (P1.6). When wave y1(x,1) arrives at the wall, a reflected wave y2(%t) is generated. Hence, at any location on the string, the vertical displacement y, will be the sum of the incident and refiected waves: yol2) = yxet) + ya). (a) Write down an expression for yo(x,#), keeping in mind its direction of travel and the fact that the end of the stcing cannot move. (©) Generate plots of yi(x,t), yalx) and ys(z,¢) versus x over the range AWA SxS Vator = 0/4 and at ox = 7/2. Solution: 2) Since wave y2(,1) was caused by wave yy(2,"), the two waves must have the same angular frequency ©, and since yo(x,t) is traveling on the same string as yi(2,"), Figure P1.6: Wave on a string tied to a wall at x = 0 (Problem 1.6). the two waves must have the same phase constant B. Hence, with its direction being in the negative x-direction, ya( x,t) is given by the general form ya(eyt) = Beos( ax +Bx+ G0), a where B and $o are yet-to-be-determined constants. The total displacement is debt) = Gut) +92" ‘Acos («ar — Bx) + Beos( r+ B+ )- Since the string cannot move at x = 0, the point at which itis attached to the wall, ys(0,t) = 0 for all r. Thus, @) (@ Easy Solution: The physics of the problem suggests that a possible solution for (2)is B = —A and do = 0, in which case we have y(0,t) = Acosar + Beos( ‘ar +60) = yalxt) = —Acos( ax + Bx). 8 Gi) Rigorous Solution: By expanding the second term in (2), we have Acosax + B(cos et costa — sin ax sine) = 0, (A+Boosdo) cose — (Bsingo)sinax = 0. @ “This equation has to be satisfied for all values of. Att = 0, it gives A+Bcosdy = 0, o 6 CHAPTER 1 and at cr = n/2, (4) gives Bsingo = 0. © ‘Equations (5) and (6) can be satisfied simultaneously only if A=B=0 a or . A=-B md $=0. 8) Clearly (7) is not an acceptable solution because it means that y;(x,f) = 0, which is contrary ‘0 the statement of the problem. The solution given by (8) leads to (3). (b) Atar= 5/4, jnlet) = Acos(n/4—Br) = Acos (3-2) , yale,t) = —Acos(ar+Bx) = —Acos (G2) . Plots of yi, yz, and ys are shown in Fig. P1.6(b). wten/4 Figure P16: (b) Plots of yi, y2, and y, versus x at ar = 1/4, Atar=n/2, valet) = Acos(n/2— Bx) =AsinBe= Asin =, CHAPTER ! 7 salest) = ~Acos(n/2-+ Ba) = AsinBr = Asia = Plots of y1, yz, and y3 are shown in Fig. PI.6(C). Figure P16: (c) Plots of yi, y2, and ys versus x at or = n/2. Problem 1.7 Two waves on a string are given by the following functions: yalxe) = 3e0s(20r—302) (em), yal.t) = —3e0s(20+30x) (em), where x is in centimeters. The waves are said to interfere constructively when their superposition [y.| = [yi +y2| is @ maximum and they interfere destructively when [psi isa minimum. (@) What are the directions of propagation of waves yi(xt) and ya(t)? (©) Atr = (/50) , at what location x do the two waves interfere constructively, and what is the corresponding value of |y.|? (© Att = (%/50) s, at what location x do the two waves interfere destructively, and what is the corresponding value of ly.|? Solution: (a) p(x) is traveling in positive xdirection. y2(z,1) is traveling in negative direction 8 CHAPTER I (b) Att = (1/50) 8, ye = 1 +92 = 3[c0s(0.4z— 30x) —cos(0.4r-+3x)]. Using the formulas from Appendix C, 2sinxsiny = cos(z—y)—(cosx+y), we have ¥z = 2sin(0.4n) sin30x = 1.9in30s. Hence, Uyslmax = 1-9 and it oceurs when sin30x = 1, o 30x= n=0,1,2, (© [yslmin = O and it occurs when 30x = nn, or x = = em, Problem 1.8 Give expressions for y(x,t) for a sinusoidal wave traveling along a string in the negative x-direction, given that ymax = 20 cm, = 30em, f = 5 Hz, and (@) y(x,0) =Oatx=0, (©) (2,0) = Oatx= 75cm. Solution: For a wave traveling in the negative x-direction, we use Eq. (1.17) with @ = 2nf = 10e (rad/s), B = 2n/R = 2n/0.3 = 20n/3 (rad/s), A = 20 cm, and x assigned a positive sign: v0) = 20 (100+ 22869) em, with x in meters (@) ¥(0,0) 20coseo. Hence, do = +n/2, and Wat) = 20e0s (10m Pe ) = §-20sin (10m + 42) (cm), ifby = 2/2, 20sin(10m+ 28x) (cm), if @ = —n/2. @)Atx=7.5em= and 5x10? m, 20cos(x/2+ 60). Hence, 6 = 0 orn, fete 20cos(10nr+ 85x) (cm), if oo = 0, 2 = 1 —20c05 (10nt + 28x) (cm), if og =m. CHAPTER I 9 —_—_———_—_ Problem 1.9 An oscillator that generates a sinusoidal wave on a sting completes 20 vibrations in 30 s. The wave peak is observed to travel 2 distance of 2.8 m along, the string in 5 s. What is the wavelength? Solution: p-Berss, upe2d osems, = wp? = 0.56 1.5 = 0.8m. I Problem 1.10 ‘The vertical displacement of a string is given by the harmonic fonction: y(t) = Seos(12nr—20nx) (), where x is the horizontal distance along the string in meters. Suppose a tiny particle were to be attached to the string atx = 5 cm, obtain an expression for the vertical velocity of the particle as a function of time. Solution: (x,t) = Scos(12nt—20nx) (mn) a(x,1)| a | e005 60nsin( 12m — 207) x=0.05 = 60nsin(12n¢—7) —60nsin(12et) (xn/). — Problem 1.11 Given two waves characterized by y(t) = Seoser, ya(t) = Gsin( ax +30), oes ya(t) lead or lag ys().and by what phase angle? Solution: We need to express ya(!) ip terms of a cosine function: 6005(60° ~ ar) = 6eos(«ar ~ 60°). 10 (CHAPTER 1 ‘Hence, y2(r} lags yx(r) by 60°. Problem 1.12 The voltage of an electromagnetic wave traveling on a transmission line is given by v(z,2) = 3e~*sin(2x x 10°r— 10nz) (V), where z is the distance in ‘eters from the generator. (2) Find the frequency, wavelength, and phase velocity of the wave. (®) Atz=2m, the amplitude of the wave was measured to be 1 V. Find 0. Sotution (2) This equation is similar to that of Eq. (1.28) with @ = 2x x 10° rad/s and B = l0xsad/m. From Eq, (1.292), f = w/2n= 10° Hz = 1 GHz; from Eq, (1.290), 2n/B = 0.2m. From Eq, (1.30), Uy = @/B = 2x 108 m/s. (b) Using just the amplitude of the wave, ataft Ta \3 Problem 1.13 A certain electromagnetic wave traveling in sea water was observed to have an amplitude of 19.025 (V/m) at a depth of 10 m and an amplitude of 12.13 (W/m) at a depth of 100 m. What is the attenuation constant of sea water? Solution: The amplitude has the form Ae™. Atz= 10m, 3a 55 Npim. and at z= 100m, The ratio gives e100 ws 1.568671, ‘Taking the natural log of both sides gives tn(ew"™) = in( 1.568619), =10c.= In(1.568) ~ 100c, ‘900 = In(1.568) = 0.45. Hence, a= 98 < 5x10 aap, Problem 1.14 Evaluate each of the following complex numbers and express the result in rectangular form: @) z= 3274, () 22 = V3 68/4, © 3=2e*, @a © : =(1-J7. © z6=( Bie (Note: In the following solutions, numbers are expressed to only two decimal places, but the final answers are found using a calculator with 10 decimal places.) (a) 2 =3e%/4 = 3(cosm/4+ jsinn/4) =2.12+ j2.12=2.12(1+J)- &) pa VieRM aE {-s(#) +isn(%)] W124 jim = 124-14). (©) 25 = 2M = Yoos(—n/2) + jsin(—#/2)] @ waPajPs—jor tgs Pm (eFI2)3 = el = cos(3n/2) + jsin(3n/2) = © wasrta(e)* © a= (1D (VMS) = (VERE (v2)°[eos(3x/4) — jsin(3z/4)] -2- j2= -2(1+ 3). © sp (1— NP (Be RYE = Ve = 1.10(0.92— 0.38) = 4(1-10~ 70.45). — Problem 1.15 Complex numbers z; and zz are given by i 4+ j2. 2 CHAPTER 1 (@) Express z; and 2, in polar form. (b) Find |zi| by applying Eq. (1.41) and again by applying Eq. (1.43). (©) Determine the product 2:22 in polar form. {@) Determine the ratio z; /z2 in polar form. (e) Determine 2 in polar form. Solution: (@) Using Ea. (1.41), 66787 a=3-2 E44 j2 = 4.5e (b) By Eq, (1.41) and Eq, (1.43), respectively, eal = [3 - j2| = of 32+ (2)? = VIB = 3.60, kal = V@=2)GFR) = VB = 3.60. (©) By applying Eq. (1.47b) to the results of part (a), ryan = 3.6797" x 4.501554" = 16,2019, (@ By applying Eq, (1.486) to the results of pare (2), 2 _ 3.6? a 45e 0.80e7/1873", (©) By applying Eq. (1.48) to the results of part (a), B= (3.637) = (3,68 = 26,6667", —_—_ Problem 116 ifz= @ yz @) 2, © ke @) Im{z}, (©) Sm{z"}. Solution: (Note: In the following solutions, numbers are expressed to only two cecimal places, but the final answers are found using a calculator with 10 decimal Places.) + j3, determine the following quantities in polar form: 1B 1 25 Fi (b) 2 =(-2+ 53) @ dt =z-z =(-24 3)-2- 73) (@ Jm{z} = Im{-2+ j3} = 3. (© 3m{z} = Jm{-2- 3} Ar (3.61) 1eAB = 0.28 4 =(-24 8) 1s (3.610 46,87 eit0T 3= 32, Problem 1.17 Find complex numbers: for each of the following pairs: Oa @a= Solution: @ . atm = (2+ 73)+(-72)=34+/1 3.162784" 2p = (24 73)—(1= 2) = 14 JS = S107, o pzy = 2—j2= 283, 2.83245". © tap = SBE ESL 230" 4 52 BY = (2.64 j1.5}+(2.6- flS)= 52, 230 _ 56“ F" = (2.64 j1.5)—(2.6— f1-5)) nee @ pany toy = 34 SLIM = (2.64 f1.5) + (-2.8- 1.5) = 0, =a = (26+ fl.5)-(-26—j15)=5.2+ 3 = 62". eS, Problem 1.18 Complex numbers 2; and zp are given by n= S28, malas, 14 CHAPTER i (@) Determine the product 2:22 in polar form. “%b) Determine the product 2:25 in polar form. (©) Determine the ratio z; /z2 in polar form. (@) Detormine the ratio 23 /z} in polar form. (© Determine /@ in polar form. Solation: (2) 2129 = Se" x Delt” = 102 ) 1g = Se" De AF = 10, za _ Se" ae = 2.57705", 0 ee @ a () 2.5108", gle © Va V5 = eV5e", Probl Solution: 119 If. 3 jA, find the value of In(z). kl=+vVP48=5, 8 za izle® = se", (2) = (Se) = 1n(5) + In(e“3") = 1.61 — j53.1° = 1.61-, jie Fo 1.61 ~ j0.93. Problem 1.20 If 2=3— j4, find the value of e. Solution: ea Bad eH = e3(cos4— jsin4), 4 = 20.09, and dred = > x 180° = 229.18", Hence, & = 20.08(cos229.18° — jsin229.18*) = —13.13+ j15.20. CHAPTER 1 15 Section 1-6: Phasors Problem 1.21 A voltage source given by v(t) = 10cos(2x x 10%r — 30°) (V) is ‘connected to a series RC load as shown in Fig. 1-19. If R= 1 MQ and C = 100 pF, obtain an expression for ve(?), the voltage across the capacitor. Solution: In the phasor domain, the circuit is a voltage divider, and Wjoc _ % SRF 1] jaC ~ (1 jORC)” Now V, = 102780" v with @ = 2x x 10° rad/s, so 10¢-80" Vv, T+ (lon x 10? rads) x (10° 3) x (100 10" F)) 102730 V 1458/5 SoH y. Converting back to an instantaneous valve, velt) = ReVcei®! = mRe8.5el*-O) v= 8.Sc0s(2n x 10% 621°) V, where r is expressed in seconds. —_—————— Problem 1.22 Find the phasors of the following time functions: (a) v(z) = Seos(ar—n/4) (Vs (&) vz) = 12sin(or+-7/4) (V), (©) ie,1) = 4e“* sina — 7/6) (A), (@) it) = —2c0s(or+3x/4) (A), (©) i(2) = 2sin( wr + n/3) + 3cos(er—/6) (A). Solution: @) Vase Fy, (b) v(t) = 12sin(cor +n /4) = 12cos(n/2—(@r+n/4)) = 12cos(ar—n/4) V. Va iae IY, © i(t) = 4e™ sin (er —#/6) A = 4e-* cos (n/2— (or — 2/6) A 4e7* cos (cor — 2n/3) A, gee PA? A. 16 CHAPTER I @ —2eos(or + 38/4), = 2eFPPI = Deel = De A, © i(t) = 2sin( ce +-n/3) + 3c0s(ar —n/6) = 2eos[n/2 — (te +-n/3)] + 3c0s( car — 7/6) cos —ar + 2/6) + 3c0s( wt 2/6) .cos( car — n/6) + 3eos( er — x/6) = Seos(ar—/6), -il6 Problem 123 Find the instantaneous time sinusoidal functions corresponding to the following phasors: (@) V=-3e (v), ®) = j6e™* W), © T=(+s4) A), @ [=-3+72 A), ©l=j, @ T= 204 (a). Solutic @ F = —3eP FV = Bel2/3-") Vx 373 V, v(e) = 3e0s (ar ~ 28/3) V. ) = joe v = Gel/t8I2) y = gel/4 y, cos (ax + 3/4) V. © 3474) A= SB a, ‘cos (cx +53.1°) A. @ 34 j2=3.61e/463"", H(t) = Re{3.61 ef4657" eH} = 3.61 cos(ar+146.31°) A. ci T=2088/4, it) = Re{2eP*4el} = 2eos(ar + 3/4) A. Problem 1.24 A series RLC circuit is connected to a generator with a voltage vs(e) = Vocos(r +/3) (W). (a) Write down the voltage loop equation in terms of the current i(t), R, L, C, and vs(t). i (&) Obzain the corresponding phasor-domain equation. . | (©) Solve the equation to obtain an expression for the phasor current J. R L WTF wo) =e Figure P1.24: RLC circuit. Solution: di. (@) w(y=RieLg+e fia. - | (b) In phasor domain: H=al+ jauls Ee. © ¥ Voeimi? oCVpel9/5 + F(@L=T]OC) ~ RE F(OL= Tao) ~ OREF PLE—1) © 18 CHAPTER 2 Chapter 2 Sections 2-1 to 2-4: Transmission-Line Model Problem 2.2. A transmission line of length ! connects a load to a sinusoidal voltage source with an oscillation frequency f. Assuming the velocity of wave propagation on the line is ¢, for which of the following situations is it reasonable to ignore the ‘presence of the transmission line in the solution of the circuit: (a) 1= 206m, f = 10KH2, (®) 1=50km, f= 60 Hz, © [= 20cm, f= 300M, @ [= 1mm, f = 100 GHz. Solution: A transmission line is negligible when J/2. < 0.01 ot Z = PN OPH 8 G67 x10 (negligible) wt IL _ (50x10 m) (60% 10 HA) _ 9.64 perenne) ot Ze (20x10 m) x (300 x10" He) .20 (nonnegligible). @t go eee 35 (nonnegligible). Problem 22 Calculate the line parameters R’, L', G’, and C’ for a coaxial line with ‘an inner conductor diameter of 0.5 cm and an outer conductor diameter of 1 cm, ‘filled with an insulating material where «= fo, &r = 2.25, and 6 = 10~} Sim. The conductors are made of copper With we = Hp and Ge = 5.8 x 10” Sfm. The operating frequency is 1 GHz. Solution: Given a= (0.5/2) om = 0.25 x 107? m, 1.0/2) em = 0.50 x 10°? m, ‘combining Bqs. (2.5) and (2.6) gives Bie B (5) ec Hu)(4x x 10"? Him) og ae (aS = 0.788 Qin. 19 Hy (8) _ Sex 10-7 Bm = £in(2) = OE Bae 139 nm. From Eq. (2.8), 2no_ _ 2nx 1073 Sim Boj tee. From Eq. (2.9), 1p 2M _ nese _ 2m x 2.25 x (8.854 x 10-"? Fm) in(@/a) ~ in(b/a Ind = 181 pF/m. Problem 23 A 1-GHz parallel-plate transmission line consists of 1.5-em-wide copper strips separated by 2 0.2-cm-thick layer of polystyrene, Appendix B gives He = Mo = 40x 1077 (im) and 6, = 5.8 x 10 (S/m) for copper, and &; = 2.6 for polystyrene. Use Table 2-1 to determine the line parameters of the transmission line. ‘Assume 11 = 1 and c ~ 0 for polystyrene. Solution: Bee a= eee * 10 (am, unit ee SEN 67x 10-7 im), G=0 because o = 0, c's Benet 26x SX 172x107! (Fim). Problem 2.4 Show that the transmission line model shown in Fig. 2-37 (P2.4) yields the same telegrapher’s equations given by Eqs. (2.14) and (2.16). Solution: The voltage at the central upper node is the same whether itis calculated from the left port or the right por: Met fat) = vet) Hae let) Seed) = v(z+Az,t)+ 4RAz (z+ Az,2) +: paedies ae). Figure P2.4: Transmission line model. ‘Recognizing that the current through the G’ || C! branch is i(2,2) - i(z+Az,t) (from ‘Kirchhoff’s current law), we can conclude that ile) Hebe) = Caeres $as,) +CarSole+ de). From both of these equations, the proof is completed by following the steps outlined in the text, ie. rearranging terms, dividing by Az, and taking the limit as Az — 0. —————— Problem 25 Find 0,B,up, and Zp for the coaxial line of Problem 2.2. Solntion: From Eq. (2.22), y= VRFILV CS FIC) (0.788 en) + j(2x x 108 5-)(139 x 10-9 Hm) 3 @.1% 10° Shen) + ja x 1 (ABT x 10M Fem) = (140x 10-3 + j31.5) m7". ‘Thus, from Eqs. (2.252) and (2.256), a= From Eq. (2.29), ).140 Npfmn and B = 31.5 zad/m. _ [Re jet! __ [ (0.788 Dim) + j(2mx 10 s-1)(139 x 10-9 Ha) =) Ge ja! G-1 x10 She) + j(Onx 10" SBT x 1O-* Fem) = (27.7 + j0.098) 2. From Eq. (233), @ _ 2xx10° =2x 10° mis. a problem 2.6 Tn addition to not dissipating power, a lossless line has two impostant feamres: (1) it is dispertionless (4 is independent of frequency) and (2) its ‘Gharacterstic impedance Zp is purely real. Sometimes, itis not possible to design " gtransmission line such that R’ < @L' and G' < @C", but itis possible to choose the “mensions of the line and its material properties so 2s to satisfy the condition : RC'=I'G' (distortionless line). Such a line is called a distortionless line because despite the fact that itis not lossless, jit does nonetheless possess the previously ‘mentioned features of the loss line. Show thax fora distortionless ine, Ea = Using the distortionless condition in Eq. (2.22) gives yaar P= (RF ALC + jac) = VEC (E+) (E+8) =vie,|(E+ ie) (F+10) vei (® =Ry + jovi } CE SS Zin Za oe Figure P2.23: (b) Equivalent circuit. (17.60— j4.31) + j50tanf(2x/2)(0.30)]} _ Ze= oof FE BD= 2ST pean| (Ex AY(O-SH J = (10757-8967). Se CHAPTER 2 33 Section 2-7: Special Cases Problem 2.24 At an operating frequency of 200 MHz, itis desired to use a section ‘of a lossless 50-@ transmission line terminated in a short circuit to construct an ‘equivalent load with reactance X = 25 ©. If the phase velocity of the line is 0.75c, ‘what is the shortest possible line length that would exhibit the desired reactance at its input? Solution: (2rradifeycte) x (200 x 108 eyclels) 0.75 x (3 x 10° ms) (Ona lossless shon-cireuited transmission line, the inpot impedance is always purely imaginary: Le., Z% = JX. Solving Eq, (2.68) for the line length, bn (ME) = 1 250) _ (0.4644) rad 8 Z }~ 5.59 radi 508) ~ 5.39radim* =5.59rad/m. B=o/uy for which the smallest positive solutionis 8.3 em (with = 0). a Problem 2.25 A lossless transmission line is terminated in a short circuit. How long (in wavelengths) should the line be in order for it to appear as an open cixenit at its input terminals? Solution: From Eq. (2.68), Z% = jZotanpil. 1f Bl = (x/2+ nm), then Zz = joo (2). Hence, ite 2e This is evident from Figure 2.15(@). a Problem 2.26 The inpat impedance of a 31-cm-long lossless transmission line of ‘uaknown characteristic impedance was measured at i MHz. With the line terminated jn a short circuit, the measurement yielded an input impedance egitivalent to an inductor with inductance of 0.128 sii, and when the line was open circuited, the ‘measurement yielded an input impedance equivalent to a capacitor with capacitance of 20 pF. Find Z of the line, the phase velocity, and the relative permittivity of the insulating material. Solution: Now @= 2nf = 6.28 x 10 rad/s, so ZB = job = J2nx 108% 0.128 x 10-5 = j0.8080 34 CHAPTER 2 and Ze = 1/joC = 1/( jam x 10° 20 x 107?) +3000 2. From Eq. (2.74), Zy = ZZ = /(J0.804)(—j8000G) = 80 O. Using Eq. (2.75), wee a BO um) —ze/ae 6.28 x 108 x 0.31 1.95 x 10° — See = ss tan! ( J 708047 J8000)) (F0.01-+nn) where n> 0 for the plus sign and n> 1 for the minus sign. For n = 0, 1.94x 108 mis = 0.65¢ and &; = (¢/up)* = 1/0.65? = 2.4. For other values Of n, up is very slow and ¢; is unreasonably high. Problem 2.27 A 60-2 resistive load is preceded by ah/4 section of a 50-@ lossless line, which itselfis preceded by another 1/4 section of 100-@ line. What is the input impedance? Solution: The input impedance of the 2./4 section of line closest tothe load is found from Eq. (2.77) ‘The input impedance of the line section closest to the load can be considered as the load impedance of the next section of the line. By reapplying Eq. (2.77), the next section of 2,/4 line is taken into accownt: % _ 100 | 47 aq ———— Zs 2402. Problem 2.28 A 100-MHz FM broadcast station uses a 300-© transmission line between the transmitter and a tower-mounted half-wave dipole antenna. The antenna impedance is 73 @. You are asked to design a quarter-wave transformer 10 match the antenna to the line. (a) Determine the electrical length and characteristic impedance of the quarter- wave section. (©) If the quarter-wave section is a two-wire line with d = 2.5 om, and the spacing ‘between the wires is made of polystyrene with &; = 2.6, determine the physical length of the quarter-wave section and the radivs of the two wire conductors. CHAPTER 2 35 Soluti {a) For a match condition, the input impedance of a load must match that of the transmission line attached to the generator. A line of electrical length 2/4 can be used. From Eq, (2.77), the impedance of such a Hine should be Lo= Vigh, = VIX = 148.2. » A up ce 3x10 a7 a7" Wey” WEexioxie and, from Table 2-2, 120 a a\? a= Bul ($)+ (z) “Ja Hence, a a\? 148Vv2.6 (@+(@-I- SES og, whieb leads to @ ay £)+ () -1=731, and whose solution is /7. 25 em/7.44 = 3.36 mm. Problem 229 A SO-MHz generator with Z, = 50 © is connected to a load Z = (S0— j25) @. The time-average power transferred from the generator into the load is maximum when Zz = Z., where Zj is the complex conjugate of Z,. Toachieve this condition without changing Z,, the effective load impedance can be modified by adding an open-circuited line in series with Z., as shown in Fig. 2-40 (P2.29). If the line’s Za = 100 , determine the shortest length of line (in wavelengths) necessary for satisfying the maximum-power-treasfer condition. Solution: Since the real part of Z;, is equal to Z,, out task is to find J such that the input impedance of the line is Za = + j25 Q, thereby cancelling the imaginary part of Z_ (once Z and the ingot impedance the line are added in series). Hence, using Eq. (2.73), —jl00corB! = 725, 36 ‘Figure P2.29: Transmission-line arrangement for Problem 2.29. 25 cot = — 755 = -0.25, which leads t0 BI = —1.326 or 1.816. Since I cannot be negative, the first solution is discarded. The second solution leads © 1.816 1.816 t= = = 0.2. B Gai) ee Problem 230 _A 50- lossless line of length / = 0.375 connects a 200-Milz generator with ¥, = 150 V and Z, = 50 toa load Z:. Determine the time-domain carrent through the load for: fa) Z = (50~ j50) 2, &) %=509, (©) Z = 0 (short ciresit, Solution: @ a = 0.45 e347, 700-750 Application of Bq, (2.63) gives: 2 = ty | Re Beea 50 [On O° Zoe ieitanBi | ** |50% 7(S0— 750) ean 135 100+ 50) 2. Figure P2.30: Circuit for Problem 2.30(a). ‘Using Eq. (2.66) gives we (&) (am) _ 150(100+ 50) ( 1 * BOF (100+ 750) \ e+ 0.45 e545 “IF =75e78" OW), (1-0.45 e849) = 1.3427" (A), iu) = Ref = ef 34 e-HORAE eHext0hy = 1.34cos(4rx 10% 108.44") (A). 38 CHAPTER 2 ) Z=502, Ties Zax %=50Q, + _ 150x50 L is T= S050 aR O) = Os > _WF 7S = 8 nse Ly see oy 30° e a, y(t) = ef1 Se eM) = 1 Scos(4 x 10% — 135°) (A). © ) = jun 3s 1 ee rs n= Fae ae (a), —f50 ©), eS (W), y(t) = 3e0s(4n x 10° 135°) (A). Section 2-8: Power Flow on Lossless Line Problem 231 A generator with ¥, = 100 V and Z, = 50 2 is comnected to a load Z, = 75 Q through a 50-2 lossless live of length [ = 0.152. (a) Compute Zn, the input impedance of the line at the generator end. (b) Compute j and (©) Compute the time-average power delivered tothe tine, Pa = 4Re[ lt. (@ Compare %, K, and the time-average power delivered to the load, A. = 1Me[V. if]. How does Pa compare to FL? Explain. (©) Compute the time average power delivered by the generator, Pp, and the time average power dissipated in Z,. Is conservation of power satisfied? Sclution: 39 152 Generator Kk 150.154 >} zoel z=0 Figure P2.31: Circuit for Problem 2.31 @ p= xo1snase, _ 7, [Ze iZotenBr 75-4 j50ran54' Zoe eee Br suas i Z Zo 72 eal Be aa (41.25 - f16.35) 2. @) 40 CHAPTER 2 © oe Sel] - poelar 860 48 y 1 98e- FOIE] = STBEH1LO8 A o(01.60") = 24 CW). @ _%.-%o _ 75-50 _ Tam)” 1545070 eee (1 47866-9149 ise = apem) = eT HL=Ve(14T) = 50eP"(1+0.2) = 60e" (W), ve eal _ —jsa° te (1-0.2) = 0.8e" @, A= prelfili]= srelo0e" x08e%]=24 (W). A, = Pa, which is as expected because the Tine is lossless; power input to the line ends up in the load © Power delivered by generator: 54c0s( 10.16") = 53.15 CW), peli SR e100 x 1.08e" Power dissipated in Z, Pag = Selle = Lora] = HAPS = 3 (0.08)? x50= 29.15 3.15 W. Note 1: Py = Pig + Pa Problem 232 If the two-antemna configuration shown in Fig. 2-41 (P2.32) is connected to a generator with ¥, = 250 V and Z, = 50 2, how much average power is delivered to each antenna? Solution: Since Hine 2 is 2/2 in length, the input impedance is the same as Zz, =75Q. The same is true for line 3. At junction C-D, we now have two 75-2 impedances in parallel, whose combination is 75/2 = 37.5 Line 1 is 2/2 long. Hence at A~C, input impedance of line 1 is 37.5 Q, and rete 20 “" Ze+ Zn 504375 86 (A), ‘Figure P2.32: Antenna configuration for Problem 2.32. (2.86)? x37.5 —~2z Pra = SULA) sre Z] = 153.37 (W). ‘This is divided equally between the two antennas. Hence, each antenna receives 45531 = 76.68 (W). BS = 76. Problem 2.33 For the circuit shown in Fig. 2-42 (P233), calculate the average incident power, the average reflected power, and the average power transmitted into the infinite 100-Q line. “The 2/2 line is lossless and the infinitely long line is slightly lossy. (Hint: The inpat impedance of an infinitely long line is equal to its characteristic impedance so long es & #0.) Solution: Considering the semi-infinite transmission line as equivalent to a load (ince all power sent down the line is lost to the rest of the circuit), Z = Z; = 100.2. Since the feed line is 7/2 in length, Eq. (2.76) gives Zq = Z = 100 © and Bl = (2n/2)(A/2) = x, so e*/8! = —1. From Eq. (2.49a), ZZ _ 100- Z+Z 100+50~ 3° 2 CHAPTER 2 Phy i Po Py Figure P2.33: Line terminated in an infinite line, ‘Also, converting the generator to a phasor gives ¥, = 2e/®° (V). Plugging all these results into Eq. (2.66), vie = (Yee ( i ) = (te oe 0 \ ZtZe ) Ve eben / ~ 304100) \(-1)4+ H-D, = rei" = 1 (W). From Eqs. (2.84), (2.85), and (2.86), Ve? _ [erie "WZ 2x50 2 pi =-iree, =-|2] x10 mw= 1.1 mw, Pips Poy = Py FRE & = 10.0 mW, 3 P. 10.0 mW — 1.1 mW = 8.9 mW. a Problem 2.34 An antenna with a load impedance Zi, = (75~+ j25) Sis connected to a transmitter through a 50-@ lossless transmission line. If under matched conditions (50-0 load), the transmitter can deliver 10 W to the load, how much power does it deliver to the antenna? Assume Zz = Zo. CHAPTER 2 43 Solution: From Eqs. (2.66) and (2.61), ve= (SE) (oa) Tzo[(1e re PBV/A=Te AB) eri Zo + Zo [(1 + Te FB) I — Ten Paty] 14 Te“ Fh! Tyen Ware) pres) = Vee Ta Te PB) Fee) itl, ‘Thus, in Eq. (2.86), Wye iB 7,2 Vee 2) — (Vel 2 a= ir?) = 20a - in). (a= rh) = 3,0“) ‘Under the matched condition, |T] = 0 and A. = 10 W, so |V_l?/8Z = 10 W. ‘When Z, = (75+ j25) 2, from Eq, (2.492), L-%m Z+Zo = MP aire a= Bram = 15 4 j25) 2-502 + 725) 24502 0.27726 $0 Py = 10 W (1- |) ena 10 W (1-0.2777) = 9.23 W. Section 2-9: Smith Chart Problem 235 Use the Smith chart to find the reflection coefficient corresponding toa load impedance: @) Z.=3%, ) Z=(2-2))Z © B= -2i%, @ Z_=0 (short circuit). Solution: Refer to Fig. P2.36. (@) Poim A is x, = 3+ j0. se () Point Bis 2, = (© Point Cis 2, @ Poim Disa, 44 CHAPTER 2 Figure P2.35: Solution of Problem 2.35, en Problem 2.36 Use the Smith chart to find the normalized load impedan corresponding to a reflection coefficient: Solution: Refer to Fig. P2.36. CHAPTER 2 45 (2) Point A’ is () Point Bi is (©) Point C' is @ Point Dis T= 0.327 © Point £' is = Oata =1+j0. © Point F’ ist = jatz=0+ ji. nS Problem 2.37 On a lossless transmission line terminated in a load Z, = 100 2, the standing-wave ratio was measured to be 2.5. Use the Smith chart to find the two possible values of Zp. 46 CHAPTER 2 Solution: Refer to Fig. P2.37. 5 = 2.5 is at point Li and the constant SWR circle is shown. 2 is real at only two places on the SWR circle, at L1, where 25, and 2, where 2, = 1/S=0.4. $0 Zoi = Zi /211 = 100 2/2.5 = 402 and Zon = Z, /zu2 = 10022/0.4 = 250.2. Figure P2.37: Solution of Problem 2.37. SS Problem 238 A lossless 50-© transmission line is terminated in a load with Z,_ = (30+ j25) Q. Use the Smith chart to find (@) the reflection coefficient I, (b) the standing-wave ratio, (©) the input impedance at 0.352 from the load, CHAPTER 2 a7 (@) the input admittance at 0.35%. from the load, (©) the shortest line length for which the input impedance is purely resistive, (the position ofthe first voltage maximum from the load. Figure P2.38: Solution of Problem 2.38. Solution: Refer to Fig. P238. The normalized impedance (50425) 2 302 = 14505 is at point Z-LOAD. (2) T= 0.246760" The angle of the reflection coefficient is read of that scale at the point &. 48 CHAPTER 2 (b) Atthe point SWR: $= 1.64. (©) Zqis0.3502 from the load, which is at 0.1442 on the wavelengths to generator scale. So point Z-IN is at 0.144% + 0.3502 = 0.4942 on the WTG scale. At point ZAIN: Zin = Zino = (0.61 — j0.022) x 50 2 = (30.5 — 1.09) 2. (a) At the point on the SWR circle opposite Z-IN. = (32.74 j1.17) ms. (e) Traveling from the point Z-LOAD in the direction of the generator (clockwise), the SWR ciscle crosses the x, = O line firs at the point SWR. To travel from Z-LOAD to SWR one must travel 0.2502—0.144%= 0.1062. (Readings are on the waveleagths to generator scale.) So the shortest line length would be 0.1062. © The voltage max occurs at point SWR. From the previous part, this occurs at 081. a Problem 239 A lossless 50-0 wansmission line is terminated in a short circuit. Use the Smith chart to find (2) the input impedance at a distance 2.32 from the load, (b) the distance from the load at which the input admittance is Yq = —j0.04 S. Solution: Refer to Fig. P2.39. (a) For a short, zig = 0+ j0. This is point Z-SHORT and is at 0.0002 on the WTG scale. Since a lossless line repeats every 4/2, traveling 2.3) toward the generator is equivalent to traveling 0.32 toward the generator. This pointis at A : Z-IN, and pq = ZinZo = (0-73.08) x50 Q= ~j154Q. (b) The adminance of a short is at point ¥-SHORT and is at 0.2502 on the WIG seale: Yin = YinZo = — 0.04 Sx 50 Q= —j2, which is point B = Y-IN and is at 0.3242 0a the WTG scale. Therefore, the line length is 0.324. 0.2502 = 0.0742. Any integer half wavelengths farther is also valid. CHAPTER 2 49 Figure P2.39: Solution of Problem 2.39. Problem 2.40 Use the Smith chart to find y, if. = 1.5 j0.7. Solution: Refer to Fig. P2.40. The point Z represents 1.5— j0.7. The reciprocal of point Z is at point ¥, which is at 0.55 + j0.26. 50 CHAPTER 2 Figure P2.40: Solution of Problem 2.40. i Problem 241 A lossless 100-0 transmission line 34/8 in length is terminated in ‘an unknown impedance. If the input impedance is Zip = —j2.5 2, (@) use the Smith chart to find Z,. (6) What length of open-circuit line could be used to replace Z:? Solution: Refer to Fig. P2.41. zn = Zin/Zo = —j2.5 0/100 0 = 0.0— j0.025 whieh 4s at point Z-IN and is at 0.004X on the wavelengths to load scale. (@) Point Z-LOAD is 0.375). toward the load from the end of the line. Thus, on the ‘wavelength to load scale, itis at 0.004) +0.3752= 0.3792. Zp = (0+ j0.95) x 100 & = 7952. a CHAPTER 2 st Figure P2.41: Solution of Problem 2.41. (®) An open circuit is located at point Z-OPEN, which is at 0.250% on the wavelength to load scale. Therefore, an open circuited line with Z, = — 0.025 must have a lengta of 0.2502 — 0.0042 = 0.246. Problem 242° A 75-Glossless line is 0.6% long. IFS ‘Smith chart to find [T7, Z, and Zin. Solution: Refer to Fig. P2.42. The SWR circle must pass through $ = 1.8 at point SWR. A circle of this radius has 5. m=3 52 CHAPTER 2 Figure P2.42: Solution of Problem 2.42. “The load must have a reflection coefficient with @, = —60°. The angle of the refiection coefficient is read off that scale at the point @;. The intersection of the cizcle of ‘constant |F] and the line of constant @, is at the load, point Z-LOAD, wihich has 2 value 2, = 1.15 j0.62. Thus, Zi, = Zp = (1.15 — 0.62)x75 = (86.5— 46.6) 2. A O.6h line is equivalent to 2 0.1A line. On the WTG scale, Z-LOAD is at 0.3332, so Z-INis at 0.333A-+ 0.1002 = 0.4332. and has a value tin = 0.63 — j0.29. CHAPTER2 33 ‘Therefore Zin = zinZo = (0.63 — j0.29)x 75 2= (47.0~ j21.8) 2 — Problem 243 Using a slotted line on a 50-@ air spaced lossless line, the following measurements were obtained: S = 1.6, |V|max occurred only at 10 cm and 24cm from the load. Use the Smith chart to find Z. Figure P2.43: Solution of Problem 2.43. Solution: Refer to Fig. P2.43. The point SWR denotes the fact that S = 1.6. ‘This point is also the location of a voltage maximum. From the knowledge of the locations of adjacent maxima we can determine that 2. = 2(24 em— 10 em) = 28 em. “Therefore, the load is 19822 = 0.3572 from the first voltage maxiesurn, which is at 0.250%.on the WTL scale. Traveling this far on the SWR circle we find point Z-LOAD 54 CHAPTER 2 at 0.250% +0.357A— 0.5002 = 0.107. 0n the WTL scale, and here 4. = 0.82— 0.39. “Therefore 2. = x1Zo = (0.82— j0.39) x 50.2 = (41.0- j19.5) 2. ee ‘Problem 2.44 At an operating frequency of 5 GHz, a 50-Q lossless coaxial line with insulating material having a relative permittivity €, = 2.25 is terminated in an antenna with an impedance Z, = 75 Q. Use the Smith chart to find Zia. The line length is 30 em. ‘Solution: To use the Smith chart the line length must be converted into wavelengths. Since B = 2n/Aand up = @/8, am _ Im __¢___3x10 ms _ Bo Ves V225x(5x 10 Hz) SA. Since this is an integral number of half wavelengths, =0.04m, Zn 2h = 152. —_ Section 2-10: Impedance Matching Problem 2.45 A 50-2 lossless line 0.6% long is terminated im a load with Z, = (50+ j25) Q. At 0.32 from the load, a resistor with resistance R = 30 Qis connected as shown in Fig. 2-43 (P2.45(a)). Use the Smith chart to find Zin. Zn H=502 B02 ZH=302 0.3) $+ 03k Z. = 60+ j25)2 Figure P2.45: (a) Circuit for Problem 2.45. CHAPTER 2 Figure P2.48: (b) Solution of Problem 2.45. Solution: Refer to Fig. P2.45(b). Since the 30-C resistoris in parallel with the input ‘impedance at that point, itis advantageous to convert all quantities to admittances. (50+ j25) 2 é COPE is os and is located at point Z-LOAD. The corresponding normalized load admittance is at point ¥-LOAD, which is at 0.3942 on the WTG seale. The inpat admittance of the load only at the shunt conductor is at 0.3842.+0.300A — 0.500% = 0.1942 and is denoted by point A. Ithas a value of Jina = 137+ 70.43. 56 CHAPTER 2 ‘The shunt conductance has a normalized conductance 502 = 5gg 71. ‘The nomnalized admittance of the shunt conductance in parallel with the input ‘admittance of the load is the sum of their admittances: ying = B+ Yin = 1.674 1.374 j0.45 = 3.04-+ j0.45 and is located at point B. On the WTG scale, point is at 0.2420. The input adminance of the entire circuit is at 0.2424+ 0.3002 — 0.5002 = 0.042 and is denoted by point ¥-IN. The corresponding normalized input impedance is at ZN and has a value of mq 19- fA ‘Thus, Zin = tinZo = (1.9— jl.4) x 50.2 = (95 - 770) Q. —— Problem 2.46 A 50-02 lossless line is to be matched to an antenna with Z. = (15— j20)2 using a shorted stub. Use the Smith chart to determine the stub length and the distance ‘between the antenna and the stub. Solution: Refer to Fig. P2.46(a) and Fig. P2.46(b), which represent two different solutions, a 2% and is located at point Z-LOAD in both figures. Since it is advantageous to work in ‘admittance coordinates, yy, is plotted as point Y-LOAD in both figures. Y-LOAD is at 0.0412. on the WTG scale. For the first solution in Fig. P2.46(2), point Y-LOAD-IN-1 represents the point at which g = 1 on the SWR circle of the load. Y-LOAD-IN-1 is at 0.1452 on the WTG scale, so the stub should be located at 0.1454 ~ 0.0412, = 0.104A from the load (or some multiple of a half wavelength further). At Y-LOAD-IN-1, b = 0.52, so a stub with an input admittance of yeut = 0— j0.52 is required. This point is Y-STUB-IN-I and is at 0.4234 on the WTG scale. The short circuit admittance 7 Figure P2.46: (a) First solution to Problem 2.46. is denoted by point Y-SHT, located at 0.2502. Therefore, the short stub must be (0.423A — 0.2504 = 0.1732 long (or some multiple of a half wavelength longes). For the second solution in Fig. P2.46(b), point Y-LOAD-IN-2 represents the point at which g = 1 on the SWR circle of the load. Y-LOAD-IN-2 is at 0.355). on the WTG scale, so the stub should be located at 0.355) — 0.0414 = 0.3142 from the Joad (or some multiple of half wavelength further). At ¥-LOAD-IN-2, b = ~0.52, so a stub with an input edmittance of Yau = 0+ j0.52 is required. This point is Y.STUB-IN-2 and is at 0.0772 on the WTG scale. The short circuit admittance is denoted by point Y-SHT, located at 0.2502. Therefore, the short stub must be 0.077 —0.250-+ 0.5002 — 0.3272 long (or some multiple of a balf wavelength 38 CHAPTER 2 Figure P2.46: (b) Second solution to Problem 2.46. longer). a Problem 247 Repeat Problem 2.46 fora load with Z, = (100+ 50) & Solution: Refer to Fig. ?2.47(a) and Fig, P2A7(G), which represent two different solutions. Fa _ 1004 j509 _ a= Z- 50D end is located at point Z-LOAD in both figures. Since it is advantageous to work im cdmitiance coordinates, yy is potiedas point ¥-LOAD in both figures. ¥-LOAD is at 0.4632 on the WTG scale. +t 59 Figure P2.47: (a) Fizst solution to Problem 2.47. For the first solution in Fig. P2.47(a), point Y-LOAD-IN-I represents the point at which g = 1 on the SWR circle of the load. Y-LOAD-IN-I is at 0.1624 on the WTG scale, so the stub should be located at 0.1622. — 0.4634 +0.500A = 0.199% from the load (or some multiple of a half wavelength further). At Y-LOAD-IN-1, b= 1, so a stub with an input admittance of Yani) = O— fl is required. This point is Y-STUB-IN-1 and is at 0.375A on the WTG scale. The short circuit admittance is denoted by point ¥-SHT, located at 0.2502. Therefore, the short stub must be 0.375A—0.250A = 0.125). long (or some multiple of a half wavelength longer). For the second solution in Fig. P2.47(b), point ¥-LOAD-IN-2 represents the point at which g = 1 on the SWR circle of the Joad. Y-LOAD-IN-2 is at 0.3382. on the 60 CHAPTER 2 Figure P2.47: (b) Second solution to Problem 2.47. WTG scale, s0 the stub should be located at 0.3382 — 0.4632. + 0.500A = 0.3752 from the load (or some multiple of a half wavelength further). At Y-LOAD-IN-2, b= —1, soa stub with an input admittance of Yenvp = 0-+ jl is required. This point is ¥-STUB-IN-2 and is at 0.1254 on the WTG scale. The shor circuit admittance is denoted by point ¥-SHT, located at 0.2501. Therefore, the short stub must be 0.1252 — 0.2502-+ 0.5001 = 0.3752. long (or some mukiple of a half wavelength longer I Problem 2.48 Use the Smith chart to find Ziq of the feed line shown in Fig. 2-44 (F2.48(a)). All lines are lossless with Zp = 50 Q. 61 Z,= (60+ 50) 2 Zq= (50 j50) 2 Figure P2.48: (a) Circuit of Problem 2.48. Solution: Refer to Fig. P2.48). _ Zi _ 50+ j509 - neZe oq sta and is at point Z-LOAD-1. B and is at point Z-LOAD-2. Since at the junction the lines are in parallel, it is advantageous to solve the problem using admitiances. yy is point Y-LOAD-1, which is at 0.412A on the WTG scale. yz is point Y-LOAD-2, which is at 0.088A on the ‘WTG scale. Traveling 0.3002 from Y-LOAD-1 toward the generator one obtains the input admittance for the upper feed line, point Y-IN-1, with a value of 1.97+ j1.02. Since traveling 0.7002 is equivalent to traveling 0.2002 on any transmission Line, the input admittance for the lower line feed is found at point Y-IN-2, which has a valve of 1.97 j1.02. The admittance of the two lines together is the sum of their admaittances: 1.97 + 1.02+ 1.97 — j1.02 = 3.94+ j0 and is denoted Y-JUNCT. 0.3002 from ¥-JUNCT toward the generator is the input admittance of the entire feed line, point ¥-IN, from which Z-IN is found. Zig = tao = (1.65 — j1.79) x 50 2 = (82.5 — j89.5) Q. 62 CHAPTER 2 y Figure P2.48: (b) Solution of Problem 2.48. Problem 2.49 Repeat Problem 2.48 for the case where all three transmission lines are 1/4 in length. Solution: Since the transmission lines are in parallel, it is advantageous to express loads in terms of admittances. In the upper branch, which is a quarter wave line, B_a Ne=3=F- hey =F CHAPTER 2 63 and similarly forthe Jower branch, % 2 he= Eee ‘Thus, the total load at the junction is Z+Ze Wier = Yin + Yom = @ ‘Therefore, since the common transmission line is also quarter-wave, Zin = BB [Zscr = BYjer = Zi + Ze = (50+ j50) 2+ (50— 750) 2 = 100 2. ne Section 2-11: Transients on Transmission Lines Problem 2.50 Generate 2 bounce diagram for the voltage V(z,t) for a i-m long lossless line characterized by Zp = 50 Q and uy = 2c/3 (where c is the velocity of light) if the line is fed by a step voltage applied at r = 0 by a generator circuit with Vg = 60 V and Ry = 100.0. The line is terminated in a load Z, = 25 ©. Use the ounce diagram to plot V(r) at 2 point midway along the length of the line froms tor= 25 ns. Solution: From Eq. (2.1246), Also, ‘The bounce diagram is shown in Fig. P2.50(2) and the plot of V(r) in Fig. P2.50@). od Figure P2.50: (2) Bounce diagram for Problem 2.50. voO.sm0 tas) 3 wo 15 OOS Figure P2.50: (b) Time response of voltage. CHAPTER 2 65 a Problem 251 Repeat Problem 2.50 for the current / on the line. Solution: i t r Figure P2.51: (a) Bounce diagram for Problem 2.51. 66 CHAPTER 2 105m, | | Ld 1 (as) 5 10 i 2) 5 3A 0489 A 9.474 a 0479 A 4al ‘Figure P2.51: (b) Time response of current. EE Problem 2.52 In response to a step voltage, the voltage waveform shown in Fig. 2-45 (P2.52) was observed at the sending end of a lossless transmission line with Ry = 502, Zy = 50 ©, and & = 2.25. Determine (a) the generator voltage, (b) the length of the line, and (c) the load impedance. vO, sv 3v ° ous Figure P2.52: Observed voltage at sending end. Solution: (@) From the figure, Vi" ‘V. Applying Eq. (2.124), ve 3 67 3x108 2x 108 pws. The first change in the wavefor = is. The first change ym occurs Astty > r S10 ot ceo jem 2.53 In response to a step voltage, the voltage waveform shown in Fig. 46 (F253) was observed 2t the sending end of a shorted line with Zo = 50. and vo.) 12V- BV o15v ° Tus Mus Figure P2.53: Observed voltage at sending end. 525 m. cy CHAPTER 2 From the voltage waveform, V;* = 12 V. Att = Tus, the voltage at the sending end is V(z= 0,t = Tus) = Ve FTL + TEDL TyV¢ (because Ty. = -1). ‘Hence, 3 V= Tx 12Vorlg = a=2(32)=9( 535) 0.25. From Eq. (2.128), 302. 50 =n or 2 30550" which gives Vg = 19.2V. a Problem 2.54 Suppose the voltage waveform shown in Fig. 2-45 was observed at the sending end of a 50-02 transmission line in response to a step voltage introduced by a generator with V, = 15 V and an unknown series resistance Rg. The line is 1 km in length, its velocity of propagation is 1 x 10° ms, and it is terminated in a load Z= 1002. (@) Determine Ry. (&) Explain why the drop in level of V(0,1) atz = 6 us cannot be due to reflection ‘from the load. (©) Determine the shunt resistance Rr and the location of the fault responsible for the observed waveform. ‘Solution: v0.0 ; sv a a Ly 0 ous Figure P2.54: Observed voltage at sending end. 69 _ 15x50 ~ Rg +50" ich gives R, = 100.2. Roundtrip time delay of pulse return from the load is aw 2x10 Fest xi Hp much longer than 6 ps, the instance at which V(0,¢) drops in level. '5:V. Thenew level of 3 V is equal to V;* plus Vj due to shunt resistance: Vi+vy =545P=3 255 A generator circuit with Vz = 200 V and R, 75-Q lossless line with a rectangular pulse of duration long, its uy = 2x 10° mys, and it is terminated in a load Z = 125 2. Synthesize the voltage pulse exciting the line as the sum of two step fumetions, = Ma) end ¥,, (1). “For each voltage step function, generate a bounce diagram for the voltage en | theline. ‘the bounce diagrams to plot the total voltage at the sending end of the line. Ve{t) = Vaile) + Ven(t)> 200U(t) W), 200U(t—0.4 us) (V). 70 CHAPTER 2 2Q 50 a — 200 V' H=BQ Fl1wQ F< 200 m —>f ‘Figure P2.55: (a) Circuit for Problem 2.55. vo ¥ef0) Figure P2.55: (b) Solution of part (a). ®) dogg oj up = 2xto8 = 1 ‘We will divide the problem into two parts, one for Vp,(#) and another for Vg, (¢) and ‘then we will use superposition to determine the solution for the sum. The solution for Vq,(t) will mimic the solution for V,,(t), except for a reversal in sign anda delay by 0.4 us. For Vg,(t) = 200U(1): Ry-Zy 25-75 _ Te Rim Br A-%H _ 125 he iene ~ BtZ CHAPTER 2 yr a Mie _ 200x75 1" RgtZo 25475 Ved 200 x 125 Reh Bris @Vi(0,2) at sending end dve to Vp, (¢): = 150V, = 166.67 V. Vee0 1s aps = ys | sus ous : : Figure P2.55: (c) Bounce diagram for voltage in reaction to Vz,(t). n 72 Gi) ¥a(0,1) at sending end due to Vg,(t): Ye roned <= 200m ans zee satis asus 54s CHAPTER 2 2B &) @Vi(0.1) at sending end dae to Ve, (2): ¥4(0,1) 161.58 150v. a a a) 2 4 6 Figure P2.55: (@) ¥i(0,2). (ii) V2(0,1) at sending end: ¥4(0,2) + oe é 44 4 il § 20) “150 2168.75 -166.41 Figure P2.55: ( V2(0,t). 74 ii) Net voltage V(0,t) = 1 (0.1) + Va(0,t): v0.1) 1s0v 1875 | 444 028 oa 2 28 6 64 fe - 234 ‘ Figure P2.55: (g) Net voltage V(0,1) —_ Problem 2.56 For the circuit of Problem 2.55, generate a bounce diagram for the current and plot its time history at the middle of the line. Solution: Using the values for I’; and I calculated in Problem 2.55, we reverse their signs when using them to constrict a bounce diagram for the current. 15 Figure P2.56: (a) Bounce diagram for /y(t) in reaction t0 Vz,(¢)- 7 CHAPTER 2 1 so Aus > “— rts tps 3.90mA Figure P2.56: (b) Bounce diagram for current /h(¢) in reaction to ¥,,(7). CHAPTER2 7 @A(1/2,2) due v0 Ve, (2): 1,0300,2) Gi) B(L/2,1) due t0 Ve,( T,( 100, #) LS igs 13125 Pa et Ef 05 15 25 35 Figure P2.56: (6) 4(2/2,¢). Figure P2.56: (6) A(I/2,1). 8 CHAPTER 2 (Gi) Net current i(1/2,t) = A(I/2,t) + B(U/2,t): 101) 15 19 25 25 0625 05 08 os 0s —> #(hs) 035 3: (e) Total KL/2,1). CHAPTER 3 9 Chapter 3 Section 3-1: Vector Algebra Problem 3.1. Vector A starts at point (1,— unit vector in the direction of A. Solution: ~2} and ends at point (2,—1,0). Find 2(2-1) + H-1-(-1)) +2(0-(-2)) = 8 [Al = vI¥4 = 2.24, g-A _%4+2 Jal 2.24 = 20.45 +20.89. Problem32 Given vectors A=#2—J3+%, B= 22-9423, and C= 84+52-22, show that Cis perpendicular to both A and B. Solution: C= (82~$342)-(84492-22) C= (82-9 +23) -(24-4+92-22) =8-6-2=0, 8-2-6=0. Problem 33 In Cartesian coordinates, the three comers of a triangle are P;(0,2.2), A(2,=2,2),and Ps(1,1,—2). Find the area of the tiangle. Solution: Let B = Pj, = 22—}4 and C= AP = 8—9—24 represent two sides of the triangle. Since the magnitnde of the cross productis the area of the parallelogram (see the definition of cross product in Section 3-1.4), half of this is the area of the triangle: 3/B x Cl= 3I(82—-94) x (2-9 -24)]| 318(—4)(—4) + (2-4) +42-1) ~ (-4)1)} 419164 98+42| = $V1G 48242? = 150 = where the cross product is evaluated with Eq. (3.27). Problem 3.4 Given A = &2—93421 and B= 8B, +§2+2B,: (2) find B, and B, if Ais parallel to B; (®) find a relation between B, and B, if A is perpendicular to B. 80 CHAPTER 3 Sointion: (@) If A is parallel to B, then their directions are equal or opposite: Sa = 8p, oF A/\Al = +B/(B), 2-342 _ $B. + 92408. Vie Vas BEBE From the y-component, =3 #2 Via” ae BeE which can only be solved for the minus sign (which means that A and B must point in opposite directions for them to be parallel). Solving for Bz +B?, “This is consistent with our result for 22 + £2 “These results could also have been obtained by assuming @xz was O° or 180° and solving |A||B| = £4-B, or by solving A xB = 0. (b) If A is perpendicular to B, then their dot product is zero (see Section 3-1.4). Using Eq. 3.17), 0=4-B=2B.-64+8,, B,= 6-2B;. “There are an infinite number of vectors which could be B and be perpendicularto A, ‘but their x- and z-components must satisfy this relation. ‘This resalt could have also been obtained by assuming Q4g = 90° and calculating 94, and C = §3—24, find 81 CHAPTER 3 (@) Amd, (©) the component of B along C, (©) Sac. @ Ax, (©) A(BxC), @ Ax(BxC), @ &xB, and a) (Ax 9)-2 Solution: (a) From Eq. (3.4), A= Ps24(3)/ evi, and, from Eq. (3.5), 2492-8 rr () The component of B along C (see Section 3-1.4) is given by Beos0xc = BE (© From Eq. 3.21), AC 1 6412 _ gt 18 Lasgs, me 0 = TRB STR () From Eq. (3.27), AxC=8(2(—4)~ (~3)3)+9((-3)0~ (4)) +20) O(-3)) = 84944 23. (@) From Eq. (3.27) and Eq. (3.17), A-(BxC)=A-(216+.912429) = (16) +2(12) + (-3)9 = 13. Eq. (3.30) could also have been used in the solution. Also, Eq. (3.29) could be used in conjumetion with the result of part (@). (® By repeated application of Eq. (3.27), Ax (BX C) =A x (164912 +29) = 854-957-220, Eq, (3.33) could also have been used. 82 CHAPTER 3 (@ From Eq. (3.27), &xB= (@) From Eq. (3.27) and Eq. 3.17), (Ax$)-2=(342)-2=1 Eq. (3.29) and Eq. (3.25) could also have been used in the solution. Sn rnnnnree ne Problem 3.6 Given vectors A = 22-3423 and B = %3—22, find a vector C whose magnitude is 6 and whose direction is perpendicular to both A and B. Solution: The cross product of two vectors produces a new vector which is Perpendicular to both of the original vectors. ‘Two vectors exist which have 2 ‘magnitude of 6 and are orthogonal to both A and B: one which is 6 units long in the direction of the unit vector parallel to A x'B, and one in the opposite direction. AxB _¢ (2-9+83)x (93-22) IAxB] © “"[(@2-$+33)x(B—2)) 524913423 2 w £(80.89+95.78 421.33) VEaESE (20.89 + 95.78 +: ). i Problem37 Given A = 2(2++3y)—9(2y+32)+2(3x—y), determine a unit vector parallel to A at point P(1,—1,2). Solution: The unit vector parallel to A = £(2x+3y)—9(2y-+3z)+%(3x—y) at the point P(1,~1,2) is A(1,—1,2) ~i5 4m TRa 844% 50.17 507042070. B01 oye caee | VB eee Problem 3.8 By expansion in Cartesian coordinates, prove: (2) the relation for the scalar tiple product given by (3.29), and (©) the relation for the vector triple product given by (3.33). Solution: (@) Proof of the scalar tiple product given by Eq. (3.29): From Eq. 3.27), AXB = 8(A,B,—A,By)+H{ABy — ApBe) +2(A.B, —A,B,). C= 56 = +6 CHAPTER 3 83 Bx C= 2(ByC— BG) + (BC. — BC) + UBC) ~ ByGe), Cx A= 8(GA,— CAy) + 9( GAs — GAs) + 2(GAy—GAr)- Employing Eq. (3-17), itis easily shown that Ax(ByC; — B-C,) +Ay(B.Cy — BxCz) + Ar(BxC, — Bye), 3 GyAs — Cy) + By (Cos — CxA,) +BAGoAy— GAs), ©-(A XB) = CAB ABs) + GAB AcBe) + ClAsBy ~AyBr)s which are all the same. (b) Proof of the vector triple product given by Eq. (3.33): The evaluation of the left hand side employs the expression above for B x C with Eq. (3.27): Ax (8(ByC.~ BiG) + 9(B:Ce— BsC.) + (Bs, — ByCz)) (Ay( BC —ByC,) ~A.(B.C. ~ B.C) + HAMByC.— BG) —As(BrGy—ByCz)) +H AMB:G— B.C.) —Ay(ByCe-B.G)), ‘while the right hand side, evaluated with the aid of Eq. (3.17), is AXx(Bxe B(A-C)—C(A-B) = BAG. +AyG +A,Cz) — C(AsBs + AyBy + A282) (Bal AyCy + AsCz) ~ Col AyBy + ABs) $9( BAG +A.C)— GAB. + AB.) + 2(By(ArCet AyCy) ~ C{AsBs + AyB,)). By rearranging the expressions for the components, the left hand side is equal to the right hand side. Problem 3.9 Find an expression for the unit vector directed toward the origin from an arbitrary point on the line described by x= 1 and z= 2. Solution: An arbitrary point on the given line is (1,y,2)- The vector from this point to (0,0,0)is: A=2(0-1) +5(0-y)+2(0-2) = -2- $y 28, lal= Vises? = V55¥, A 2 eal al 84 CHAPTER 3 EE Problem 3.10 Find an expression for the unit vector directed toward the point P located on the z-axis at a height h above the x-y plane from an arbitrary point Ofe,y.2) in the plane z= 2. Solution: Point P is at (0,0,h). Vector A from O(xy,2) t0 P(0,0,h)is: A= 8(0-x)+§(0-y)+3(h-2) = —Be— Fy A(h-2), (P+ +(h-2y7, —tx—Py+2(h—2) ty + h-2P)? —_— Problem 3.11 Find a unit vector parallel to either direction of the line described by de-z=4. Solution: First, we find any two points on the given line. Since the line equation is not a function of y, the given line is in a plane parallel to the x-z plane. For convenience, we choose the x-z plane with y= 0. For. —4, Hence, point P is at (0,0,—4). Forz = 0, x=2. Hence, point Qs at (2,0,0). ‘Vector A from P 10 Q is: 2-0) +9(0-0)+2(0+4) = 22424, Rt v20 ‘Problem 3.12 Two lines in the x-y plane are described by the expressions: Line) x+2y=-6, Line? 3xb4y=8. Use vector algebra to find the smaller angle between the lines at their intersection point, Solution: Intersection point is found by solving the two equations simultaneously: ~2x—4y= 12, Bxt4y=8, CHAPTER 3 85 BEBO SS 29 IS, ‘Figure P3.12: Lines I and 2. ‘The sum gives x= 20, which, when used in the first equation, gives y= —13. Hence, intersection point is (20,—13). Another point on line 1 isx=0, y = —3. Vector A from (0,~3) to (20,~13) is A=2(20)+(-13 +3) = 820-710, [Al = 202+ 10? = V500. A pointon line 2 is x=0, y=2. Vector’B from (0,2) to (20,13) is B=%(20)+H(-13-2 Bl = V20F 15 = Vea. Angle between A and B is #20-915, Problem 3.13 A given line is described by xt2y=4. ‘Vector A starts at the origin and ends at point P on the line such that A is orthogonal to the line. Find an expression for A. tai al ye arn yy 86 CHAPTER 3 Solution: We first plot the given line. Next we find vector B which connects point P\(0,2) to P,(4,0), both of which are on the line: B=X(4—0)+9(0-2) = 24-92. ‘Vector A starts at the origin and ends on the line at P. If the x-coordinate of P is x, Figure P3.13: Given line and vector 4. then its y-coordinate has to be (4—z)/2 in order to be on the line. Hence P is at (x, (4—2)/2). Vector A is se9(42) But A is perpendicular to the line. Hence, A-B=0, [eee (42)] e-sn= 4x—(4—x) =0, oF 4 Z=08. x= 5 =08 Hence, 4-08 assosts( ) =208+926 2 Problem 3.14 Show that, given two vectors A and B, CHAPTER 3 87 {a) the vector C defined as the vector component of B in the direction of A is given by C=8(8-4) where 4 is the unit vector of A, and (®) the veetor D defined as the vector component of B perpendicular to A is given by Solution: (@) By definition, B (B-4) along Ais the component of B along 4. The vector component of A[, A\_ ABA) c=as-a)=A(s. A) = A) 20-075 (im) =F (b) The figure shows vectors A. B, and C, where Cis the projection of B along A. ‘tis clear from the triangle that B=C+D, A(B-A) D=B-c=B-“=N. iAP Figure P3.14: Relationships between vectors A, B,C, and D. 88 CHAPTER 3 Problem3.15 A certain plane is described by 2r43y+4i 16. Find the unit vector normal to the surface in the direction away from the origin. Solution: Procedure: 1, Use the equation for the given plane to find three points, Py, Pz and Ps on the plane. 2, Find vector 4 from P, to P; and vector B from P; to Ps. 3. Cross product of A and B gives a vector C orthogonal to A and B, and hence tothe plane. 4, Check direction of &. Steps: 1, Choose the following three points: 22 (0,0,4), Fat(8,0,0), Prat (0,40). 2. Veetor A from Pi to P, A=4(8-0)+9(0-0)+2(0-4) = 28-24 ‘Vector B from P; to Ps 3=2(0-0)+9 (2-0) +2(0-4) <9 28 C=AxB ‘%(AyB,—A.By) +9 (ArBs— AsBz)+2(AsBy —AyBe) 22 (0-(-4)--4)- 2) 49((-4)-0- 8-0) 48 —~ 6% 128 Frye CHAPTER 3 89 ‘Verify that Cis orthogonal to A and B ACH (*-$) +e2.04 (Pca) BB ao 3 3-c=(0- 4. CHk$+ 932428 C__ 8849324048 10 igh +3e= cay = 20.37 + 70.56 + 20.74. @ points away from the origin as desired. ——— Problem 3.16 Given B= (2 —3y) + §(2x—3z)— a{x+), find a unit veetor parallel to B at point P(1,0,—1). Solution: At P(1,0,-1), B=%(-2)+9(2+3)-A(1) fo 8 ee Bl V4+25+1 Problem 3.17 When sketching or demonstrating the spatial variation of a vector field, we often use arrows, as in Fig. 3-25 (P3.17), wherein the length of the arrow is made to be proportional to the strength of the field and the direction of the arrow is the same as that of the field’s. The sketch shown in Fig. P3.17, which represents the vector ficld E = fr, consists of arrows pointing radially away from the origin and ‘their lengths increase linearly in proportion to their distance away from the origin. ‘Using this arrow representation, sketch each of the following vector fielés: (© Es=2x43y, @ Es =Sx492y, ©) Es=ér, 90 CHAPTER 3 Figure P3.17: Arrow representation for vector field E = fr (Problem 3.17), Solution: @) P3.17a: CHAPTER 3 93 P3.17f: Eg= Fsing Problem 3.18 Use arrows to sketch each of the following vector fields @eE b) Ex © a @ Ex= feoso, Solution: CHAPTER 3 a8 P3.18d:Ev= Fos — 96 CHAPTER 3 Sections 3-2 and 3-3: Coordinate Systems Problem 3.19 Convert the coordinates of the following points from Cartesian to cylindrical and spherical coordinates: (@) Ai(1,2,0), (®) P(0,0,3), (©) A(1,1,2), © P(-3.3,-3) Solution: Use the “coordinate variables” column in Table 3-2. (@) In the cylindrical coordinate system, P= (VIP +2 tan! (2/1),0) = (V5,1.107 rad, 0) = (2.24,63.4°,0). In the spherical coordinate system, Ra(VP+P 40, tan "(VP 42/0), tan“ (2/1) V5,m/2 rad, 1.107 rad) = (2.24,90.0°,63.4°). ‘Note that in both the cylindrical and spherical coordinates, 6 is in Quadrant I. (b) In the cylindrical coordinate system, P, = (V0? +07, tan“(0/0),3) = (0,0 rad,3) = (0,0°.3). In the spherical coordinate system, Pra (VO+0+3?,tan-* (VP + 07/3), tan™! (0/0)) = (3,0 rad, 0 rad) = (3,0°, Note that in both the cylindrical and spherical coordinates, ¢ is arbitrary and may take any value. (© In the cylindsical coordinate system, Ps = (V1 +12, tan (1/1),2) = (V2,2/4 rad,2) = (1.41,45.0°,2). In the sphetical coordinate system, Pea (VP +P +2 tan! (1? + 12/2),tan-* (1/1) = (V6, 0.616 rad,x/4 rad) = (2.45,35.3°,45.0°). ‘Note that in both the cylindrical and spherical coordinates, $ is in Quadrant I. 7 the cylindrical coordinate system, hay ot pea (C3) +32 /-3)~3) = (GV3,3n/4 rad, —3) © (4.24, 135.0%, -3). coordinate system, —_— Varese (sh te (y 32/—3),tan“1(3/—3)) 5y/3,2.187 tad, 3n/4 rad) Fe (5.20,125.3°, 135.0°). and spherical coordinates, @ a Quadrant I 3) = A(1.41,1.41,-3). y,2) = Py(reosd,rsingyz) = Pi (2cos%.28in3, P2(3,0,0). P3(—4.0,2). ea P,(3c0s0,3sin0,0) ©) Fa(x,9,2) = Ps(4cos,4 sine. Convert the coordinates of the following points from spherical to irical coordinates: =) P.(5,0.n). © AG.n/2,%). = Fi(5sin0,0,Sc0s0) Py(,6,2) = Ai(Rsin@, ,Reos) = A,(0,0,5). @) Pa(s,6,2) = Pe(Ssin0, x,5c080) = Fa(0.%,5). 98 CHAPTER 3 © P(x 0,2) Py(Bsin§ x, 3083) = A3,7,0). Problem 3.22 Use the appropriate expression for the differemtal surface area ds t0 determine the area of each of the following surfaces: @ r=3, OS . Figure P3.24: Outline of section. CHAPTER 3 101 wD pa S= g Rsind9d6 les lp=x/6 ont =4 G L 5 [-coseig”] =4x v= foe oe “1G 2 5-3) -eoselg Problem 3.25 A vector field is given in cylindrical coordinates by E= freose+-grsing +22 Point P(4.x,2)is located on the surface of the cylinder described by r= 4. Atpoint P, find: (@) the vector component of E perpendicular to the cylinder, (®) the vector component of E tangemtial to the cylinder. Solution: . (@) Eq =4(F-E) =iff-(Freoso+Grsing +22)) = froose. ALP(4,5,2), Ey = Peost = ©) E=E-E, =$rsino +: ALP(4,1,2), Ey = S4sinz + 222 = 24. LL Problem 3.26 At a given point in space, vectors A and B ere given in spherical coordinates by A=R4+62-6, B=-R2+§3. Find (@) the scatar component, or projection, of B in the dection of A, () the vector component of B in the direetion of A, (©) the vector component of B perpendicular to A. Solution: 102 CHAPTER 3 (a) Scalar component of B in direction of A: (Ra + 62-4) 1er4e1 =p-ace.* = (-2443)-6 C=B-a=B ia = ¢ R2+63) = ~(R2.09 +6 1.05 — $0.52). (c) Vector component of B perpendicular to A: D=B-C = (—R2+43) +(R2.09 + 61.05 $0.52) 0.09 +61.05+92.48, —_ Problem 3.27 Given vectors A= f(coso+3z)—(2r+4sing) + 2(7—22), find (a) Sap at (2,7/2,0), ‘b) a unit vector perpendicular to both A and B at (2,%/3,1). ‘Solution: It doesn’t matter whether the vectors are evaluated before vector products are calculated, or if the vector products are directly calculated and the general results are evaluated at the specific point in question. (@) At (2,7/2,0), A = 98 +22 and B = -f. From Bq. 3.21), agnor’ (422) =sor (2) aso! () At (2.0/3, 1), 4 = #3 -G4(1 + 4V3) and B = Fh V3+24. Since Ax B is perpendicalar to both A and B, a unit vector perpendicular to both A and B is given (2+ 43) (0.487 + 0.228 + 20.843). 28 Find the distance between the following pairs of points: f Gi2,3) and Px(~2,—3,2) in Cartesian coordinates, 1. n/4,2) and Pe(31%/4,4) i eylindsical coordinates, 2,0) an4 Pe(3:,0) in spherical coordinates. Scant 4 (3 2F Haha 9 Has4 = I= 592 a= [Rt77 -2nrrcos(O2— a1) + (@— ayy? *) +a-a]" foe 1e2x3xixex(G- i = (10-6+4)? = 8? = 2.83. d= {RE + Ri — 2RiRoleose2cosr 4sin®; sin@zcos(d2—- i)” Bake 12 = (944-2x3 x2 [cosmeos 5 + sin sinnecos(0—0)] } = {944-0}? = VIS = 3.61. splem 329 Determine the distance between the following pairs of points: yy PA(1,1,2) and (0.2.2), “t) Ps(2,n/3,1) and Pa(4,2/2,0), (©) Ps(9.,n/2) and Pal 42/2.%) a= VO-1P +Q-12 +(2-2) = V2, _ 0) From Ea. 3.67), ee ante? — 2(2)(4)e0s (3 ~ 2) +(0-3)* = 21 —8V3 = 267. 104 (@) From Eq. 3.68). problem 3.30 Transform the following vectors into cyli vhon evaluaze them at the indicated points: Mindiecal coordin: fay A= R(x+y) at PH(12,3), (&) Be Ry) + 9-H), 2 (7.0.2), () CH RPMF AY) ~ PEM +7) 424 at PA(1,—1,2) sin + 0cos@+$c0s?6 at A(2,n/2,2/4), @D Sos 0 + $08 fq) B= Bcoso+Osind + sin at P,(3,n/2,n), Solution: From Table 3-2: @ A= (2e0sd —Gsing)(rcos$+rsing) = frees (cos + sing) — Grsing(cosg + sing), P= (SPF tan" 1(2/2).3) = (V5,644",3), A(P:) = (70.447 ~ $0.894)V5 (447 + 894) = #134 $2.68, ®) B cos —Gsino)(rsing — e050) + (6c0s0+Fsing)(rcos@— rsing 7(2singcos@— 1) +$r(cos"@—sin®6) = fr(sin26—~ 1) + §reos2o, Py = (VIPS Ota! (0/1),2) = (1,0°,2), BA) +6. © 008d —Gsing) SE? _ Georg ttsingy = aes cind-cos 6(sind ~ cos) ~8(sin*9 4.0054) « 24, )? tan? (—1/1),2) = (V2,—45°,2), @ Dx (Fsind +2c088)sin® + (Feos® — 2sind)cos@ + Gcos%y = F+-Geos?e CHAPTER 3 105 Py = (2sin(/2),1/4,2e08(n/2)) = (2,45°,0), Dir) =F+44. © E= (fsinO + 2cos8)coso+ (Fcos@—Zsin9) sind +4 sin’ 6. a=(Ea), E(Ps) = (sin +20085) cosm+ (eos 5 ~ asin) sinn+$sin? 2 = ~7+6. Problem 3.31 Transform the following vectors into spherical coordinates end then ‘evaluate them at the indicated points: () A=? + fz+H4 a A(1,-1,2), &) B=ai(P4y+2)-He +) a A(-1,0,2), (©) C= Feosd—§sing+fcososing at Pi(2,7/4,2), and @ Daa +y)— H/C ty) +96 a (1,-1.2) Solution: From Table 3-2: @ A= (Rsin6cosd +8 cosBeos$—$sind)(RsinOsine)* + (Rsin€sing +8cos@sind +4.c0s6)(RsinOcos)(Reos9) + (Reosd—6sine)s = R(R* sin” @singcoso(sin@sing +cos8)+4cos6) +8(R?sinScosOsingcoso(sinOsind -+cos8) —4sin8) +4R’ sin@(cos@cos* —sin®sin®6), A= (PSDP tart (verre) s"=1/)) = (v6,35.3°,~45°), A(P;) = R2.856 — 62.888 + §2.123. ®) B = (Rsin6sing +cos0sing +$cos o)R? — (Reos@~6sin@)R* sin20 = RR’ sin€(sind—sin6cos®) + 8R?(cos@ sino + sin?) +4 cos, Re (Vane ca! ( (iF +8/2) sta (0/(-)) = (V5, 26.6°, 180°), 106 CHAPTER 3 B(P)) = —RO0.896 + 60.449 95. © C= (Rsin6 +6.cos6)cosd—§sino+ (Rcos 8 — 8sin®)cososing .cos@(sin8 + cos sind) +8 coso(cos@ —sin€sind) — sino, Ps= (VP42 tan (2/2),n/4) = (2V3,45°,45"), (Ps) = R0.854 + 60.146 — 40.707. © _ 5 . Resin? sin? D= (RsinOcos¢ +6 cos8eos6—Ssing) eam Osi? 6+ Rois? Oca R'sin? cos? - (sindsing + Gece sing +-$e086) se ae Raat oaTS +(Rcos@—6sino)4 = R(sin@cosgsin’ 6 — sin@singcos?¢ + 4e086) +6(cos@cos sin? @—cos @sing cos? ~ 4sin®) ~4(cos? o+sin?6), Pa(1,-1,2) = [ THT FF tan VTFT/2) ca0"\(—1/1)] = Px(v6,35.26°, 45°), D( Ps) = R(sin35.26* cos 45° sin” 45° — sin 35.26" sin(-45°)cos*45* + 4c0s35.26°) +6(cos35.26° cos45* sin? 45° — cos 35.26" sin(—45° cos? 45° — 4sin35.26°) —$(cos* 45° + sin? 45°) =83.67-61.73 40.707. er Sections 3-4 to 3-7: Gradient, Divergence, and Curl Operators Problem 3.32 Find the gradient of the following scalar functions: @ T=2(e+2), ©) Vans, 4x 4 @+2yP @+27 (b) From Eq. (3.72), We SF +92 423072, (©) From Eq. (3.82), aS gazsind_, , cose Wey ier) ee (@) From Eq. (3.83), VW = —Rev® sind +6(e"/R) cos®. (e) From Eq, (3.72), Saxetsy, a, 525, 488 vs= = Ree + f2y- Be, (0 From Eq. (3.82), ¥2rcos—Grsing. (@ From Eq. (3.83), MM = ReosOsing, aM gM ¢™ om ae vu=8 R30 +? Rand Oo ‘ReosOsing — GsinOsing +6 2 108 Problem 333 The grdient ofa salar function T is given by vr ET soa: Solution: , find T(z). VI = fe-%, BY choosing Py at z= 0 and Fat any point 2, (3.76) becomes T()~1(0) = 1 at Follow 2 procedure similar to that leading to Eq. (3.82) 10d = by Eq. (3.83) for V in spherical coordinates From the chain rule and Table 3-2, Vr=38% ar ar a tI Pression Zive, 9(2TaR ara +9 (2 aR ar VRS SS +2 (27 2R aree \BR Se * Se era ara ara 8 2? oe 3, anf 2 fz) OR 9g VOTES SS tan" TP/e 5) F2 an CHAPTER 3 109 aT Rsindcose (aT Reos0 Rsindcos6 ar OR OR 28 oR Rsind | OO ang (2 Rein@sind , 2 Reos® RsinOsin sensndeosb) aR OR 8 RF Rsin@ "OG RP sin*O aT Reosd | aT —Rsin® Dae a R 7a oR BT cosOcose , aT - Se th 21 (Zsaoans 2 ar aT cos@sing . aT cos: +9 (Fanesing + = rT aes) ar ar -sind +2 (Heese FP?) = (asinbcose-+9singsing-+2e050) + (Reos8eos + FeosOsing—2sine) 2 oF ; 1 + (sing + 90080) gg ar gla7, 1 ar ‘aR OR D6 Reins 06° which is Eq. (3.83). — 2, determine its directional jz) and then evaluate it at Problem 335 For the scalar function V derivative along the direction of vector P(1,-1,2). Solution: The directional derivative is given by Eq. (3.75) as dV /di = VV -8y, where the unit vector in the direction of A is given by Eq. 3.2): 3-52 +e nie CHAPTER 3 and the gradient of V in Cartesian coordinates is given by Eq. (3.72): WY = yt $x~ 82s. ‘Therefore, by Eq. (3:75), AtP(L,-1,2), (Mls 2 AN\a-12 V5" ee Problem 3.36 For the scalar function T = e~"/Scos@, determine its directional derivative along the radial direction and then evaluate it at P(2,x/4,3). Solution: T a eon _gerrsing 7 aa a lena) Problem 3.37 For the scalar function U = }sin®®, determine its directional erivative along the range direction R and then evaluate it at P(4,/4,n/2) Solution: =i site, a a| (s/4.0/2) CHAPTER 3 i ———— eee Problem 3.38 Vector field E is characterized by the following properties: (a) E points along R, (b) the magnitude of E is a fonction of only the distance from the origin, (c) E vanishes at the origin, and (4) V-E = 6, everywhere. Find an expression for E that satisfies these properties. Solution: According to properties (a) and (b), E must have the form E=REp ‘where Ep is a function of R only. ‘Hence, and Problem 3.39 For the vector field E = sxz— yz? — any, verify the divergence theorem by computing: (a) the total outward fiux flowing through the surface of a cube centered at the origin and with sides equal to 2 units each and parallel to the Cartesian axes, and (®) the integral of V-E over ihe cube’s volume. Solution: (2) For acube, the closed surface integral has 6 sides: PE: ds = Fag + Fonom + Fran + Fe + Fron+ Pots 12 ° - (2-2 -20)|,_, (Baya) LL wee (BLL) =» l es LL, B2-DE 20) a (Bees) oot (2) Lae _, (22-2 —25)|,_, (Gaede) [_cuue-((2)[_)f =z. _, Behe 29) |, (“Pdedz) ‘Pawn (LY st wor fy B2- HF -29)|,-1 Razey) -f, Lose ( @L) [L- Lo oe ~SyP — By) |__-(-taedy) 1 Lone (OL dh Jani +04 f+ Groro=S 232, verify the divergence theorem =0,and2=4 for the cylindrical region enclosed by 7 = 2, Solution: fea f, fe ((F10e~" 232) (—2rdrao))|__, coe + [CEE (eve ~#30)-(eraoae) |, 2 + JE (C00 —130) rar) ag te 2 pe =04 f 10e*2dodz+ ~l2rdrde fon [lstorndvaes [i [ards = 160ne~? — 48x ~ -82.77, fires [fF (BEE=2-s) canara = a f° 0er" =r)=3r)dr 2 =8n (-toerrs 10e"(1 +7) =) a0 = 160ne"? — 48x = -82.77. Problem 341 A vector field D = #7? exists in the region between two concentric cylindrical surfaces defined by r= I and r= 2, with both cylinders extending between 2 = 0 and z = 5. Verify the divergence theorem by evaluating: a) fD-as, is 14 CHAPTER 3 ) LY Dav. Solution: @) [fp ds = Fases-+ Foxes + Foonom + Foops Fam = ff, (@)-(-trdedo))|,_, = EE raed) ay = 10% Foues = EL (@).(razdo))|_, “EL (Fazd4)|,_» = 160%, Bosom = 7, fo, (EP )-(-trdode)) ag =0, 2 pan Foo= [, [7 (GP): Grdbar))|ag=0- ‘Therefore, [[D-ds = 150m. (b) From the back cover, V-D = (1/r)(@/2r)(r73) = 47°. Therefore, fff vrees [IE [i strardode= (RIE) Problem 3.42 For the vector field D = R3R°, evaluate both sides of the divergence theorem for the region enclosed between the spherical shells defined by R= 1 and R=2. Solution: The divergence theorem is given by Eq. (3.98). Evaluating the left hand side: 5 = 1508. lea [ypar= ane (eRrory) RsingaRded = 2x(—cos)[fg (3R4)|?_, = 1800. CHAPTER3 4s ‘The right hand side evaluates to fP-as= (EL @® (-Ra*sined0d)) + CE L888 sineaoa) leo = nan [° 3sinodo+2n [" 48sina0= 180%. foxo fono Inet Problem 343 For the vector field E = Say $(32 +22), caleulate (a) Bal around the triangular contour shown in Fig. P3.43(2), and @) [0 8)- ds over tne area ofthe triangle, Solution: In addition to the independent condition that z = ‘wiangle are represented by the equations y = 0.x = I, and y the three lines of the respectively. ; . : La @) @) Figure P3.43: Contours for (a) Problem 3.43 and (b) Problem 3.44. @) fe M=h+th+h, Lia [ Gay- 2 +27) (Rae gay 42de) = [hanced f° 0242) aot f°, Olaade=0, 16 CHAPTER 3 tga [ ye 42*)) Rade Fay-+2de) 1 1 = [oodles [LP +2 eo* [0 lara =0- (+%)|_+0= eI 3 b= | (Gy-5(2 +27) (Rast 44242) 7 0 = [rls 08 [P+ eay 0 [Ls Olyme de = GL wrth (b) From Eg, (3.105), VxE = -23z, so that f[ovees= LL [lB Care0 ce LLosavee= [pte ae=— he -3 Problem 3.44 Repeat Problem 3.43 for the contour shown in Fig, P3.43(b). Solution: In addition 20 the independent condition that z = 0, the three lines of the triangle are represented by the equations y = 0,y = 2—x, andy = x, respectively. @) pea Hhithth, b= [ @y—9 +29") (Bart By tae) = formant |? 2427) nett f°, Ohande=0, hae f (e962 +2%))- (det Fay+2de) . =f ecogerst [4242 Veer senat+ [,(Oherede a: -(? -) L- (47-2249?) [Ly to= CHAPTER 3 uy Loa [ (0-908 + 297)) (Rader gaye 2) =f henodt [C+D Magee [1 lad aye “Ol -Phed ‘Therefore, 2 n feaq0-D45=-3 (b) From Eq. (3.105), ¥xE = —23x, so that J[fvvease [Lo (88) eer Nae +L Lo C9 Cavaaleco ee 3 2 =~ [13s oyas- fi 3x(2-2)-0) ax =-@o- GF lh 3, Problem 3.45 Verify Stokes’s theorem for the vector field B = (freosg+§siné) by evaluating: @ fs dl over the semicireular contour shown in Fig. P3.46(@), and ® [i 2) as over she surface ofthe semicircle Solution: @ fea-[ a+ sas fs a, B-dl = (frcos6-+6sind)-(Fdr+$rdo + dz) = reosodr+rsinodp, oes (ome) ee = (PB pt0=2 us CHAPTER 3 2, Ok @ Figure P3.46: Contour paths for (a) Problem 3.45 and (b) Problem 3.46. [24> (freee) = 0+ (-2c089)[-0 [>a = (free) (4? )Irea*® + (fLorsineae)| Ir=2, 220 ana (ree), fBdareat2= 8. ©) VxB = Vx(éreoso+6sind) [fv a [Le et )) eraras) = [Lsnert rae = ((-eoseg?+n)ie Problem 346 ‘Solution: = a +6 (Zeevose)— 2 ” edna) =tsing (1+ 2). Repeat Problem 3.45 for the contour shown in Fig. P3.46(b). @) f3 a= [eas [sar fs as f Ba, B-dl = (ércos+$sing)- (Fdr+¢rdo+2dz) = reosodr+rsinodd, Le a= (f, reoseer) Io wt (Lursmoae)| =(GA)2, 405 juan (foors)| (Cal) wat (Caro) = = 04 (-2e080)/84, =2, [pa= (LL reosear) = [a= (fLreosear) at Chr), a =0+(-c080)ia = 1, 3.240-122 fB-aq3+24+0-1=3 ® VxB = Vx(frcos$+§sing) =f (250- Ztsine)) +6 (Zevcese) ~ 3) +25 (Zeasing)- Eircose)) = 2040444 (sing + (rsind)) = Ssind ( + +) + [[veea= ce ff, (ssne(144)) -eraraay = [Of sneies tydrae = (Ceosetb +n) ha) =F: 120 CHAPTER 3 Problem 3.47 Verify Stokes’s Theorem for the vector field A = Reos6 +$sin® by evaluating it on the hemisphere of unit radius. Solution: . . A= Reos8+Gsine = Ran +6A0+ Gy. Hence, Ag = c0s®, Ap = 0, Ap = sin. a) (Fung, é VxA=Rgaig (Siaesine)) ~6 For the hemispherical surface, ds = RR’sin@d8 do. an pat f (Vx A)-ds fo-oJoa0 “LCE ire snocode| Inet ‘The contour C is the circle in the 2-y plane bounding the hemispherical surface. fa a= | (eexe0+ js) ba, sino ["48| 2m. een)? > Bt i Problem 348 Determine if each of the following veetor fields is solenoidal, conservative, or both: (@) B= 22 — Hy +222, sing) /7 + §(cose)/77, /R. G-Th) te @ F=>H/(F+y), P+2)+ FOP Ee) ter +2), (CHAPTER 3 121 Solution: @) - i a a VA = V(Rxy— fy") = dy Fy = B20, VxA = Ux (i2y- $9") a (a 2 =1($o- Zi) +9 (Zem-Zo) +2(ZO%- Zen) = $04 $0-2(2x) ‘The field A is solenoidal but not conservative, ) VB = V(r — ape ge BRE Oe VxB = Vx (8x? — fy? + 222) =1(Z09-Z09 “aye (2e- Be) +4(209-2) = 20+90+20. ‘The field B is conservative but not solenoidal, ©) sing , gc08@) ¥uC= vx (852 a) no(3e-2 (3) (87-8) ECE) 3) woes (($)-(S2)) a" ‘The field Cis neither solenoidal nor conservative. 122, (CHAPTER 3 @ B\)_ 19 ffl 13 oe woe @) = jean (® (3) *mamoae 8) Ramws0 = Bg =vx(2 VxD xR at Fea (Rios) 2o) +6 as (@)- Zen) +64 (Zemoy- & (j)) =s0+60+40. ‘The field D is conservative but not solenoidal, © 1 ar Pe tb- Stata 1 jose yaweer) = 2 |e tg 7 GaP vunae(12 22) 24 2+4ar43 aa (E-B) (Bem ‘Hienee, Eis conservative, but not solenoidal. © CHAPTER 3 123 vorowe-nsso-ose2 (ss)-2 (2) 2y w(e aoe seas) =1(233) «0 ace, Fis both solencisal and conservative. ® Sante) tae ts) ey ve o= ers 2yeZorerys 2 OF +2) =2e+2yt 2240, vxGna(Zots2)-Zo*+x)) +9(2e+2)-Zo%+2)) a a +2(Zor+7)-Z 242) SR2yt 9222080. Hence, G is neither solenoidal nor conservative. (hy H=R(Re*), di Ores cay. Liga ss i Via Me") = BORE ReM) = eR 3—R) #0, VxB=0. ‘Hence, H is conservative, but not solenoidal. ‘Problem 3.49 Find the Laplacian of the following scalar functions: Solution: (2) From Eq. 3.110), V?(xy72)) = 2x23 + 62. 124 FR @VGytyt2) = Meare bik Svat of the book, aVe ja art= @+ “(ap) @ V%(Ser" cose) = Se"e059 ( -2 -3) : © 2 v2(10e"* sin) = 108 ( (-3) este are (106-*sine) sino (1-5) + ase sul CHAPTER 4 125 Chapter 4 Sections 4-2: Charge and Current Distribations Problem 4.1 A cube 2 m on a side is located in the first octant in a Cartesian coordinate system, with one of iis comers at the origin. Find the total charge contained in the cube if the charge density is given by py = xy7e7* (mC/m*). ‘Solution: For the cube shown in Fig. P4.1, application of Eq. (4.5) gives o=frav= of, [i ptePanayae -(ger)f ff =262mC. |z=oly=0le=0 Figure P4.1: Cube of Problem 4.1. ee Problem 42 Find the total charge contained in a cylindrical volume defined by r<2mand0Sz<3mifp, = 10re(mC/m*). Solution: For the cylinder shown in Fig. P4.2, application of Eq. (4.5) gives Q= LK fi porreraeee = (3-2) PPP 240m (mC) = 0.754 C. 0 ax r=0lg=ol ent 126 CHAPTER 4 3m aa Figure P4.2: Cylinder of Problem 4.2. Problem 4.3 Find the total charge contained in a cone defined by R <2 m and 08 < 7/4, given that p, = 20R?cos?8 (mC/m?), Solution: For the cone of Fig. P4.3, application of Eq. (4.5) gives 8p ° £ _[f20R cos? oR? sinoaeaode JexoJoxo Jao 2 pale solely > = = (: - (4) ) =1733 (m0) =0.173C. -RSQcos* ) CHAPTER 4 127 ‘Figure P4.3: Cone of Problem 4.3. Problem 4.4 Ifthe line charge density is given by py = 12y? (mCfm), find the total charge distributed on the y-axis from y = —5to y= 5. = 1000mC=1C. Problem 4.5 Find the total charge on a circular disk defined by r< a and z = if: (2) ps = prosing (Cim*), () 95 = pxosia6 (Chm?), (©) 6 = pave’ (Cim?), (@) 9s = page" sin® 6 (Chm), ‘where po is a constant, Solution: @ a a= = fras=[" [easing rérd9= ~po 5 | cose = ®) _ pt pF _ Bl pP* (1 =c0s20 o= fi, [ospssste rardo= p05 ( + ae Pp naz = Sow. = (=) lo 128 CHAPTER 4 © an a= [0 [poe rarae = aapes [rear = 2npa[-re* 715 = 2npyo[l-e™“(1+a)]. @ o= [i [eae rsnte rdrdo a = pe [rear [osetoee = poll —e*(1+a)] 2 = mpgil —e-%(1+a)]. Problem 4.6 If J = $2x¢ (A/m?), find the current / flowing through a square with comers at (0,0,0), (2,0,0), (2,0,2), and (0,0,2). Solution: Using Eq, (4.12), the net current Slowing through the square shown in Fig, PA6is 1 [oa fom] easton (2) f =8A. “a 0 v= Le=Ole=0 Figure P4.6: Square surface. CHAPTER 4 129 Problem 4.7 If J = R25/R (Alm), find J through the surface R= 5 m. Solution: Using Eq, (4.12), we have [oe [" [(82) + (RR? sined0de) an 500m 25R0cose) 1,570.8 (A). L_ ls=olo=0 Problem 48 Anelectron beam shaped like a circular cylinder of radius 79 carries a charge density given by r= fee (Cim’), 7 ‘where po is a positive constant and the beam’s axis is coincident with the z-axis. (2) Determine the total charge contained in length L of the beam. (®) Ifthe electrons are moving in the ++<-direction with uniform speed u, determine the magnitude and direction of the current crossing the z-planc. Solution: @ ae es —npoLin(1 +73). &) (Current direction is along —2. ice 130 CHAPTER 4 Section 4-3: Coulomb’s Law Problem 4.9 A square with sides 2 m each has a charge of 20 uC at each of its four comers. Determine the electric field at a point 5 m above the cemter of the square. Rs Qsl-1-1,0) 24-1.1,0) Qe{1.-1.0) 2401.10) Figure P4.9: Square with charges at the comers. Solution: The distance [R| between any of the charges and point P is CHAPTER 4 131 iR| = VPS $S = V7. BR ire’ RF RE 2 [8985 B95 neq | (2757 GIpe * (ase Gye o. apm oe SE 3 10° (vem = 425.61 (vim). Problem 4.10 Three point charges, each with q= 3 nC, are located at the comers of a triangle in the x-y plane, with one comer at the origin, another at (2 em, 0,0), and the third at (0,2 em,0). Find the foree acting on the charge located at the origin. Solution: Use Eq. (4.19) to determine the electric field at the origin due to the other two point charges [Fig. P4.10}: 1 [ss 000) , 20 70.02) = gel (0.02 (0.02) -67.4(8+§) (Vim) at R= 0. Employ Eq. (4.14) to find the force F = gB = -202.2(8+9) UN). Figure Pé.10: Locations of charges in Problem 4.10. i Problem 4.11 Charge 41 = 4 uC is located at (1.em,1em,0) and charge a2 is located at (0,0,4¢m). What should q2 be so that E at (0,2 cm,0) has no component? Solution: For the configuration of Fig, P4.11, use of Eq, (4.19) gives 132 CHAPTER ¢ Ri=8+92D=Ck+ Hem Ro= 2-44) em Figure P4.11: Locations of charges in Problem 4.11. 1 [4eC(-84+9) x 107 | gol 92. 2 Hae =a [iene + Soe | = zglaa4 x 1076 + 9(14.14 x 10° 40.2249) —20.447¢:) (Vim). IEE, = 0, then gp = —14.14 x 10-*/0,224 = —63.13 (uC). Problem 4.12 A line of charge with uniform density py = 4 (uC/m) exists in air along the z-axis between z = O and z = Som. Find E at (0,10. cm.0). Solution: Use of Eq. (4.21¢) for the line of charge shown in Fig. P4.12 gives “7 ax wg =%) oaeeee _4x10s[_gtoe+2 JE 0 5.93 x 103(9 4.47 21.06] = 7160.7 x 108 238.1x10° (Vim). 133 Figure P4.t2: Line charge. Problem 4.13 Electric charge is distributed along aa arc located in the x-y plane and defined by r= 2cm and 0<@ _ , 3246? 36005 7° “Ancol,” Ane R, ~ * Seyi? ~* dreg(d— a? F; = Bi301Os , Ress ___, 9s 4236005 4mepR?; " 4nepR3, ‘Sntox *” Gaegt ‘Hence, equilibrium requires that 32He 90s _324e 3601 _ 905 eee Section 4-4: Gauss’s Law Problem 4.17 Three infinite lines of charge, all parallel to the z-axis, are located at the three comers of the kite-shaped arrangement shown in Fig. 4-29 (P4.17). If the i 138 CHAPTER 4 two right triangles are symmetrical and of equal corresponding sides. show that the electric field is zero at the origin. 201 Pr Pr Figure P4.17: Kite-shaped arrangment of line charges for Problem 4.17, Solution: “The field due to an infinite line of charge is given by Eq. (4.33). In the ‘present case, the total E at the origin is E=E,+Eo+Es. ‘The components of Ey and E> along % cancel and their components along —7 add. ‘Aso, Es is along because the line charge on the y-axis is negative. Hence, _2picos8 |. 201 - +9. 9 paso 7 ZreoRs But cos@ = Ri /Rz. Hence, Pr Ri, Pr B= 9 reoR Ri”) Teak Problem 4.18 Three infinite lines of charge, px, = 5 (aC/m), Pie 1, = 5 (aC/m) are all parallel tothe z-axis. If they pass through the respective points 139 Figure P4.18: Three parallel line charges. (0,—b), (0,0), and (0,6) in the x-y plane, find the electric field at (a,0,0). Evaluate your result for a = 2m and b= 1 cm, Solution: P,=5 (aClm), Pe=—5 (acim), Ps = Ps E=E)+E2+Es. Components of line charges 1 and 3 along y cancel and components along x add. Hence, using Ba. (4.33), 20m 2me Ry cosO +i. 140 CHAPTER 4 Fora =2cmandb= lem, E=8270 (kV/m). ce Problem 4.19 A horizontal strip lying in the x-y plane is of width d in the edirection and infinitely long in the x-direction. If the strip is in air and has a ‘nniform charge distribution p,, use Coulomb's law to obtain an explicit expression for the electric field at a point P located at a distance h above the centerline of the strip. Extend your result to the special case where d is infinite ané compare it with Eq, (4.25). Figure P4.19: Horizontal strip of charge. Solution: The strip of charge density 9, (C/m?) can be treated as.a set of adjacent line charges each of charge p: = 9, dy and width dy. At point P, the fields of line charge at distance y and line charge at distance —y give contributions that cancel each other along and add along 2. For each such pai, 2padycos® FB peek CHAPTER 4 141 1/cos®, we integrate from y = 0 to d/2, which corresponds to @ = 0 to Bo = sin“ "[(d/2)/(h2 + (4/2)?)"7], Thus, a Be pees prea apa PS yng aa J melo RS zh Kasra? 3 =e, Re 2s stich is identical wit Pe, which is identical with 2&0 * For an infinitely wide sheet, 8) = x/2 and E = 2 (4.25). Problem 4.20 Given the electric flux density D=W[x+y)+93x-29) (Cm), determine (@) py by applying Eq. (4.26), (b) the total charge Q enclosed in a cube 2 m on a side, located in the first octant with three of its sides coincident with the x-, y-, and z-axes and one of its comers at the origin, and (© the total change @ in the cube, obtained by applying Eq, (4.29). Solution: (a) By applying Eq. (4.26) a a Pea V-D = F224 29)+ 5152-29) 30, (Integrate the charge density over the volume as in Eq, (427): 2p v-pav= =0. o=' fv -pav LL Lower (©) Apply Gauss’ law to caleulate the total charge from Ea. (4.29) fds = Fron+ Feu + Fate + Ft + Fp + Feaon; 22 Fon= [, [pcotrneser-an|_ ease = f i) oe (=@+) ) 2 ly=o 142 CHAPTER 4 an £2 7 2 | kdzd) Raa = [_, [C200 +98— DD (Bae) 2 \ 2 =f £ Arty) aa--(v1 ) pie leo Jno) bao ; Fane [fe (faeryevex-29] (gaeas) .) bee “(-$aedz) 2 Lo Lere|_ee- (G24) rae fi, [lsat a ee (@OL)L Fop £ 7 (8249) +832— >») -(2ayax) : “ff . 2A E Fnoa= [ fsespsser20| (dyas) = Pala Thos Q= fD-ds=24-8-4—12+040=0 i Problem 4.21 Repeat Problem 4.20 for D = 72° (Cim?). Solution: (a) From Eq, (4.26), py = V- p= Zire yay. (©) Total charge Q is given by Eq. (4.27): a= | v-Dav= LLL Peace ® 0 dydx=0, Iso? dydz=0. 64 LL CHAPTER 4 143 (©) Using Gauss’ law we have fo f Note that D = £D,, so only Fin: 24 Foscx (integration over’ surfaces) will contribute to the integral. 2 \p ) 2 = Fhiont+ Fact “+ Faghs + Fes Fop + Poot: -(Rdydz) Faas [2 [er2] Lhe oe- OGLE LL? tee Face Thus Q= f as Sroro+0+04 SS Problem 4.22 Charge Q; is uniformly distributed over a thin spherical shell of radivs a, and charge Q» is uniformly distributed over 2 second spherical shell of radius b, with b> a. Apply Gauss’s law to find E in the regions Rb. Solution: Using symmetry considerations, we know D = RDg. From Table 3.1, ds = RR sin@ d0d6 for an element of a spherical surface. Using Gauss’s law in integral form (Eq. (4.29)), fP-ds= Duss ‘where Qjor is the total charge enclosed in S. For a spherical surface of radius R, eu 5 2, ke [E(B sind 6d8) = One, Dak*(2n)[—cos8]§ = Dror From Eq, (4.15), we know a linear, isotropic material has the constitutive relationship D=cE, Thus, we find E from D. 144 CHAPTER 4 (2) In the region R< a, ro = 9, (hm). (b) In the region a < R< b, Qec= 215 (c) In the region R > b, an, — MOi+O) Qe=O1+02, B= Rie =P EEE vim). Problem 4.23 The electric flux density inside a dielectric sphere of radius a centered at the origin is given by D=BpoR (Cia), where po is a constant. Find the total charge inside the sphere. Solution: a pm cm oporas= [EC Boor nat sino cede = 2xpoa? J" sindd0 = ~2npoa®cos8|§ = dmpoa? ©- Problem 426 Ina certain region of space, the charge density is given in cylindrical coordinates by the functi Pv =20re" (Chm). Apply Gauss’s law to find D. Solution: CHAPTER 4 145 ees pe Figure P4.24: Gaussian surface. ‘Method 1: Integral Form of Gauss’s Law ‘Since py varies as a function of r only, so will D. Hence, we construct a cylinder of radius r and length L, coincident with the z-axis. Symmetry suggests that D has the functional form D = #D. Hence, fP-de=0, [eo ds = D(2nrL), = ant ["20re~"-rar = 40nt[—e~’ +2(1—e7"(1+7))], D #20 2a meen) —re"]. Method 2: Differential Method V-D=py, D=FD,, with D, being a function of r. 1a - FRO = re", | | 146, CHAPTER 4 a a renee . [ gemiar= [r0rea, 7D, = 2[2(1-e"(1+7))— Pew], D iro, =10 2a -eraany-] Problem 4.25 An infinitely long cylindrical shell extending between r = 1 m and = 3 m contains a uniform charge density py. Apply Gauss’s law to find D in all regions. Solution: Forr <1 m,D=0. Fori3m, D,-2nrL. = pyonL(3* ~ 1?) = SpyonL, D=#D, CHAPTER 4 147 Figure P4.25: Cylindrical shell. Problem 4.26 If the charge density increases linearly with distance from the origin such that py = 0 at the origin and py = 10 Chm? at R= 2 m, find the corresponding variation of D. Solution: pe(R) = a+5R, Pr(0) 0, 148 CHAPTER 4 pe(2) = 2b = 10. Hence, b= 5. p(R)=5R (Cin). “Applying Gauss's law to a spherical surface of radius R, foa= Jovan, 4 Dante = [ sp-anfPar = 200%, lo Da= ik? Cin*), Be Cm). i Section 4-5: Electric Potential Problem 427 A square in the x-y plane in free space has a poiat charge of +@ at ‘comer (a/2,a/2) and the same at comet (a/2,~a/2) and a point charge of ~O at each of the other two comers. (a) Find the electric potential at any point P along the x-axis. (b) Evaluate V atx= a/2 Solution: Ry = Rz and R 2g CHAPTER 4 149 Figure P4.27: Potential due to four point charges. kOe) Problem 428 The circular disk of radius a shown in Fig. 4-7 (P4.28) has uniform charge density p; across its surface. (@) Obtain an expression for the electric potential V at 2 point P(0,0,2) on the zaxis. (®) Use your result to find E and thea evaluate it for <=. Compare your final expression with Eq. (4.24), which was obtained on the basis of Coulomb's law. Solution: (@) Consider a ring of charge at a radial distance r. The charge contained in width dris q= p,(2nrdr) = 2np,rdr. ‘The potential at P is _ 4g 2nperdr = GregR ~ Saag eye” ‘The potential duc to the entire disk is _ rdr Pe v f ave ge alma [@+27e-4 fe 2&0 150 CHAPTER 4 Figure P4.28: Circular disk of charge. ) ov _ 95-8 SO [ees ‘The expression for E reduces to a (4.24) when z= A. Problem 429 A circular ring of charge of radius a lies in the x-y plane and is, centered at the origin. If the ring is in air and carries a uniform density p,, (a) show ‘that the electrical potential at (0,0,2) is given by V = py2/[2eq(a* +z°)"/?], and () find the corresponding electric field E. Solution: (2) For the ring of charge shown in Fig. P4.29, using Eq. (3.67) in Eq, (4.48¢) gives vry= ‘ame [Ra whos S07 +e a Point (0,0,z) in Cartesian coordinates corresponds to (7,4,2) = (0,,2) in cylindrical coordinates. Hence, for r = 0, a pia ¥(0,0,z)= -— ade = Pe (0.02) sa Joao Jarre Delete 151 Figure P4.29: Ring of charge. (b) From Eq, (4.51), soit ao jp a 1/2 E=-W (?+2) tn ae 2a 92 (hn). Problem 4.30 Show that the electric potential difference V;2 between two points in air at radial distances r; and rp from an infinite line of charge with density py along the z-axis is Vig = (,/2n9) In(r2/r1). Solution: From Eq. (4.33), the electric field due to an infinite line of charge is ‘Hence, the potential difference is Yoo fea Problem 431 Find the electric potential V at 2 location a distance b from the origin in the x-y plane due to a line charge with charge density py and of length !. The line charge is coincident with the z-axis and extends from z= ~1/2t0z = 1/2. 152 CHAPTER 4 Figure P4.31: Line of charge of length ¢. Solution: From Eq. (4.48c), we can find the voltage at a distance b away from a line of charge (Fig. P4.31]: veoye [Pare OL dz _ oy, ( 14 Va are ln RS are Jin Ve4¢R Se” \ Veep) ee Problem 432 For the electric dipole shown in Fig. 4-13, (mV/m) at R= 1 m and 6 = 0°. Find E at R = 2m and @ = 90°. Solution: For & = 1 mand @ = 0°, |E| = 2 mV/m, we ean solve for gusing Ea. (4.56): E zit. (R2cos8-+8sing). Freak Hence, 1El= ie) 2=2mVin a0=0°, ARE, 10-3 x4méq _ 107 x 4neq ge SS gO cote ©. Again using Eq. (4.56) to find E at R = 2 m and @ = 90°, we have = DARE X1O 5 gal B= a (0) +8) 63 (avin), CHAPTER 4 153 Problem 4.33 For each of the following distributions of the electric potential V, sketch the comesponding distribution of E (in all cases, the vertical axis is in volts, and the horizontal axis is in meters) Solution: B16 VV 420 a 134 CHAPTER 4 26 A SRR TRE TR REE ETE TEI & © Figure P4.33: Electric potential distributions of Problem 4.33. —————— Problem 4.34 Given the electric field snd B= 22} forz> 0, and R = : ‘Along z-direction, 8 = z<0. Hence, Vap = m2. aes [25 a [sb ae CHAPTER 4 155 Figure P4.34: Potential between B and A. Problem 4.35 An infinitely long line of charge with uniform density p; = 6 (aC/m) lies in the «-y plane parallel to the y-axis at x = 2 m. Find the potential Vag at point A(3.m,0,4m) in Cartesian coordinates with respect to point B(0,0,0) by applying the result of Problem 4.30. Solution: According to Problem 4.30, oy v= Pf n(2 2m G where rj and rp are the distances of A and B. In this case, \VG-27+4=viTm, m=2m. 6 x10-9 “Ao = sexes KIO 156 CHAPTER 4 Figure P4.35: Line of charge parallel to y-axis. Problem 4.36 The 2-y plane contains a uniform sheet of charge with p,, = 0.2 (nC/m?) and a second sheet with p,, = —0.2 (nC/m*) occupies the plane z = 6 m. Find Vas, Vac, and Vac for A(0,0,6 m), B(0,0,0), and C(0,—-2m,2m). Solution: We start by finding the E field in the region betweea the plates. For any point above the 2~y plane, E; due to the charge on x-y plane is, from Eq. (4.25), In the region below the top plate, E would point downwards for positive ps, om the top plate. In this case, Ps, = —Ps,. Hence, CHAPTER 4 157 Figure P4.36: Two parallel planes of charge. ‘The voltage at C depends only on the z-coortinate of C. Hence, with point A being at the lowest potential and B at the highest potential, (-135.59) 3 135.59+45.20. = 45.209, 90.39. Section 4-7: Conductors ‘Problem 437 A cylindrical bar of siliconhas a radius of 2mm anda lengthof Sem. If a voltage of 5 V is applied between the ends of the bar and jie = 0.13 (m?/V's), ity = 0.05 (m2/V-s5), Ne = 1.5 x 10" electrons/m’, and My = Ne, find (@) the conductivity of silicon, (b) the current / flowing in the bar, (©) the drift velocities v, and Un, (@) the resistance of the bar, and (@) the power dissipated in the bar. | d | tocassiailaica i 158 CHAPTER 4 Solution: (2) Conductivity is given in Eg. (4.65), O= (Nets + Nan le = (1.5 x 10"9}(0.13 +.0.05)(1.6x 107!) = 4.32 x 10-* (Sim). (©) Similarly to Example 4.8, parts b and c, T= JA= GEA = (432% 10-4) (&) (a(2x 1073)9) = 542.9 (na). (©) From Eqs. (4.62a) and (4.621), E E =-(0:13)(100) 2 = 13 (vs, Bes “HE = ~(0.18)(100) oa (nis), E B= ME= H0.05)(100)F = 5 a. (@) To find the resistance, we use what we calculated above, Problem 438 Repeat Problem 4.37 for a bar of germanium with y, = 0.4 (m?/V-s), My = 0.2 (mm? /V-s), and Ne = Ny = 2.4 x 10! electrons or holes/m?. Solution: (a) Conductivity is given in Bq. (4.65), = (Netle + Nate = (2.4% 1019)(0.4+0.2)(1.6 x 1079) (©) Similarly to Example 48, pars b and, 3 (Sim). 12th = 084 =(22)(, 392) = FB) exe ?)=2.89 (mA). (©) From Eqs. (4.62a) and (4.625), a= 4B = -(0.4)( 10) = = IE, @=AB= (0.2,100) 5 =20. CHAPTER 4 159) (@) To find the resistance, we use what we calculated above, ov 7 289mA (@) Power dissipated in the bar is P= IV = (5V)(2.89 mA) = 14.5 (mW). —_—_——— SS Problem 439 A 100-m-long conductor of uniform cross section has a voltage drop of 2 V between its ends. If the density of the current flowing through it is 7 x 10° (A/m?), identify the material of the conductor. Solution: We know that conductivity characterizes a material: 2) 100 (mn), R 1.73 GQ). J=0E, Tt cant = 0 ( ), =3.5x10" (Sim). From Table B-2, we find that aluminum has 6 = 3.5 x 10” (S/m). a Problem 4.40 A coaxial resistor of length 7 consisis of two concentric cylinders. ‘The inner cylinder has radius a and is made of a material with conductivity 1, and the ovter cylinder, extending between r = a and r= b, is made of a material with conductivity 02. If the two ends of the resistor are capped with conducting plates, show that the resistance between the two ends is R = I/[re(oya* + 62(47 —a”))). Solution: Due to the conducting plates, the ends of the coaxial resistor are each uniform at the same potential. Hence, the electric field everywhere in the resistor ‘will be parallel to the axis of the resistor, in which case the two cylinders can be considered to be two separate resistors in parallel, Then, from Eq. (4.70), guna? __ oon(t—a*) T t 3 r Wattaeaay i Problem 4.61 Apply the result of Problem 4.40 to find the resistance of a 10-cm- long hollow eylinder Fig. P4.41) made of carbon with 6 = 3x 10* (S/m). Solution: From Problem 4.40, we know that for two concentric cylinders, L © n(Gia + o2(8? —@)) @. 160 CHAPTER 4 o> ‘Carbon Figure P4.41: Cross section of hollow cylinder of Problem 4.41. Por air 6; = 0 (Sm), G2 = 3x 10* (S/m); hence, 1 R= aaosyeomp 2) a Problem 4.42 A2X 10~3-mm-thick square sheet of aluminum has 10 em x 10cm ‘faces. Find: (2) the resistance between opposite edges on a square face, and (b) the resistance between the two square faces. (See Appendix for the electrical constants of materials). Solution: @ 1 Raz: For alumminam, 6 = 3.5 x 10" (S/m) [Appendix B). =10cm, A= 1emx2X 107? mm =20x 107 x 10-6 = 2x 107m, 10x 107? R= gayi xanto 7 (@) Now, = 2 10-2 mm and A = 10cm x 10cm = 10"? mi. _ 2x10 = 33x 10x10 a R 5.71 pQ. CHAPTER 4 161 Section 4-9: Boundary Conditions Problem 4.43 With reference to Fig. 4-19, find Ey if By 1 = 29, & = 180, and the boundary has a surface charge density Ps = 7.08 x 107"! (C/m*), What angle does Ez make with the z-axis? Solution: We know that Ey, = Ex: for any 2 media. Hence, Ey, = Also, (Dj ~ D2}-'= p, (from Table 43). Hence, €; (By i) — ex(E: gives 83 92+26 (Vim), peterkre 7.08 10- 18(4) __7.08 x 10°! es Beg 2 2x8.85xK 10 Hence, Ey = 3 ~ 2+ 240 (V/mm). Finding the angle By makes with che z-axis: Eu +36=40 (Vim). Ey-2=[Ez|cos8, 4= 9444 16c0se, a=cos"! (42) ar. VB, Problem 4.44 An infinitely long dieiectric cylinder with e1, = 4 and described by 1S 10 cmis surrounded by 2 material with €2, = 8. IFE; = #°sino—832coso+23 (tm) in the cylinder region, find E and Dz in the surrounding region. Assume that no free charges exist along the cylinder’s boundary. Solution: Using Table 4-3, £1 = En, and t- (Di ~ Da] =P Ey =Ey=—437cosh+23 (Vm), 21Ein =E2E2n (Since p= 0), Thus, = B57 sine 4377 cos+23, Da = exEn = Be (Hhsine —$37-2s¢+5) (Cm). Problem 4.45 A 2-cm dielectric sphere with e1, =3 is embedded in a medium with Exe = 9. If Ep = 83cos @ — 63 sin (V/mn) in the surrounding region, find Ey and Di in the sphere. Solution: Using Table 43, £1; = 2, and -(D; —D2) = ps, Ps=0, Dr, =Dr,, te CHAPTER 4 Ey = Ex = -3sind. So Ey =R9ces0-€3sin9 (Vim), Dy =e1E; = 3e9(R9cos@ — 63sin6) = e9(R27cos0—69sin) (Cim?). Problem 4.46 If E = 850 (V/m) at the surface of a S-cm conducting sphere centered at the origin, what is the total charge Q oa the sphere’s surface? Solution: From Table 4-3, fi- (D, — D2) = 9s. Ez inside the sphere is zero, since we assume it is a perfec: conductor. Hence, fora sphere with surface area S = érca*, o_o Seq" Q = ExSca = (50)sn(0.05)*e9 = 2), Die=Ps Eia= Problem 4.47 Figure 4-34(a) (P4.47) shows three planar dielectric slabs of equal thickness but with different dielectric constants. If Eo in air makes an angle of 45° with respect to the z-axis, find the angle of E in each of the other layers. £9 (air) Figure P4.47: Dielectric slabs in Problem 4.47. CHAPTER 4 163 Solution: Labeling the upper air region as region 0 and using Eq. (4.99), 6) =tan? (Zane) = tan7!(3tan45°) = 71.6", = tan (Zene;) stn! (ja0716°) 278.7, er 3 05= tan” (Zune) =n? (Zan78.7°) =819°. & 3 In the lower air region, the angle is again 45°. Sections 4-10 and 4-11: Capacitance and Electrical Energy Problem 4.48 Determine the force of attraction in a parallel-plate capacitor with A= 10cm, d= 1m, and; = 4if the voltage across itis 50 V. Solution: From Eq. (4.131), 2 228(10 1074) (&) -24427%10° (N). Problem 449 Diclectric breakdown occurs in a material whenever the magnitude of the field E exceeds the dielectric strength anywhere in that material. In the coaxial capacitor of Example 4-12, (2) At what value of ris |E| maximum? (b) What is the breakdown voltage if a= 1 om, b= ‘material is mica with e, = 6? Solution: (a) From Eq, (4.114), E= —fpi/2ner fora 200 (MVim) (see Table 4-2), or cm, and the dielectric mp FEl= 5a = Ta(eeqy(ios = 200 AVI). which gives py = (200 MV/m)(2n)6(8.854 x 10°")(0.01) = 667.6 (Cm). ‘From Eq, (4.115), we cen find the voltage corresponding to that charge density, b (667.6pClm) 2) = aes oe ap MO= 129 OO”. 164, CHAPTER 4 1,39 (MV) is the breakdown voltage for this capacitor. nD Problem 450 An clectron with charge Qe = -1.6 x 107? C and mass me = 9-1 x 1072! kg is injected at a point adjacent to the negatively charged plate in the region between the plates of an air-filled paraliel-plate capacitor with separation of 1 em and rectangular plates each 10 cm? in area Fig. 4.33 (P4.50). If the voltage across the capacitor is 10 V, find (@) the force acting on the electron, (b) the acceleration of the electron, and (©) the time it takes the electron to reach the positively charged plate, assuming ‘that it starts from rest. =the Yo=10V ‘Figure Pé.50: Electron between charged plates of Problem 4.50. Solution: (a) The electric force acting on a charge Qe is given by Eq. (4.14) and the electric field in a capacitor is given by Eq. (4.112). Combining these two relations, we have v 10 F=0.E=0.% 10 _igxto-® om. OnE = QF = 16x 1G = 1S KIT OW. ‘The force is directed from the negatively charged plate towards the positively charged plate. ” 16 F_ 16x10" = Eo LORI = 176x100" (nis? OF a FIR 10 mia (oie (6) The electron does not get fast enough at the end of its short tip for relativity to manifest itself classical mechanics is adequate (o find the transittime. From classical do-+uot + dar? where in the present case the start position is dy CHAPTER 4 165 the total distance traveled is d = 1 em, the initial velocity xo = 0, and the acceleration is given by part (b). Solving for the time t, a [RKO -% FEV ZV Trexi9m = 10710 Problem 4.51 Ina dielectric medium with ¢, = 4, the electric field is given by =10.7 (as). E=X(7422)+9-Hy+2) (Vim). Calculate the electrostatic energy stored in the region 1 m in terms of €1, €2, V, and the indicated dimensions of the capacitor. (b) Calonlate the energy stored in each of the dielectric layers and then use the sum to obtain an expression for C. (© Show that C is given by Eq. (4.136). Solution: (@) If V) is the voltage across the top layer and V3 across the bottom layer, then venti, 170 CHAPTER 4 Figure P4.54: (a) Capacitor with parallel dielectric layers, and (b) equivalent circuit (Problem 4.54). Figure P4.S4: (c) Electric fields inside of capacitor. According to boundary conditions, the normal component of D is continuous across the boundary (in the absence of surface charge). This means that at the interface between the two dielectric layers, Di= Dae CHAPTER 4 m or 212) = &2E2- 212 V = Eid) + End2 = Bid + das which can be solved for Ey: z R=. atta & Similarly, © 2 Jyaf_eredad . ya [ees 6.30 [aaa 2 1 ejeeAd |. Ad; = = V? | 12 72 (eter 1ys cad'ee Wen Wa tie = 9 v [seat eso. But We = }CV?, hence, S1Adi + ejerAde _ 4 (Edi teid) _ __tr€2A (edi Fed ead peice? edi ted” (©) Multiplying numerator and denominator of the expression for C by A/dida, we have SA GA ce A _ OG ey 4A 4G” “ae ms aa, Gat. a @ 172 CHAPTER 4 Problem 4.55 Use the expressions given in Problem 4.54 to determine the capacitance for the configurations in Fig. 4.35(a) (P4.55) when the conducting plates are placed on the right and left faces of the structure, 3 emt Bet) ans Figure P4.55: Dielectric section for Problem 4.55. (@2x5)x10+ 1x10? a A ag, (245) x10 Conta = 403K 10 CG _ 17Tx LAB “G$Q” V77+118 =e % = 260 = 2069 x 107? = 1.77 x 107" F, A = 118x107? F, x10? = 0.71 x10 F Section 4-12: Image Method Problem 456 With reference to Fig. 437 (P4.56), charge Q is located at a distance d above a grounded half-plane located in the x-y plane and ata distance d from another grounded half-plane in the x-z plane. Use the image method to (@) establish the magnitudes, polarities, and locations of the images of charge Q with respect to each of the two ground planes (as if each is infinite in extent), and (b) then find the electric potential and electric field at an arbitrary point P(0,»;2). Solution: (@) The original charge has magnitude and polarity +@ at location (0,d,¢). Since the negative y-axis is shielded from the region of interest, there might as well be a ‘conducting half-plane extending in the —y direction as well as the +y direction. This 173 Qe d+ Figure P4.56: (a) Image charges. ground plane gives rise to an image charge of magnitude and polarity —Q at location (0,d,—d). In addition, since charges exist on the conducting half plane in the +z rection, an image of this conducting half plane also appears in the —z direction. ‘This ground plane in the x-z plane gives rise to the image charges of —Q at (0,—d,) and +Q at (0,—d.—d). (b) Using Eq. (4.47) with = 4, 1 | RIOT A+ v2) aA) ie+90— 2) +240) 174 CHAPTER 4 - &( 1 a ane \ Jeeo-dee(c—a? #+ 0+) +-a? ‘ 1 _ 1 ) \Vetorareetae [Pe(y-dy (cea) 2 1 Llaeeeee i 0 Vets bd +2 = 2ed OP 1 + Jee de ed L 7S) o> From Eq. (4.51), B=-wW 2a a ane Jesy—dPte-ae f+ 0+d)*+@-4? +¥. u -v. 3 ) Ve+ordP eer? Ve+o-aP (eae 2 ( Sx+H(y-4)+3(e—d) Bx+9 ta) +2{z-d) “Re \ By o-ae-ay Brora teary 4 SEHG+a) taba) Bxtfy—d) +844) ) nay Broraeeeray”™ Gro-a+erary” Problem 4.57 Conducting wires above a conducting plane carry currents J} and zy in the directions shown in Fig. 4-38 (P4.57). Keeping in mind that the direction of a current is defined in terms of the movement of positive charges, what are the Girections of the image curents corresponding to Jy and /2? Solution: (q) In the image curreat, movement of negative charges downward = movement of ‘positive charges upward. Hence, image of f is same as CHAPTER 4 115 @) Figure P4.57: Currents above a conducting plane (Problem 4.57) ; +qery 4A +4 @ 120 -q@ 00 h : mage) | 4 eer Figure P4.57: (a) Solution for part (2). (®) In the image current, movement of negative charges to right = movement of positive charges to left. —1, oo +9@ ist, +9 eo me dy Tees —=—— f, (image) ‘Figure P4.57: (b) Solution for part (b). . Problem 4.58 Use the image method to find the capacitance per unit length of an sili eitanactsetinn er 176 CHAPTER 4 infinitely long condveting cylinder of radius a situated at a distance d from # parallel conducting plane, as shown in Fig. 4-39 (P4.58). © Figure P4.58: Conducting cylinder above a conducting plane (Problem 4.58). Figure P4.58: (a) Cylinder and its image. Solution: Let us distribute charge p; (Cit) on the conducting cylinder. Its image cylinder at z = —d will have charge density ~p1. For the line at : =, the electric field at any point z (at a distance of ~z from the center of the cylinder) is, from Eq. (4.33), pu Bis 4 da2) CHAPTER 4 17 where —2 is the direction away from the cylinder. Similarly for the image cylinder at distance (d +z) and carrying charge —p1, (py) Preo(d +2) ‘The potential difference between the cylinders is obtained by integrating the total electric field from z= —(d—a) to z= (d—a): bane Ca R= i -[@ +E) -2é2 I, * Pr 1 ) é PL LL) 2d fe a (gata * awe fP* (24h * ep [a acta) « = Pe bias) +d tag = es [+ In(a) + 1n(2d—a) + In(2d~ a) —In(a)] oP gg (rE = n(42). For alengthL, Q'= piL-and pel (p1/xe0)talC ‘and the capacitance per unit length is, c Cate 178 Chapter 5 Sections 5-1: Forces and Torques Problem 5.1 An electron with a speed of 4 x 106 f m/s is projected a positive direction into a medium containing a uniform ee a Be (a2 83) 7. Given that e= 16x10"! C and che mast of an mez 9.1% 10-2! kg, determine the initial acceleration vector of the elecioy moment itis projected into the medium), lee Solution: The acceleration vector of a free particle is the net fore the particle mass. Neglecting gravity, and using Eq. (5.3), wehne 6x 10-19 | Dab Tost (R4 = 10°) x (82-23) = 92.1 x10! (als). Problem5.2 When a particle with charge g and isi 3 Panes a Wid B suai that thetoltial velocity of Adlai 1B, as shown in Fig. 5-31 (P5.2), the magnetic force exerted on the parti tomove ina circle of radius 2. By equating Fy, to the centripetal ee z determine a in terms of g, m, u, and B, Solution: The centripetal force acting on the particle is given by =m Figure P52: Particle of charge 9 projected with velocity u into a medium uniform field B perpendicular to u (Problem 5.2). CHAPTER 5 179 Equating F, to Fa given by Bq, (5.4), we have mu /a = quBsin®, Since the magnetic feld is perpendicular to the particle velocity, sin@ = 1. Hence, a = mu/qB. Problem 53 The circuit shown in Fig. 5-32 (P5.3) uses two identical springs to supporta 10-cm-long horizontal wire with a mass of 5 g. In the absence of a magnetic field, the weight of the wire causes the springs to stretch a distance of 0.2 em each. ‘When a uniform magnetic field is tarned on in the region containing the horizontal wire, the springs are observed to stretch an additional 0.5 cm. What is the intensity of the magnetic ffux density B? Te aw i i Figure P5.3: Configuration of Problem 5.3. Solution: Springs are characterized by a spring constant k where F = kd is the force exerted on the spring and d is the amount the spring is stretched from its rest configuration. In this instance, each spring sees half the weight of the wire: Folmg=id, px BE SXI07 x98 = bmg = kd. = E , SRI RSS Wd ~ axaeios ~ 1225 Wim) ‘Therefore, when the springs are further stretched by an additional 0.5 cm, this amounts to an additional force of F = 12.25Ném x (5x 10-3 m) = 61.25mN per spring, or a total additional force of F = 0.123NN. This force is equal to the force exerted on the wire by the interaction of the magnetic field and the current as described by Eq, (5.12): Fm = Jé xB, where and B are at right angles. Moreover ] LT TT ae SR ETA 180 CHAPTER 5 £x B is in the downward direction, and / = V/R = 12.V/4Q=3 A. Therefore, [Emi WB, Bi= > Problem 5.4 The rectangular loop shown in Fig. 5-33 (P5.4) consists of 20 closely ‘wrapped tums and is hinged along the z-axis. The plane of the loop makes an angle of 30° with the y-axis, and the current in the windings is 0.5 A. What is the magnitude of the torque exerted on the loop in the presence of a uniform field B = $1.2 T? When viewed from above, is the expected direction of rotation Clockwise or counterclockwise? Figure PS.4: Hinged rectmgular loop of Problem 5.4. Solution: The magnetic torque on a loop is given by T = mx B (Bq. (5.20)). where m= ANIA Eq, (5.19)). For this problem, itis given that / = 0.5. A, N'= 20 tums, and ~2c0s30' +9sin30*. Therefore, 80.8 (A-m?) and T = 10.8 (A-m®) x $1.2T = —20.83 (N-m).. As the torque is negative, the direction of rotation is clockwise, looking from above. Problem 5.5 In a cylindrical coordinate system, a 2-m-long straight wire carying a current of 5 A in the positive z-direction is located at r = 4 em, @ = %/2, and -Imszsim (a) IfB = £0.2cos (1), what is the magnetic force acting on the wire? CHAPTER 5 181 Figure P5.5: Problem 5.5. (0) How much work is required to rotate the wire once about the z-axis in the negative 6-direction (while maintaining r = 4 cm)? (©) At what angle @ is the force a maximum? Solution: ‘) MxB = 522x [F0.2c0s¢] = $2080. ~&2c08(n/2) =0. 2% (i, | [Ceat= [Peecoseltrae 8x 107? |sing* =0. = -m [coseae| ‘The force is in the +$-direction, which means that rotating it in the —}-direction would require work. ‘However, the force varies as cos, which means it is positive 182 when —1/2 < o < n/2 and negative over the second half of the circle. Thus, provided by the force between = x/2 and @ = —n/2 (when rotated in —4 direction), and work is supplied for the second half of the rotation, resulting. net work of 2e70. (©) The force F is maximum when cos6 = 1, or $= 0. Problem 5.6 A 20-turn rectangular coil with side / = 15 em and w= Semis pl in the +z plane as shown in Fig. 5-34 (P5.6). Figure P5.6: Rectangular loop of Problem 5.6. (@) If the coil, which carries a current flux density = 10 A, is in the presence of a magnetic determine the torque acting on the coil. (©) At what angle @ is the torque zero? (©) At what angle @ is the torque maximum? Determine its valve. Solation: (@) The magnetic field is in direction (2+ 92), which makes an angle 0 = tan! 3 = 63.48" “The magnetic moment of the loop is m = GNIA = 20x 10x (15x5)x10 = a1.5 (Am), CHAPTER 5 183 Figure P56: (2) Direction of B. ‘where fis the surface normal in accordance with the right-hand rule. When the loop is in the negative-y of the y-z plane, & is equal to Z, but when the plane of the loop is moved to an angle 6, fi becomes Seoso+Jsing, T=mxB=a15x2x107%(2+92) (cos + Hsing) 15x 2x 10-7(4+92) = 43x 1077[2cos@—sing] (N-m). (b) The torque is zero when 2c0so—sing = 0, or tang=2, = 63.43° or 116.57". ‘Thus, when # is parallel to B, T = 0. (©) The torque is a maximum when & is perpendicular to B, which occurs at = 63.43 £90" = —26.57° or +153.43°. Mathematically, we can obtain the same result by taking the derivative of T and ceqaating it to zero to find the values of @ at which {| is a maximum. Thus, Hox 10-*(2cos- sing)) = 0 Iwi naarenarre CRT 186 CHAPTER 5 =2sing+c0s9=0, which gives tang= 26.57° or 153.43°. Section 5-2: Biot-Savart Law Problem 5.7 An 8 cm x 12 cm rectangular loop of wire is situated in the x-y plane with the center of the loop at the origin and its long sides parallel to the x-axis. ‘The loop has a current of 25 A flowing with clockwise direction (when viewed from above). Determine the magnetic field at the center of the loop. Solution: The total magnetic field is the vector sum of the individual fields of each of the four wire segments: B= By +B; +B; +B:. An expression for the magnetic field from a wire segment is given by Bq. (5.28). Figure P5.7: Problem 5.7. Forall segments shown in Fig. P5,7, the combination of the direction of the current and the right-hand role gives the direction of the magnetic field as —z direction at the origin. With r = 6 cm and / = 8 cm, alt B= 4g ARX1077 250.08 rex 0.06 x V4 x0.062-+ 0.08 2462x1075 (T). CHAPTER 5 185 For segment 2,7 = 4.cmand/'= 12cm, wll 4x 10-7 x 25 x 0.12 2m x 0.08 x V4 x 0.08 +012" -210.40x10- (m), 2462x105 (1), By= 210.40 1075 (fT). 20.30 (mT). Problem 5.3 Use the approach outlined in Example 5-2 to develop an expression for the magnetic field Hat an arbitrary point P due to the linear conductor defined by the geometry shown in Fig. 5-35 (P5.8). If the conductor extends berween z; = 3m and za = 7 m and camties a cumrent / = 5 A, find Hat P(2,0,0). P62) Figore P5.8: Current-carrying linear conductor of Problem 5.8. Solution: ‘The solution follows Example 5-2 up through Eq. (5.27), but the expressions for the cosines of the angles should be generalized to read as. zz goa ama * fP+e-ay cos®; = q | 186 CHAPTER 5 instead of the expressions in Eq. (5.28), which are specialized to a wire centered at the origin. Plugging these expressions back into Eq. (5.27), the magnetic field is given as wag {are | 4 \ [pena VP+e-2 For the specific geometry of Fig. P5.8, a es -yesl Snx2 VPP VEE. = $25.8 x 10"? (Am) = $25.8 (mA/m). Problem 59 The loop shown in Fig. 5-36 (P5.9) consists of radial lines and segments of circles whose centers are at point P. Determine the magnetic field at Pin terms of a, b, 8, and /. Figure P5.9: Configuration of Problem 5.9, Solution: From the solution to Example 5-3, if we denote the z-axis as passing out of the page through point P, the magnetic field pointing out of the page at P due to the current flowing in the outer are iS Hauer = ~2/0/4nb and the field pointing out of the page at P due to the current Rowing in the inner arc is Hiayer = 2/0/4na. The other wire segments do not contribute to the magnetic field at P. Therefore, the total field flowing direcdy out of the page at is B= Bogert Hane = 22 (2-3) =e) i Problem 5.10 An infinitely long, thin conducting sheet defined over the space OS x is, from Eq. (335), B pan 2, Equating the magnitudes of the two fields, we obiain the result Ne_ 2a” Bnd? or 2a _ 1x50 & =08A, DaNd ~ zx2x10 Problem 5.12 Two infinitely long, parallel wires carry 6-A currents in opposite Z directions. Determine the magnetic fox density at point P in Fig. 5-38 (5.12). | Figure P5.12: Arrangement for Problem 5.12. sna Solution: Boh 5a a 20.5) *? 2n(15 Po 80 2(642)=32 om, 190 CHAPTER 5 Problem 5.13 A long, East-West oriented power cable carrying an unknown current / is at a height of 8 m above the Earth’s surface. If the magnetic flux density recorded by a magnetic-field meter placed at the surface is 12 T when the current is, flowing through the cable and 20 uT when the current is zero, what is the magnitude of? Solution: The power cable is producing a magnetic flux density that opposes Earth’s, own magnetic field. An East-West cable would produce a field whose direction at the surface is along North-South. The flux density due to the cable is B= (20-12) iT = Sut. ‘As a magnet, the Earth’s field lines are directed from the South Pole to the North Pole inside the Earth and the opposite on the surface. Thus the lines at the surface are from North to South, which means that the field created by the cable is from South to North. Hence, by the right-hand rule, the current direction is toward the East. Its ‘magnitude is obtained from 8ut=8x10%= which gives / = 320A. Problem 5.14 ‘Two parallel, circular loops carrying a current of 20 A each are arranged as shown in Fig. 5-39 (P5.14). The first loop is situated in the x-y plane with its center at the origin and the second loop’s center is at z= 2m. If the two loops have the same radius a= 3 m, determine the magnetic field at: @ Solution: The magnetic feld due to a circular loop is given by (5.34) for a loop in the ary plane carrying a current / in the +@-direction. Considering that the bottom loop in Fig, P5.14 is in the x-y plane, but the current direction is along —$, 3 Wwe where z is the observation point along the z-axis. For the second loop, which is at a height of 2 m, we can use the same expression but z should be replaced with (2-2). Hence, B= - CHAPTER 5 191 VY Hi Figure P5.14: Parallel circular loops of Problem 5.14. ‘The total field is Ia 1 1 wemtm--2@ [4 | ae mitBe ey ert areal = (a) Atz = 0, and with a= 3m and. 204, 20x9 fl 1 ai H= (tel 25.25 Afm. (b) Atz= 1 m (midway between the loops): 20x9 1 1 2 (sae sal W859 Alen (©) Atz = 2m, H should be the same as at z= 0. Thus, Section 5-3: Forces between Currents Problem 5.15 The long, straight conductor shown in Fig, 5-40 (P5.15) lies in the plane of the rectangular loop at a distance d = 0.1 m. The loop has dimensions 192 CHAPTER 5 bosm cy haze aoe Figure P5.15: Current loop next to a conducting wire (Problem 5.15). a= 0.2 mand b= 0.5 m, and the currents art fy = 10 A and h = 15 A. Deiermine the net magnetic force acting on the loop. Solution: The net magnetic force on the loop is due to the magnetic field surrounding the wire carrying current /. The magnetic forces on the loop as a whole due to the current in the loop itself are canceled out by symmetry. Consider the wire carrying Jy to coincide with the z-axis, and the loop to lie in the +x side of the a-z plane. Assuming the wire and the loop are surrounded by free space or other nonmagnetic material, Eq. (5.30) gives ‘Then, from Eq, (5.12), the force on the side of the loop nearest the wire is > Emi =htxB= 16s) (sfit)| =~! “The force on the side of the loop farthest from the wire is, boliab lanesa™ *2a(a+-4)” Fra = xB = h(—2) x ( | CHAPTER S 193 ‘The other two sides do not contribute any net forces to the loop because they are ‘equal in magnitude and opposite in direction. Therefore, the total force on the loop is, FeFm+Fro polled . « uoliheb 2nd ~"2n{atd) woh baab ‘Badlas dy _gf8X 1077 x10 15 0.20.5 Mx O.1x03 ‘The force is pulling the loop toward the wire. =-8 ~2107* (N) = -20.1 (aN). Problem 5.16 In the arrangement shown in Fig. 5-41 (P5.16), each of the two long, ‘parallel conductors carries a current /, is supported by 8-cm-long strings, and has 2 ‘mass per unit length of 0.3 g/cm. Due to the repulsive force acting on the conductors, ‘the angle 6 between the supporting strings is 10°. Determine the magnitude of # and the relative directions of the currents in the two conductors. © © Figure P5.16: Parallel conductors supported by strings (Problem 5.16). Solution: While the vertical component of the tension in the strings is counteracting ‘the force of gravity on the wires, the horizontal component of the tension in the strings is counteracting the magnetic force, which is pushing the wires apart. According to Section 5-3, the magnetic force is repulsive when the currents are in opposite directions. Figure P5.16(b) shows forces on wire 1 of part (a). The quantity F" is the tension force per onit length of wire due to the mass per unit length m! = 0.3 g/em. The i i i 194 CHAPTER 5 vertical component of F’ balances out the gravitational force, Rams, ° where g = 9.8 (mv/s*). But Fi = F'cos(8/2) oy Hence, ee Faso ap ‘The horizontal component of F’ must be equal to the repulsion magnitude force given by Eq. (5.42): = oP ol? B= ond = Dalai O/D) ay where d is the spacing between the wires and ¢ is the length of the string, as shown in Fig. P5.16(c). From Fig. 5.160), sin(@/2) = m'gtan(9/2). 3) A= F'sin(8/2) = men Baquating Eqs. (12) and (13) and then solving for J, we have - anli'g__. og, [RXOOBXOUBXOS Fs sim(8/2)) e077 SBS — ato Peass? Problem 5.17 An infinitely long, thin conducting sheet of width w along the _x-direction lies in the x-y plane end carries a current / in the — y-direction. Determine (@) the magnetic field at a point P midway between the edges of the sheet and at height h above it (Fig. 5-42 (P5.17), and then (b) determine the force per unit length ‘exerted on an infinitely long wire passing through point P and parallel to the sheet if the current through the wire is equal in magnitude but opposite in direction to that carried by the sheet. Solution: (2) The sheet can be considered to consist of a lange number of infinitely long but narrow wires each dx wide lying next to each other, with each canying 2 current CHAPTER $ 195 Figure P5.17: A linear current source above 2 current sheet (Problem 5.17). Iq = Idz/w. If we choose the coordinate system shown in Fig. P5.17, the wire at a distance x from the origin is at a distance vector B from point P, with R txt th. ‘Equation (5.30) provides an expression for the magnetic field due to an infinitely long wire carrying a current J as ‘We now need to adapt this expression to the present situation by replacing / with J, = I dx/w, replacing r with R = (x° +4)'/?, and by assigning the proper direction for the magnetic field. From the Biot-Savart law, the direction of H is governed by 1c R, where lis the direction of current flow. In the present case, 1 is out of the page, which is the -¥ direction. Hence, the direction of the field is IxR TxR] §x(-Sep2h) _ ~(#h+2x) = 9x (Erte SR ‘Therefore, the field dH due to current J, is di= —(#h+2x) i, _ (ah +ie)de = (PSR IaR ~ Deol PY and the total field is Hf Tax —(&h+ 4x) fran /2 ¢ ee) Tf is Been Cer en de (uf, Brett 18(0,0,h) iincialacinslith — Aibcnaiitansints i i & 196 CHAPTER 5 -1 (w (hae A) [E cagme =) Ian) = Fn” (, (Aim). we At P in Fig. P5.17, the field is pointing to the left. The z-component could have been assumed zero with a symmetry argument. An alternative solution is to employ Eq, (5.242) directly. (®) From Eq, (5.9), a differential force is of the form dFm = dl xB or, assuming i = 4 dl, the force per unit length is given by = xB=19x (-2ttf a ‘The force is repulsive; the wire is experiencing a force pushing it up. Problem 5.18 Three long, parallel wires are aranged as shown in Fig. 5-43 (P5.18(@)). Determine the force per unit length acting on the wire carrying Js. T © =a $28 y= 104 yh Oh-= Figure PS.18: (a) Three parallel wires of Problem 5.18. Solution: Since f and J; are equal in magnitude and opposite in direction, and equidistant from /s, oor intuitive answer might be that the net force om Js is 2er0. AS 197 { Figure P5.18: (c) Forces acting on J5. ‘we will see, that’s not the correct answer. The field due to /; (which is along 7) at location of fs is B= ‘where 5 is the unit vector in the direction of By shown in the figure, which is 198 CHAPTER 5 perpendicular to ft. The force per unit length exerted on is is Ks ‘Similarly, the force per unit length excited on Js by the ficld due to Jb (which is along — moon “The two forces have opposite components along # and equal components along 2. ‘Hence, with Ry = Ro = V8 mand 6 = sin“!(2/V8) = sin” (1/2) = 45°, hts uolels Bat e8ia ne (St + Be) sine =2 (2 Problem 5.19 A square loop placed as shown in Fig. 5-44 (P5.19) has 2m sides and ‘carries a current /, = 5A. Ifa straight, long conductor carrying a current Jz = 10 Ais introduced and placed just above the midpoints of two of the loop’s sides, determine the net force acting on the loop. Figure P5.19: Long wire carying current J, just above a square loop carrying J; (Problem 5.19). Solution: Since J is just barely above the loop, we can treat it as if it's in the same plane as the loop. For side 1, fy and J are in the same direction, hence the force on CHAPTER 5 199 side 1 is attractive. That is, soliba 2a(a/2) J; and Jy are in opposite directions for side 3. Hence, the force on side 3 is repulsive, which means itis also along f. That is, Fy = Fy ‘The net forces on sides 2 and 4 are zero. Total net force on the loop is y =F 92x10 N. F=2F =§4x107N. Section 5-4: Gauss’s Law for Magnetism and Ampére’s Law Problem 520 Current / flows along the positive z-direction in the inner conductor ‘of a long coaxial cable and retnrns through the outer conductor. The inner conductor hhas radius a, and the inner and onter radii of the outer conductor are b and ¢, respectively. (@) Determine the magnetic field in each of the following regions: 0 e. (b) Plot the magnitude of H as a function of r over the range from + = 0 to r= 10cm, given that! = 10 A,a=2cm,b=4.cm, and e= Sem. Solution: (a) Following the solution to Example 5-5, the magnetic field in the region r < a, a are and in the region a ¢, the total enclosed current is zero: the total current flowing on the inner conductor is equal to the total current flowing on the outer conductor, but they are flowing in opposite directions. Therefore, = 0. (b) See Fig. P5.20. os o7 06 os. oa Magnetic field magnitude (Jom) Radial distance r (em) Figure P5.20: Problem 5.20). nn Ue Problem 52% A long cylindrical conductor whose axis is coincident with the z-axis hhas a radius @ and carries a current characterized by 2 current density J = 2o/r, where Jp is a constant and r is the radial distance from the cylinder’s axis. Obtain an expression for the magnetic field H for (a) 0 a. Solution: This problem is very similar to Example 5-5. (a) For 0 < ry < a, the total current flowing within the contour Cy is LE LF) Grareey=2e |" doar= CHAPTERS 201 Therefore, since, = 2urjH), Hy = Jp within the wire and Hy = $Jp. (®) Forr> a, the total current flowing within the contouris the total eurrent flowing within the wire: I= [fs-as= [™ f°, (2) erdras)=28 [" soar nah, ‘Therefore, since jga/'r within the wire and Ee = 4Jo(a/r). nH, Hy Problem 522 Repeat Problem 5.21 fora current density J = 2/oe~”. z Figore P5.22: Cylindrical current. Solution: uoaaiiibinuanala imalsiaciatewn ‘ E 202 CHAPTER 5 (@) For r (ore asm EER SReesRs)-(Baxdy) (fg. cosmy 0125/0325 = (ea) ns F(=@-=(@))-° (¢) From Eg. (5.67), ©= faa, where Cis the square loop in the x-y plane with sides of length 0.25 m centered at the origin. Thus, the integral can be written as @= fi b-dt= Stn + Sia St + Signe where Stone Spats Sie: and Sapa 27° the sides of the loop. es sas Stone = £ arag S284 224 SiR) Ipe_g325-(84x) ' [OSes a = a ' Jea—0.125 y= 0.125 = ((Sxc0s%y)},.-0125) | ous oxas In=-0.125 Soack = (Boosay+2(2-+sinxx))|, 0.195 fx~0.125 uss t Eras 58 aonas et frm0.125 e125 5 (®) ~ (Sern ans) =f) oe Sit = [yas S08 +124 sn) asas (94) CHAPTER 5 205 ax2s = frag b-on2sdy = 0, oazs 5 Saote = [ER Seo +H +n cos (94y) pa ai ano = [aes Jxnorssdy = 0. As dl = Strom + Spock + Store + Seigne Problem $25 A uniform current density given by J=2lo (Alm), gives rise to a vector magnetic potential A=-i soheia +9) (Woim). (@) Apply the vector Poisson’s equation to confirm the above statement. (©) Use the expression for A to find HL. (©) Use the expression for Jin conjunction with Ampére’s law to find Hl. Compare your result with that obtained in part (b). Solution: @ VA =RVALF9V7A, +2074, 32 (2 + ge 3) [-=? erry] 206 CHAPTER 5 Figure P5.26: Current cylinder of Problem 5.26. © B= $4, =645 ‘We need to convert the expression from cylindrical to Cartesian coordinates. From Table 3-2, § = -Ssing + Foosg = ra VBe¥ Seat . as ES CHAPTER 5 207 Hence ye) yee ath ay Bh (eae) PR which is identical with the result of part (b). Problem 5.27 A thin current element extending between z = -L/2 and z = L/2 carries a current / along +2 through a cireular cross section of radius a. {a) Find A ata point P located very far from the origin (assume Ris so much larger than Z that point P may be considered to be at approximately the same distance from every point along the current element). (b) Determine the corresponding H. z Cross-section na? Figure P5.27: Current element of length L observed at distance R >> L. Solution: (2) Since R > L, we can assume that P is approximately equidistant from all segments of the current element, Hence, with R treated as constant, (5.65) gives aif Stee f tele HP 4. a gba a8 Lee alia? 5 208 CHAPTER 5 Section 5-6: Magnetic Properties of Materials Problem 5.28 In the model of the hydrogen atom proposed by Bobr in 1913, the electron moves around the nucleus at a speed of 2 x 10° m/s in a circular orbit of radius 5x 10-'! m, What is the magnitude of the magnetic moment generated by the electron’s motion? Solution: From Eq. (5.69), the magnitude of the orbital magnetic moment of an electron is Feur| = $x 1.6x 107 x2x 10% x5x 10" =8x 10 (A-m?). Problem 5.29 Iron contains 8.5 x 10° atoms/m?. At saturation, the alignment cof the electrons’ spin magnetic moments in iron can contribute 1.5 T to the total magnetic flux density B. If the spin magnetic moment of a single electron is 9.27 x 10-™* (A-m*), how many electrons per atom contribute to the saurated field? Solution: From the first paragraph of Section 5-6.2, the magnetic flux density of a magnetized material is By = gM, where M is the vector sum of the microscopic magnetic dipoles within the material: M = N-im,, where m, is the magnitude of the spin magnetic moment of an electron in the direction of the mean magnetization, and ‘Ne is net number of electrons per unit volume contributing to the bulk magnetization. {if the number of electrons per atom contributing to the bulk magnetization is me, then Ne = reNatoms Where Newoos = 8-5 x 1078 atoms/m' is the aumber density of atoms foriron, Therefore, B 1s HomNeoms EXIT XO.2T X10 KES x 108 = 1.5 (electrons/atom). CHAPTER 5 209 Section 5-7; Magnetic Boundary Conditions Problem $30 The xy plane separates two magnetic media with magnetic permeabilities yn and pz, 2s shown in Fig. 5-45 (P5.30). If there is no surface current at the interface and the magnetic fielé in medium 1 is By = Shi + FMiy + ie, find: @) Eb, (@) 8 and @, and (©) evaluate Hy, 01, and 62 for Hie = 3 (A/m), My = 0, His =4 (Alm), pn = pos and 2p = 4p. By : Be —- - my plane eM Figure PS.30: Adjacent magnetic media (Problem 5.30). Solution: (2) From (5.80), Bila = poHn. ‘and in the absence of surface currents at the interface, (5.85) states Hy = Bo In this case, Hi: = Hig, and Mi, and Hy are tangential fields. Hence, inHg = salto, | Fiz = He, Ay = Bay, 210 CHAPTER 5 Hb= Big t IHiy tHe ) © Hh=8342]-4e83+42 (Atm), 3 0, = tant 2) = 36.86", = tan? (3) =1357. ‘exists at y = 0, the interface between two magnetic media, and Hy = 28 (A/mm) in medium 1 (y> 0), determine Hz in medium 2 (y < 0). Solution: 5,=24A/m, Hy =28A/m. 1, is tangential to the boundary, and therefore Hp is also. With fg (6.84), we have CHAPTER 5 au Figure PS.31: Adjacent magnetic media with J, on boundary. ‘which implies that Hy does not have an 2-component. Also, since pn Flty = s2Hoy and Hy does not have a y-component, it follows that Ha does not have a y-component either. Consequently, we conclude that Hy =24. —_— Problem 532 In Fig. $-46 (P5.32), the plane defined by x—y = 1 separates ‘medium 1 of permeability pn from medium 2 of pecmeability pp. If no surface current exists on the boundary and B= 2433 M, find B, and then evaluate your result for jy = Syn. Hint: Start out by deriving the ‘equation for the unit vector normal to the given plane. Sclution: We need to find fg. To do so, we start by finding any two vectors in the plane x—y= 1, and to do that, we need three non-collinear points in that plane. We choose (0,~1,0),(1,0,0), and (1,0,1). ‘Vector Ar is from (0,—1,0) 10 (1,0,0): A Skt yl. ‘Vector Aa is from (1,0,0)t0 (1,0,1): Hence, if we take the eross product Aa x Ar, we end up in a direction normal to the given plane, from medium 2 to medium 1, AaxAr _ 21x (R1+F1) [Aa x Ail 22 CHAPTER 5 Figure P5.32: Magnetic media separated by the plane x—y = 1 (Problem 5.32). Ip medium 1, normal component is, ‘Tangential component is By = Bi — Bi = (#2493) — 5) 2254925. ‘Boundary conditions: Hy = Ho, ot B= BH ms ° we 4 = 2p, = 29254925). Bu = By = F825 +925). Finally, 9254925). mentaita (2-2)+2 CHAPTER 5 213 ee Problem 5.33 The plane boundary defined by z iron, IF By for iron. Solution: From Eq. (5.2), 0 separates air from 2 block of 54 — 96-4 28 in air (2 > 0), find Bp in iron (z < 0), given that p= 5000H9 Bot -s Hy = — = — (84-96 + 28). ba aft ). “Te z component is the normal component to the boundary at z= 0. Therefore, from Eq, (5.79), Bas = Bi; = 8 while, from Bg. (5.85), Hay = Hy = Bay = wnHoy = ty = 5000. Therefore, By = £20000 — 930000 + 28. ——_—————_——_————— Problem 5.34 Show that if no surface current densities exist at the parallel interfaces shown in Fig. 5-47 (P5.34), the relationship between @, and & is ‘independent of p- Figure P5.34: Three magnetic media with parallel interfaces (Problem 5.34). 214 CHAPTER 5 Solution: 8 a mmr 5." and : a eB But Bay = Bin and 22 = 22. Hence, as ‘We note that @2 = 03 and tans which is independent of 12. nnn Sections 5-8 and 5-9: Inductance and Magnetic Energy Problem 5.35 Obtain an expression for the selfinductance per unit length for the parallel wire transmission line of Fig. 5-27(a) in terms of a, dl and w, where a is the radius of the wires, d is the axis-to-axis distance between the wires, and is the permeability of the medium in which they reside. Solution: Let us place the two wires in the 2-2 plane and orient the current in one Of them to be along the +-z-direction and the current in the other one to be along the “z-direction, as shown in Fig. P5.35. From Eg. (5.30), the magnetic field at point ‘P(x,0,2) due to wire 1 is ‘where the permeability has been generalized from free space to any substance with permeability y, and it has been recognized that in the x-z plane, $= 9 and r= as Tongs x > 0. Given that the current in wire 2is opposite that in wire 1, the magnetic field created by wire 2 at point P(z,0,2) is in the same direction as that created by wire 1, and itis given by CHAPTER 5 25 Figure P3.35: Parallel wire transmission line. endorse meget elie sheen even Wi 1 wd E-Bthes ois aa) ima)" rete (201 acestgte sie ee loaponn te es ne SIA Sra nisnneca = LO LT sata) 80 2 (@E)IC -& (o()-»G)) ~Baa(t) =a) 216 CHAPTER 5 Since the number of “tums” in this structure is 1, Eq. (5.93) states that the flux linkage is the same as magnetic flux: A = ®. Then Ea, (5.94) gives a total inductance over the length / as ‘Therefore, the inductance per unit length is ve$=#n(2)=2i(2) cum, where the last approximation recognizes that the wires are thin compared to the separation distance (i.e., that d > a). This has been an implied condition from the deginning of this analysis, where the flux passing through the wires themselves have ‘been ignored. This is the thin-wire limit in Table 2-1 for the two wire line. ‘Problem 536 A solenoid with a length of 20 cm and a radius of 5 cm consists ‘of 400 ams and carries a current of 12 A. If z= 0 represents the midpoint of the solenoid, generate a plot for |H(z)| as a function of z along the axis of the solenoid for the range —20 em < z < 20cm in 1-cm steps. Solution: Let the length of the solenoid be 1 = 20cm. From Eq, (5.88a) and Eq. (G.88b), z= atan@ and a? +1? = a* sec? 8, which implies that 2/V2 +a? Generalizing this co an arbiary observation point z' on the axis of the solenoid, =2)/4/(2-z) +a? = sin8. Using this in Eq, (5:89), B al . FFP int sine) H(0,0,2/) al ( 12 _ =z 2\ fue-2P4+e2 (-172-2P +2 #( yonz 1242 tes) wim). V2— Ze Vath +e, A plot of the magnitude of this function of z’ with a= 5 em, n = 400 turns/20 em = 20,000 turns/mm, and J = 12 A appears in Fig. P5.36. 27 Magnitude of Magnetic Field 11 (ki SSS Pesition on axis of solenoid 2 (em) Figure 5.36: Problem 5.36. —_— Problem 537 Interms of the d-c current /, how much magnetic energy is stored in {he insulating medium of a 2-m-long, air-filled section of a coaxial transmission line, given thet the radius of the inner conductor is 5 em and the inner radius of the outer ‘conductor is 10cm? Solution: From Eq. (5.99), the inductance per nit length of an air-filled coaxial cable is given by Hoi (2 Sin (2) im). ‘Over a length of 2 m, the inductance is 2x4mx 10-7 baa —“oe »(2) =277x107 From Bq, (5.104), Wm = LP? /2.= 1397 (al), where Wn is in nanojoules when Jis ip amperes. Alternatively, we can use Eq. (5.106) to compute Win! Wo= tf potPav. as | RT TNC : 218 (CHAPTER 5 From Eq. (5.97), H = B/o = 1/2nr, and “ape pod ge We = SEE Lag) rdrdode=139P (a3). Problem 5.38 The rectangular loop shown in Fig. 5-48 (P5.38) is coplanar with the long, straight wire camying the current | = 20 A. Determine the magnetic flux through the loop. Figure P5.38: Loop and wire arrangement for Problem 5:38. Solution: The field due to the long wire is, from Eg. (5.30), sat a, where in the plane of the loop, $ becomes ~% and r becomes y. ‘The flux through the loop is along —, and the magnitude of the flux is o= [a aa (PO 3 ~8(30 em x dy) = »Y Hol 3[-2 = x03 [2 2a < Ioos y 0 02\ _ 6 xa0xin (33) = 66x 10-6 (Wh). CHAPTER 6 219 Chapter 6 Sections 6-1 to 6-6: Faraday’s Law and its Applications Problem 6.1 The switch in the bottom loop of Fig. 6-17 (P6.1) is closed at r = 0 and then opened at a later time r. What is the direction of the current / in the top loop (clockwise or counterclockwise) at each of these two times? Re AA “Ny RX Figure P6.1: Loops of Problem 6. Solution: The magnetic coupling will be strongest at the point where the wires of the two loops come closest. When the switch is closed the current in the bottom loop ‘will start to flow clockwise, which is from left to right in the top portion of the bokom loop. To oppose this change, a current will momentarily flow in the bottom of the top loop from right to left. Thus the current in the top loop is momentarily clockwise when the switch is closed. Similarly, when the switch is opened, the current in the top loop is momentarily counterclockwise. a Problem 62. The loop in Fig. 6-18 (P6.2) is in the x-y plane and B = 2Bqsinar ‘with Bp positive. What is the direction of / (6 or ~8) at (a) r= 0, (&) or = n/4, and Oo=n/2? Solution: / = Vemg/R. Since the single-tam loop is not moving or changing shape with time, VE.= 0 V and Very = Vii Therefore, from Eq. (68), ale If we take the surface normal to be +2, then the right hand rule gives positive flowing current to be in the +6 direction, [= Wag) R= 220 CHAPTER 6 Figure P6.2: Loop of Problem 6.2. where 4 is the area of the loop. (2) A, @ and R are positive quantities. At = 0, cosar = 1 so J < 0 and the ‘current is flowing in the —¢ direction (so as to produce an induced magnetic field that opposes B). (b) Ateor = 1/4, cose = V2/2 so I <0 and the current is still flowing in the ~ direction. ©@aAtar -/2, cose = 050 = 0. There is no current flowing in either direction. —_— Problem 63 A coil consists of 100 ums of wire wrapped around a square frame of sides 0.25 m. The coil is centered at the origin with each of its sides parallel to the x- or y-axis. Find the induced emf across the open-circuited ends of the coil if the magnetic field is given by @ B=810* (0, (@) B=210c0sx cos 10% (1), {c) B= 210cosz sin2y cos 10° (1). Solution: Since the coil is not moving or changing shape, Vy = OV and Verat = VE. From Eq. (6.6), w§ fren -0§ (2 [Sp coon vihere N = 100 and the susface normal was chosen to be in the +2 direction. (@)For B= #10e* (1), Vent = 1004 (10¢-*(0.25)4) = 252 (V). CHAPTER 6 221 (®) For B = 210cosxcos 10% (T), a 5, pois pans Vee = ~1002. (:ocesie% L [ cosxard) at Jx=—0.125 Sy=—0,125 62.3sin10% (KV). (©) For B = 210 cosxsin2ycos 10%: (1), a as pots Vent = —1005 (soos L L coszsin2ydxd) =0. a |em—0125 Jyao.s2s' Problem 64 A stationary conducting loop with internal resistance of 0.5 Q is placed in a time-varying magnetic field. When the loop is closed, a current of 2.5 A flows through it. What wil the current be if the loop is opened to create a small gap ‘and a 2-2 resistor is connected across its open ends? Solution: Vong is independent of the resistance which is in the loop. Therefore, when the loop is intact and the internal resistance is only 0.5.2, Vent = 2.5 AX05.2 25V. ‘When the small gap is created, the total resistance in the loop is infinite and the current flow is zero, With a 2-0 resistor in the gap, 1 = Vens/(20+0.5 2)=125V/25Q=05 (A) Problem 6.5 A circular-loop TV antenna with 0.01 m? area is in the presence of a ‘niform-amplitude 300-MHz signal. When oriented for maximum response, the loop Gevetops an emf with a peak value of 20 (mV). What is the peak magnitude of B of the incident wave? Solution: TV loop antennas have one turn. At maximum orientation, Ea. (6.5) evaluates to ® = [B-ds = BA for 2 loop of area A and a uniform magnetic field with magnitude B= |B|. Since we know the frequency of the field is f = 300 MEIz, ‘we can express B as B = By cos (wt +d9) with @ = 2x x 300 x 10° rad/s and cg an arbitrary reference phase. From Eq. (6.6), Veot = we = 44 (Bycos(we +00)] = ABpeosin(t +09). Vertis maximum when sin( ae +9) 1. Hence, 20% 1073 = ABow = 107? x By x 6x 10%, 222 CHAPTER 6 which yields By = 1.06 (nA/m), Problem 6.6 The square loop shown in Fig. 6-19 (6.6) is coplanar with a long, straight wire carrying a current i(t) = 2.5cos2nx 10'r (A). (2) Determine the emf induced across a small gap created in the loop. (b) Determine the direction and magnitude of the current that would flow through a4-Q resistor connected across the gap. The loop has an intemal resistance of 12 Figure P6.6: Loop coplanar with long wire (Problem 6.6). Solution: (@) The magnetic field due to the wire is page where in the plane of the loop, and r= y. The flax passing through the loop [+810 (em)] dy = SEX IO x25 cos(2nx 10%) x10" 5 an x 10-7eos(2m x 10%r) CW). 1 0.55 x 2x x 10* sin(2n x 10r) x 10-7 = 3.45x 1073 sin(2nx 10%) (V). &) Vert _ 3.451073 dnt 9 zi sin(2nx 104) =: ).69sin(2e x 10%r) (mA). At t= 0, B is a maximum, it points in —Z-irection, and since it varies as cos(2n x 10%), it is decreasing, Hence, the induced current has to be CCW when looking down on the loop, as shown in the figure. Problem 6.7 The rectangular conducting loop shown in Fig. 6-20 (P6.7) rotates at 6,000 revolutions per minute in a uniform magnetic flux density given by B=950 (mt). ‘Determine the current induced in the loop if its intemal resistance is 0.5 2. Solution: o- [Bas $50 10°3.9(2x3 x 10-4) cose(z) = 3 x 10-Fcoso(#), + area 200m (rads), ®=3x10-5cos(200mt) (Wb), — -2 = 3x 1075 x 200nsin(200nr) = 18.85 x 10-3sin(200m) (V), Vous : : 224 CHAPTER 6 Figure P6.7: Rotating loop in a magnetic field (Problem 6.7). “The direction of the current is CW (if looking at it along —2-direction) when the loop is in the first quadrant (0 < $< x/2). The current reverses direction in the second ‘quadrant, and reverses again every quadrant. Snes Problem 68 A rectangular conducting loop 5 em x10 cm with a small air gap in one of its sides is spinning at 7200 revolutions per minute, If the field B is normal to the loop axis and its magnitude is 5 x 107° T, what is the peak voltage induced across ‘the air gap? Solution: 2n rad/cycle x 7200 cycles/min 60 Simin A=5omx 10 cm/(100 eam)? = 5.0.x 10-3 m?, From Eqs. (6.36) or (6.38), Vems = AcoBo Sings; it can be seen that the peak voltage is VBE AeaBo = 5.0.x 107? x 240nx5 x 10°§ = 18.85 (HV). enn Problem 6.9 A 50-cm-long metal rod rotates about the z-axis at 180 revolutions per minute, with end 1 fixed at the origin as shown in Fig. 6-21 (P6.9). Determine the induced emf Viz if B= 23x 10* T. Solution: Since B is constant, Vear = Vit. The velocity & for any point on the bar is given by u= Gro, where _ 2wradicycle x (180 cyeles/min) ae ed. = 240n rad/s, 225 Figure P69: Rotating rod of Problem 6.. From Bq. (624), 1 Via= Vine [ (axB)-a f (Gone x23 x 10™)-Fadr = 1srx107 [° rar laos = onx10-tf =-9nx 10x 0.25=-707 (1). Problem 6.10 The loop shown in Fig. 6-22 (P6.10) moves away from a wire carrying a current Jy = 10 (A) at a constant velocity u = $5 (mvs). If R= 10 Q and the direction of fy is as defined in the figure, find J as a function of yp, the distance between the wire and the loop. Ignore the intemal resistance of the loop. Solution: Assume that the wire carrying current / is in the same plane as the Loop. “The two identical resistors are in series, so fy = Veme/2R, where the induced voltage is due to motion of the loop and is given by Bq. (6.26): Vera = Vig = fers) a “The magnetic field B is created by the wire canrying /. Choosing £to coincide with the direction of /1, Eq. (5.30) gives the extemal magnetic field of a long wire to be aaeet. i 226 CHAPTER 6 a : Wt veo TO i ie ’ 7 Figure P6.10: Moving loop of Problem 6.10. For positive values of yo in the -z plane, 9 = F, so gels _ poe Dar ler Integrating around the four sides of the loop with dl = %dz and the limits of integration chosen in accordance with the assumed direction of /;, and recognizing that only the two sides without the resistors contribute to Vip we have: Var= [" (2) eet (a) _ 4m x 10-7 x10x5x02(1 1 ) - 2m yo yor 0.1 ax B= 9ful xB = Ful x. (Bde) pay =2x107°(1_-_1 wrcirt(E tg) and therefore ne lferoo(L <3) ow. ‘Problem 6.11 ‘The conducting cylinder shown in Fig. 6-23 (P6.11) rotates about its axis at 1,200 revolutions per minute in a radial ficld given by B=t6 @. CHAPTER 6 227 Sliding contact Figure P6.11: Rotating cylinder in a magnetic field (Problem 6.11). ‘The cylinder, whose radius is 5 em and height 10 em, has sliding contacts at its top and bottom connected to a voltmeter. Determine the induced voltage. Solution: The surface of the cylinder has velocity u given by usdor 5x10? Problem 6.12 The electromagnetic generator shown in Fig. 6-12 is connected to an electric bulb with a resistance of 100 ©. If the loop area is 0.1 m? and it rotates at 3,600 revolutions per minute in a uniform magnetic fix density By = 0.2 T, determine the amplitude of the current generated in the light bulb. Solution: From Ea. (6.38), the sinusoidal voltage generated by the ac generator is Vert = AcaBq sin( «ae +Co) = Vosin( r+ Cp). Hence, 0 0.2=7:54 WM), 4, FBT we a Problem 6.13 The ciscular disk shown in Fig. 6-24 (P6.13) lies in the x-y plane and rotates with uniform angular velocity « about the z-axis. The disk is of radius and is present in a uniform magnetic fux density B = 289. Obtain an expression for the emf induced at the rim relative to the center of the disk. ie pa iIRMMOC, 4 1 j 4 : | SENN e 228 CHAPTER 6 Figure P6.13: Rotating circular disk in 2 magnetic field (Problem 6.13). Figure P6.13: (a) Velocity vector u, Solution: At a radial distance r, the velocity is where 6 is the angle in the x-y plane shown in the figure. The induced voltage is v= “ a= 6 fF dr. Foxe) a [or x28) e x is along #. Hence, vas [rar = Section 6-7: Displacement Current Problem 6.14 The plates of a parallel-plate capacitor have areas 10 cm? each and are separated by 1 cm. The capacitor is filled with a dielectric material with CHAPTER 6 229 € = 4eo, and the voltage across it is given by V(r) = 20cos2n x 10% (V). Find the displacement current. Solution: Since the voltage is of the form given by Eq. (6.46) with Vy = 20'V and @ = 2x 10 rad/s, the displacement corrent is given by Eq. (6.49): FAY si Fvoosinar 4x 8.854 x 107 x 10x 10+ ~ 1x10 = —48Ssin(2x x 105r) WA). 20 x 2 x 10% sin(2r x 106) Problem 6.15 A coaxial capacitor of length / = 6 em uses en insulating dielectric material with €, = 9. The radii of the cylindrical conductors are 0.5 em and 1 om. If ‘the voltage applied across the-capacitor is V(t) = 100sin(120m) (V), ‘what is the displacement current? Figure P6.15: Solution: To find the displacement curreat, we need to know E in the dielectric space ‘between the cylindrical conductors. From Eqs. (4.114) and (4.115), 422 sio(120m) (Vim), 200sin( 120m) rind “a SSoRERRRERN TTR 230 CHAPTER 6 Dar =ec0E £98.85 x 107 x sin(120m2) (C/m”). 3 sin(120n1) 15x 10-8 ‘The displacement current flows between the conductors through an imaginary cylindrical surface of length | and radius r. The current flowing from the outer conductor to the inner conducior along ~F crosses surface S where s Qn. 2a (2iSx10" & = 1.15 x 1078 x 120x x 2mlcos(120n1) = 1.63c05(120n2) WA). sin( 120%) -(-f2ar!) Altematively, since the coaxial capacitor is lossless, its displacement current has to ‘be equal to the conduction current flowing through the wires connected to the voltage sources. The capacitance of a coaxial capacitor is given by (4.116) as 2nel n@’ The coment is av _ 2xel = Ta CF = Fy ltr « Wes 120m)] = 1.630s{ 120m) A), which is the same answer we obtained before. Problem 6.16 The parallel-plate capacitor shown in Fig. 6-25 (P6.16) is filled with a lossy dielectric material of relative permittivity ¢, and conductivity 6. The separation between the plates is d and each plate is of area A. The capacitor is connected to a time-varying voltage source V(t). (@) Obtain an expression for J, the conduction current flowing between the plates inside the capacitor, in terms of the given quamtities. (CHAPTER 6 231 Figure P6.16: Paraliel-plate capacitor containing a lossy dielectric material (Problem 6.18). {) Obtain an expression for Je, the displacement current flowing inside the capacitor. (c) Based on your expression for parts (a) and (b), give an equivalent-circuit representation for the capacitor. (@) Evaluate the values of the circuit elements for A = 2.em?, d= 0.5 em, e,=4, = 2.5 (Sim), and V(z) = 10cos(3n x 10°) (V). Solution: @) 4 R=s @) ¥ ap aE _ Aa. E=5, be Garage So. (© The conduction carreat is directly proportional to V, as characteristic of a resistor, whereas the displacement current varies as 2V /8:, which is characteristic of a capacitor. Hence, (@) 0.5 x 10-2 25x2x 107 232 CHAPTER 6 ve ‘Acuual Cirenit Equivalent Circuit Figure P6.16: (a) Equivalent circuit x 8.85 x 107}? x 2x 10-* © Osx 107 = 142x10-P F. ee Problem 6.17 An electromagnetic wave propagating in seawater has an electric field with 2 time variation given by E = #£qcoscxr. If the permittivity of waver is Slep and its conductivity is 4 (S/m), find the ratio of the magnitudes of the conduction current density to displacement current density at each of the following frequenci (a) 1 kB, (b) 1 MEz, (6) 1 GHz, (4) 100 GHz. Solution: From Eq. (6.44), the displacement current deasity is given by apeed Jem GD=e5E and, from Eq. (4.67), the conduction current is J = OE. Converting to phasors and taking the ratio of the magnitudes, ob s jece,eoE | SEréo i| 5.) (@) Atf = 1 KHz, @= 2n x 10° rad/s, and i Ja ‘The displacement current is negligible. (©) At f= 1 MBz, @ = 2x 10° rad/s, and 4 ~ ERT RET EASE TOT 7 BEX IO. 4 Bmx 109 x BI x 8.854 10" CHAPTER 6 233 ‘The displacement current is practically negligible. (At f= 1 GHz, @ = 20x 10° rads, and 4 Bex TO KET xB aSax TOE ~ OFS ‘Neither the displacement current nor the conduction current are negligible. (@) At f = 100 GHz, @= 2nx 10" rads, and 4 Ppa ig se tor TaN TOT XBL x Sax 10 ~ FSEX 1 ‘The conduction current is practically negligible. Sections 6.9 and 6-10: Continuity Equation and Charge Dissipation Problem 6.18 At? = 0, charge density pyo was introduced into the interior of a material with a relative permittivity & = 4e9. If at ¢ = 1 us the charge density has dissipated down to 1072pyo, what is the conductivity of the material? Solution: We start by using Eq. (6.61) to find =: Pele) = pwoer, or 10"? px0 = woe“, which gives 6 nig = 2 | % o 10-6 i Tea agee SSX IO) Butt, = €/o = 4e9/s. Hence As _ 4x 8.854 x 107? ee 10~* im), E 234 é Problem 6.19 If the current density in a conducting medium is given by U(a.n,zit) = (22-93y? + 22x) cos ar, determine the comesponding charge distribution py(x,9.z5t). Solution: Eq, (6.58) is given by F ‘The divergence of J is Using this result in Eq. (14) and then integrating both sides with respect to p= [ova= where Co is a constant of integration. [Secs ar = Zsinar +0, Problem 6.20 In acertain medium, the direction of carrent density Jp radial direction in cylindrical coordinates and its magnitude is independent and z. Determine J, given that the change density in the medium is Pr =Pporcosar (Cém*) Solution: Based on the given information, J=Suir). Jz = 0, in cylindrical coordinates the divergence is given by 1a vant ee (rd). (Porcosex) poresiner. CHAPTER 6 235 Hence 12 (a) = poresina, rar Zen = PoP asinax, [Zeve= pocsiner [Par rig = (poeosinear) c , siner, Problem 621 If we were to characterize how good a material is as an insulator by its resistance to dissipating charge, which of the following two materials is the better inselator? Dry Soil: ‘Fresh Water Since it takes longer for charge to dissipate in fresh water, itis a betier insulator than dry soil. Sections 6-11: Electromagnetic Potentials Problem 622 The electric field of an electromagnetic wave propagating in air is given by E(z,t) = £400s(6 x 10% —2z} + 93sin(6 x 10%—22) (Vin). 236 CHAPTER 6 Find the associated magnetic field Hi(z,t). Solution: Converting to phasor form, the electric field is given by Eg) = tte — jy3e* (Vim), which can be used with Eq. (6.87) to find the magnetic field: Be) z e 2 ajay a/ae HOH! get _fzeP 0 0 g v 1 = R672 — 9 j8e sant 9 j8e"™*) apes SKI Kae x 107 ‘Converting back to instantaneous values, this is ‘A(t,z) = —£8.0sin(6 x 10% — 22) +910.6cos(6 x 10°r— 22) (mA/m). (26-7 j8)e7/** = j28.0e-7 4+ 910.62" (mA/m). Problem 623 The magnetic field in a dielectric material with: is given by 0, = Ho, and AG! ‘Scos(2nx 10"r+ky) (A/m). ‘Find & and the associated electric field E. Solution: In phasor form, the magnetic field is given by Eq. (6.86), 25239 (Alm). From hvxtt = hse = Fae and, from Eq. (6.87), which, together with the original phasor expression for Hi, implies that _ _ Ve _2nx0Va _ an ke olen = STi = 30 (adtm). CHAPTER 6 2 Inserting this value in the expression for 5 above, 4/30 gts 130 goa CV NiO NaxeseRIO B94le! im): —— Problem 6.24 Given an electric field E = osinaycos( ar — kz), where Bp, a, ©, and k are constants, find B, [5g eosinayer)- ig (eosineye*)] = 2 [pksinay—2 jacosayle"*, H=fe[Hie™) =Me { 2 ipesinay + 2acosaye Me rs} ® 2 [p&sinaycos( ax — ke) +#acosayeos (o-k-F | = 2 [Fksinaycos( x — kz) + 2acosaysin(oe— kz)]- ee Problem 6.25 The electric field radiated by @ short dipole antenna is given in spherical coordinates by E(R.8;1)= sin cos(6mx 10%:-2nR) (W/m). Find H(R,6:t). Solution: Converting to phasor form, the electric field is given by . _.axi0r? (R,0) = 6p = 02=I inte * wie), 238 CHAPTER 6 which can be used with Eg. (6.87) to find the magnetic field: 1 1_ af 518 Ba )= aE yon [*taine ao +42 Re)] Mend 2 gn fak —k Ber =a? singe) 2 - zs 2107 get "exe xanx io 43 =o ysinge Palm). ‘Converting back to instantaneous value, this is (2, 8:2) = 65 sinde0s (6nx 10%—2nR) (Afm). Problem 626 A Hertzian dipole is a short conducting wire carrying an approximately constant current over its length /. If such a dipole is placed along the z-axis with its midpoint at the origin and if the current flowing through it is i(r) = Ipcosee, find (@) the retarded vector potential A(R, 9,6) at an observation point O(R,,6) in a spherical coordinate system, and (b) the magnetic field phasor H(R,@,4). Assume ! to be sufficiently small so that the observation point is approximately equidistant to all points on the dipole; that is, assume that R’ = R. Solution: _ (a) In phasor form, the current is given by 7 = ip. Explicitly writing the volume integral in Eq. (6.84) as a double integral over the wire cross section and a single imegeal over nat, Eh JPR aac, where 5 is the wire cross section. The wire is infinitesimally thin, so that R’ is not a function of x ory and the integration over the cross section of the wire applies only to the current density. Recognizing that J = 2%/s, and employing the relation R’ = R, Xa pte aes CHAPTER 6 239 In spherical coordinates, 2 = Reos8—@sin8, and therefore ie — 8 sino) Hot ae A= (Roos -6si OER (b) From Eq. (6.85), eae R Be = _ hl é VxA = Vx [Roose Gsin@)- Ay z Problem 627 ‘The magnetic field in a given dielectric medium is given by Hi = F6cos2zsin(2x 10r-0.1z) (A/m), where x and z are in meters. Determine: @E (2) the displacement current density Ja, and (©) the charge density Pv. Solution: f) HL= $6c0s2zsin(2 x 10" 0.1.2) = f 6cos2zcos(2 x 107 -0.1x—m/2), B= p6c0s2ze te? = 7 jScos2ze M1, 3 2 a/ey 9/22] 0 —j6cos2ze“P4* 0 tse). From the given expression for H, @=2x10" (rad/s), 240 CHAPTER 6 B=01 (adm). Hence, =2x10° (ms), Using the values for and €, we have (-830sin22+ 2 j1.Scos2z) x 108e4* (Vim), E = [-£30sin2zcos(2 x 107 —0.1x)—21.Scos2zsin(2 x 10'r—0.1x)]_ &V/m). £0.6sin2z-+ 2 j0.03cos2z) x 10-Se™1* (Cim?), Ja = elJee/™] [g12sin2zsin(2 x 1071 — 0.1.x) -20.6cos2zcos{2 x 107-0.1x)] (A/m?), (©) We can find 9, from VD=p or from S v5=-% Applying Maxwell’s equation, a VD aeV-B =e) (2 4 p= VD =eV-E=em (S24 yields Pv = So {2 [-30sin2zeos(2 x 107r-0.1x)] + 2 [+1.Seos2zsin(2 x 107 ~oas)}} = &€q [-3 sin2zsin(2 x 107 — 0.1x) + 3sin2zsin(2 x 107 — 0.12)] = 0. CHAPTER7 241 Chapter 7 Section 7-2: Propagation in Lossless Media Problem 7.1 The magnetic field of 2 wave propagating through a certain nonmagnetic material is given by H=250cos(10%—5y) (mA/m). Find (a) the direction of wave propagation, (b) the phase velocity, (c) the wavelength in the material, (¢) the relative permittivity of the material, and (e) the electric field phasor. Sohution: (a) Positive y-direction. (©) @ = 10° rad/s, k= 5 radi. (A= 2n/k = 2/5 = 126m. e\?_ (axio8)?_ wes (£) = (Fag) =228 (e) From Eq. (7.390), E=—nkxH, net oe se = 251.33 (2), , and H=250e-%x 10% (A/m). Hence, . — 251.339 x 250e- x 1079 = -812.57e-*” (Vim), and E(t) = Re(Be™) = nen Problem 7.2 Write general expressions for the electric and magnetic fielés of @ 1-GHiz sinusoidal plane wave traveling in the +y-direation in a lossless nonmagnetic medium with relative permittivity &- = 9. The electric cle is polarized along the xedirection, its peak value is 3 V/m and its intensity is 2 V/m atz = 0 and y= 2em. £12.57 cos(10°t—Sy) (Vim). | i TESS 242 CHAPTER 7 Solution: For f = 1 GHz, ur = l.ande, = 9, = 2nf = 2nx 10° rads, Qnf —_ 2nx10? oe a E(y,1) = S3c0s(2x x 10% — 20my+60) (W/m). Atr=Oand V9 = 207 rad/m, 2em,E=2Vim: 2 = 3c0s(—20n x 2x 107? +49) Hence, 0 —0.4n = cos™? (3) = 0.84 rad, which gives 0 = 2.1 rad = 120.19° and E(),1) = &3.cos(2n x 10° — 20my+ 120.19") (W/m). Problem 73 The electric field phasor of a uniform plane wave is given by E=f10e!02 (V/m). Ifthe phase velocity of the wave is 1.5 x 108 m/s and the relative ‘permeability of the medium is ur = 2.4, find (a) the wavelength, (b) the frequency f of the wave, (C) the relative permittivity of the mediam, and (¢) the magnetic field Hr). Solution: _ (a) From E = 10e CV/m), we deduce that k = 0.2 rad/m. Hence, 2m _ 2m de Z =F = 10n=3142m. ®) fe2 LSA? 4.77 x 108 Ha = 4.77 Mie. (© From CHAPTER 7 ae @ = [8 w 1200, fH = 2a ax fZe10 [= 1208 ZB -108 @ fi- x) xE= 1) x $100 = 322.132 (mArm), Hi(z,1) = %22.13c0s(@r+0.22) (mAMm), with « = 2nf = 9.54n x 10° rad/s. Problem 7.4 The electric field of a plane wave propagating in a nonmagnetic snaterial is given by E= [p3sin(2nx 101 —0.4nx) +24cos(2n x 107~0.4nx)} (Vim). ‘Determine (a) the wavelength, (b) &, and (c) E. Solution: (a) Since k= 0.47, &) @_ 2nxi07 > tps P= pag X10" ms Bat ‘Hence, 2 2 3x10 - (FS 3 © = aie [p3sin(am x 10% — O.4nx) +24cos(2n x 1077 0.4n2)] - 2 sin(2n x 10% — 0.4x) — costae X10 04m) (Ala), onal with UE oron= 02.83 ale Problem 7.5 A wave radiated by 2 source in air is incident upon a soil surface, ‘whereupon a part of the wave is transmitted into the soil medium. If the wavelength of the wave is 30 cm in air and 15 cm in the soil medium, what is the soil’s relative permittivity? Assume the soil to be a very low loss medium. Sokution: From 2 = 2o/./E, a= (2 Problem 7.6 The electric field of a plane wave propagating in a lossless, nonmagnetic, dielectric material with e, = 2.56 is given by E= $20cos(8x x 10°—kz) (Vim). Determine: (a) f.up. A, & andy, and (b) the magnetic field HL Solution: @) @= Inf = 8nx 10° rad’s, f =4x 10? Hz = 4 GHz, _e _3xi0t “o” Te VE56 1,875 x 108 4x 10° = 1.875 x 10% mis, = 4.69 em, ecscsiaiitl CHAPTER 7 245 ® | 2 cos(8x x 10 = kz) 20 Fag gp CONST x 10°r — 134.042) £8.49 x 107? cos(Sn x 10°r — 134.042) (A/m). Section 7-3: Wave Polarization Problem7.7 An RHC-polarized wave witha modulus of 2 (V/m) is traveling in free space in the negative z-direction. Write down the expression for the wave's electric field vector, given that the wavelength is 6 cm. ‘Figure P7.7: Locus of E versus time. Solution: For an RHC wave traveling in —2, let us try the following: E= Racos(car + kz) + Jasin( wor +42). ‘Modulus |E| = Va? +a? = av2 = 2 (Vim), Hence, 2 a =v2. 246 CHAPTER 7 Next, we need t0 check the sign of the $-component relative to that of the S-component, We do this by examining the locus of E versus 7 at 2 = 0: Since the wave is traveling along —2, when the thumb of the right hand is along —2 (into the page), the other four fingers point in the direction shown (clockwise as seen from above). Henee, we should reverse the sign of the j-component: E= kV 2cos(wr+kz)—$-V2sin(wr+ ke) (Vim) vie 2n_ 2m Fe Gar MT (radi, and x10 (rads). Problem 7.8 For a wave characterized by the electric field E(z,t) = 2a,cos(«r — kz) +Jaycos(ar~ke+8), ‘identify the polarization state, determine the polarization angles (7,2), and sketch the locus of £(0,t) for each of the following cases: (a) a =3 Vinay = 4 Vim, and =0, (b) a, =3 Vim, ay = 4 Vim, and 5 = 180°, (© a, =3 Vina, = 3 Vim, and = 45°, @ a= 3 Vim,ay = 4 Vim, and 5 = ~135". Solution: yo =tan"Y(ay/ax), (Ea. (7.60)], tan2y = (tan2yo)cos5 [Eq. (7.59a)], sin2x = (sin2wo)sind [Eq. (7.59b)}. Case | a | 4 5 ‘Wo 7 x Polarization State @ [3/4 oO ‘53.13 | 53.13° oO ‘Linear () | 3. | 4} 180° || 53.13° | —S3.13° | oO ‘Linear © |343 45° 45° 45° 25° Left elliptical (@) | 3 | 4 | —135° || 53.13° | -56.2° | —21.37° | Rightelliptical (a) E(z,1) = 83 c0s( coe — kz) + $4c08(«at — kz). (b) E(z.1) = 83c0s( cor — kz) ~ 4cos( — kz). (©) E(z,t) = $3c08( car — kz) + 93cos{ wx - ke + 45°). (4) E(z,1) = £3 cos (or — kz) + 4c0s( ar — kz 135°) 247 Figure P7.8: Plots of the locus of (0,1). Solution: E(z,t) = Re[Be™} el( + 79)20<" Fel] el(8+ Fe™) 200/50] = 20c05( cor — 12/6) +§20c0s( or — nz/6-+ 2/2) 220cos( cr — 2/6) - $20sin(ear—nz/6) (Vim), ye (E| = [E2+E5]"=20 (vim), yest (2) ==(ur—nz/6). From = 2.5% 10" Hz, ‘Therefore, the wave is LHC polarized. ‘Problem 7.10 A linearly polarized plane wave of the form E = Za,e~ can be expressed as the sum of an RHC polarized wave with magnitude ag and an LHC polarized wave with magnitude ay. Prove this statement by finding expressions for @x and ay in terms of a;. Solution: Extae*, RHC wave: By = ag(t+$e-/*/?)e“* = ag(d— jp)e"™*, Lac wave: Ey = a(&+$el")e = a(t + ie, E-h+h, Baz = an(S— 9) +a(Z+ I9). By equating real and imaginary parts, a; = ax +41, an = 2,/2. an + OL, OF OL = ae/2, CHAPTER 7 249 ee Problem 7.11 The electric field of an elliptically polarized plane wave is given by E(z,t) = [-£10sin( ar — kz — 60°) + ¥20cos(@r — kz} (W/m). Determine (2) the polarization angles (7,72) andl (b) the direction of rotation. ‘Solution: @ Eze) = [-810sin{ or — kz — 60°) + $20cos( «or —z)] 10 cos{ cor — kz + 30°) +920cos( ax —kz)] (Vim). Phasor form: {b) Since x, < 0, the wave is right-hand elliptically polarized. Sn Problem 712 Compare the polarization states of each of the following pairs of plane waves: {@) wave 1: Ey = $2cos( «or — ke) +$2sin(ax — kz), wave 2: Ey = $2cos( at + kz) + j2sin( ot + kz), (b) wave I: Ey = 82c0s(« — kz) ~ j2sin( co — kz), wave 2: Ep = #2e08(« + kz) — 92sin( wr + kz). Solution: f) Ey = R2cos(cor — kz) + F2sin(er — kz) = 82cos( or ~ kz) +3 2e08(ar—ke— 2/2), By 82 492 ene, 250 CHAPTER 7 Hence, wave 1 is REC. Similarly, . Bp = 220i + grein in? ‘Wave 2 has the same magnitude and phases as wave 1 except that its direction is along ~2 instead of +2. Hence, the locus of rotation of E will match the left hand instead of the right hand. Thus, wave 2 is LHC. (b) Ey = £2c08( cat ~ ke) — F2sin(cor — kz), By = 82e yp re Heeisl2, Wave 1 is LHC. _ E, = 22e!@ + 920i eN?, ‘Reversal of direction of propagation (relative to wave 1) makes wave 2 RHC. Problem 7.13 Plot the locus of E(0,1) for a plane wave with E(gyt) = Rsin(or+ kz) +9 2eos(ar + kz). Determine the polarization state from your plot. Solution: Es ksin(wr+kz)+$2c0s(at+hz). ‘Wave directions —2. Atz= E=Ssinar +92cosax. ‘Tip of E rotates in accordance with right hand (with thumb pointing along —2). Hence, wave state is RHE. CHAPTER 7 251 ‘Figure P7.13: Locus of E versus time. i Sections 7-4: Propagation in a Lossy Medium Problem 7.14 For each of the following combination of parameters, determine if the materia is @ low-loss dielectric, a quasi-conductor, or a good conductor, and then calculate of By 2, tp, 200 Te: (@) glass with p= 1, & =5,and ¢ = 10- Sim at 10 GHz, (©) animal tissue with uy = 1, & = 12, and 6 = 0.3 S/m at 100 MEz, (©) wood with w= 1, & =3,and = 10~ Sfm at 1 Kitz. Solution: Using equations given in Table 7-1: Case (@) Case (5) Case © ofoe| — 3.6x10-8 45 600 ‘Type | low-lossdielectric quasi-conductor good conductor | 842x10°"Npim —9.75Npim 6.3 x 10~* N/m 468.3 madim 1216 radim 6.3.x 10~* radien 1.34em 51.69em 10km Up 1.34x 108 m/s 052x108 m/s (0.1.x 108 m/s ne H1GBSQ 39.544 3D 6.2814) @ oe Problem 7.15 Dry soil is characterized by €, = 2.5, py = 1, and ¢ = 10* (S/m). Ateach of the following frequencies, determine if dry soil may be considered a good conductor, 2 quasi-conductor, or @ low-loss dielectric, and then calculate ot, B, B. fly. and Me: (a) 60H, (b) 1kH2, © 1MEz, (@) 1 GHz. f Solution: ¢, = 2.5, w= 1, 0=10-* Sim. T= SoHE Taiz TMB Taz == es 12x10 720 072 72x10 = tafee ‘Type of medium | Good conductor | Good conductor | Quasveondustor | Low loss dielectric om | saxio~ | 28x10 | 113x107 119 10 8 crac) 1s¢x10* | 628x107 | 349x107 33.14 tem) 4.08 x 107 Ww 180 (0.19 eos 245x108 | 10? 18x 10" 19 «108 ne Q 1541+) | 62804) | 208.28 765.89 238.27 Problem 7.16 In a medium characterized by & = 9, p= 1, and 6 = 0.1 Sim, determine the phase angle by which the magnetic field leads the electric field at 100 Mz. Solution: The phase angle by which the magnetic field leads the electric field is —G, where @, is the phase angle of ne. S__O1x36e we” ex 108x107 x9 2 CHAPTER 7 253 “Therefore Oy = 31.72". Since H = (1/7)k x E, H leads E by —6,, or by —31.72°. In other words, Flags Eby 31.72". Problem 7.17 Generate a plot for the skin depth 5, versus frequency for seawater for the range from I kHz to 10 GHz (use log-log scales). The constitutive parameters of seawater are y= 1, @ = 80 and o = 4 Sim. Solution: 1 o @=2nf, 80 ee (00) Hel = woeoes = 3 = S= Gx eo s 4x 36m 72 27 we” wee TRFRIO KOO OF See Fig. P7.17 for plot of 8, versus frequency. x 10°, : ‘Senco pte sate SC ‘in ce [ow cre) Figure P7.17: Skin depth versus frequency for seawater. SS 254 CHAPTER 7 Problem 7.18 Ignoring reflection at the air-soil boundary, if the amplitude of a 2-GHz incident waveis 10 V/matthe surface of a wet soil medium, at what depth will, itbe downto 1 mV/m? Wet soilis characterized by ue= 1, &=16,ando = 5x 10-* Sim. Solution: E(g) = Boer = 100", s $x10~* x 360 == = 28x 1078, Ge” Tex2xIP X10 x16 Hence, medium is a low-loss dielectric. go 120m 5x10~*x 120K Ve 2x V6, 102%, In 10~* = -0.0242, 383.76 m. .024 (Nplm), Problem 7.19 The skin depth of a certain nonmagnetic conducting material is 2 ym at 5 GHz. Determine the phase velocity in the material. Solution: For a good conductor, « = B, and for any material 8, = 1/c, Hence, 2nfB, = 2mx5 x 10° x2x 10° = 6.28 10° (ms). Problem 720 Based on wave ettenvation and reflection measurements conducted at 1 MHz, it was determined that the intrinsic impedance of a certain medium is 28.1245 (©) and the skin depth is 5 m. Determine (a) the conductivity of the ‘material, (b) the wavelength in the medium, and (c) the phase velocity. Solution: (2) Since the phase angle of 7. is 45°, the material is a good conductor. Hence, ne =O4dE 28.1e* = 28.1 cosd5* + j28.1sinas*, a 28.1.cos45* = 19.87. o CHAPTER 7 255 Since a= 1/8, =1/5=0.2Np/m, o = 5557 = Tog = 0.01 Sim. (®) Since a = B for a good conductor, and = 0.2, it follows that B = ‘Therefore, (©) Up = fA = 106 31.4 = 3.14 x 107 mis. ee Problem 7.21 The electric field of a plane wave propagating in a nonmagnetic medium is given by E = 225e~-% cos(2n x 10°r— 40x) (W/m). Obtain the corresponding expression for H. Solution: From the given expression for E, @=2nx 10° (rads), =30 (Npim), B=40 (radlm). From (7.65) and (7.658), 2-8 = eye! = —Cuptoe = oon 208 = ape" = Fe Using the above values for ©, c, and B, we obtain the following: g=16, ef =5.47. =f ey? we E(~) £0 gy" 5.47\-¥? ss sal = = 157.9% ) a ( ze 16 ®) -_-_-_——o ees Section 7-5: Current Flow in Conductors Problem 7.22 Ina nonmagnetic lossy, dielectric medium, a 300-MHz plane wave is characterized by the magnetic field phasor Hs (&— jazje%e—™ (4m). ‘Obtain time-domain expressions for the electric and magnetic field vectors, Solution: . _ E=-nkx a. To find N., we need e’ and e”. From the given expression for Hi, a=2 (p/m), B=9 (rad/m). Also, we are given than f = 300 MHz = 3 x 108 Hz. From (7.652), OB = —atye', 9 4-81 = ~(2 x3 x 108) x4 x 1077 et x 36m whose solution gives 2=195, ‘Similarly, from (7.65b), 208 = oP ye", 2x2x9 = (20x 3x 10) x4nx 10-7 xe! x 10? 36x” which gives 0.91. 2357 (0.93 + j0.21) = 256.92. Hence, 256.9228 $x (R— jade Me™ K j4+9)256.9e-Me PAE = (R4e? 4.2) 256.92 Me Mel, ref Bel} £1.03 x 10%e~® cos( «nt ~ 9y-+ 102.6") +2256.9e- cos(ae — Sy + 12.6°) (W/m), H= Re{fie™)} = Ref(R+ j4Z)e Me Mel} Se"? cos( cat —9y) +242" sin{cor— Sy) (Alm). Ee - Problem 7.23 A rectangular copper block is 30 em in height (along z). In response +0 a wave incident upon the biock from above, a current is induced in the block in the positive x-direction. Determine the ratio of the a-c resistance of the block to its ¢-e resistance at 1 Kz. The relevant properties of copper are given in Appendix B. sh | Figure P7.23: Copper block of Problem 7.23. 258 CHAPTER 7 Solution: I de resistance Rye = 0. (2) Write an expression for the electric field phasor of the incident wave, given that the field is a positive maximum at z= O and t = 0. (®) Calculate the reflection and transmission coefficients. (©) Write expressions for the electric field phasors of the reflected wave, the tsansmitted wave, and the total field in the region z < 0. (@) Determine the percentages of the incident average power reficcted by the boundary and transmitted into the second medium. CHAPTER 8 269 Solution: () 2nx2x 108 3x10 LHC wave: (e+e, ape0s( ar — kz) — Jag sin( wr ~ kz), [3] = [abcos? (ar — kz) +. sin?(cor — kz)]"/? = ay = 10 (Vim), B= ag(t+ se?) Br Hence, O(S+ jeH™ (Vie). © =120r (Q), | ‘Beuations (8.82) and (8.9) give i: = Se 208 te14rs ‘ i © i Hr rories net =—- Rees jaye vim, i = 1oeae eM = Deas Neh hm), | #, = B48 = 1012+ 49) [me - jerms| (vim) @ % of reflected power = 100 x [I]? = 2 1.11%, > 207% se of wana pover = 100 x72! = 100x (2) x gE = en.09%. of transmit power x If 7 100x 3 * 88.89%. 270 CHAPTER & —_—_— Problem 85 Repeat Problem 8.4 after replacing the dielectric medium with a poor conductor characterized by & = 2.25, = land = 10~* Sim. Solution: (a) Medium 1: © 2nx2x108 _ 4x tene=120r (0), m= 22 XI LS oa, Medium 2: oe W-*x368 gags oy ~ Dexa xx Zasxi0 —**10 Hence, medium 2 is a low-loss dielectric. From Table 7-1, oe a= 3/2 ot Wily or Eile: _ 10 ta 2 =D TERT Ts x10 OP, B= oVma= ave 2m (rad/m), =,/2 120m, mee (0: p= 80e (2). LHC wave: (a4 HM, 0 (Vim), = 108+ j9)e A"? (Vim). (b) According to gs. (8.82) and (8.9), _Ta—m _ 80x=120r “matt 80n+120n © EF = 109 (2+ sp)e** = -2(2+ je" (Vim), B= 10+ foe Pe Be we B(R4 fem MXI He PE Vien), B, =H +E = 10(8+ jp) M™ 0.20] (vim). CHAPTER 8 m @ % of reflected power = 100|F |? = 100(0.2)? = 4%, % of transmitted power = 100[r|?"4 = 100(0.8)? x 122% = 96%. m2 807 ‘Problem 8.6 A 50-MHz plane wave with electric field amplitude of 30 W/m is ‘normally incident in air onto a semi-infinite, perfect dielectric medium with &; = 36. ‘Determine (a) P”, (b) the average power densities of the incident and reflected waves, and (¢) the distance in the air medium from the boundary to the nearest minimum of the electric field intensity, [E Solution: Problem 8.7 What is the maximum amplitude of the total electric field in the air medium of Problem 8.6, and at what nearest distance from the boundary does it occur? mm CHAPTER 8 Solution: From Problem 8.6,T = 0.71 andi= 6m. (Brlmax = (1+ IT])2§ = (140.71) x 30 = 51.3 Vim, = OA _ x6 _ f= ag Gg Sm Problem 88 Repeat Problem 8.6 after replacing the dielectric medium with it = 2.78% 10"? Sim. To =120n= 377 (0), Salo ‘Medium 2: 2 __2.78x 10-3 x36 we, Bex Sx 10 x 10-5 Hence, Medium 2 is a quasi-conductor. From Eq, (7.70), wo (8) "amir" Ber = 120n(1 = 1)" = 120n(V2)-7eP5° = (299.884 7121.31) (@) Mommy _ (292.884 12131)—377_ Ma+ny (292.884 f12131) 4377 ~ 0.09 + 70.12 = 0.2201 ) : BE 3 sy cw Se 2m: (0.22)7(1.19)= 0.06 (Wim), (©) In medium 1 (air), 3x 108 5x10 CHAPTER 8 273 For @, = 114.5° = 2 rad, Eqs. 8.16) and (8.17) give dielectrics. For a wave in medium 1 incident normally upon the boundary at z= —d, what combination of ¢,, and d produce no reflection? Express your answers in terms of €:, fr, and the oscillation frequency of the wave, f. Figure P8.9: Three dielectric regions. Solution: By analogy with the transmission-line case, there will be no reflection at 2=—d if medium 2 acts as a quarter-wave transformer, which requires that & 4 and m2 = Vis ‘The second condition may be rewritten as to _ {to mo ]” _ (#24) 7 f= VEE a 274 CHAPTER 8 ¢ ¢ do Vin Fite lent)” ¢ Sf Ent) —— Problem 8.10 For the configuration shown in Fig. 8-32 (P8.9), use transmission- line equations (or the Smith chart) to calculate the input impedance at z= —d for & = 1, ty =9 Sy = 4, d= 1.2m, and f = 30 MHz. Also determine the fraction of the incident average power density reflected by the structure. Assume all media are lossless and nonmagnetic. Solution: In medium 2, _ 3x10 _, *3x10x3 Hence, Atz = —d, the input impedance of a transmission line with load impedance 2: is given by Eq, (2.63) as Zt saafad) Zo + je.tanBod, In the present case, Zo = 12 = No/V&q = No/3 and Z. = ns = No/ Vey = No/2 where No = 120n (Q). Hence, immune) ny (3+ 5(4) nln . —d) = 1, (3+ zene) _ = no(0.28+ j0.11 saad eee 3 (da ()end an) ~ O74 IO10. Zal-d) = 2a( Atz= Za—Z, _ o(0.28-+ j0.11)—no = 576 PRO, ZqtLi ~ to(0-28-+ j0-11)+n0 Tr Fraction of incident power reflected by the structure is |T'/? = [0.57] = 0.33. a Problem 8.11 Repeat Problem 8.10 afier interchanging &,, and &,,. CHAPTER § 215 Solution: In medium 2, ae M62 3x10" Ven Pee Seta Hence, Be 2 rad/m, fad = 1.2nerad. Atz=~d, the input impedance of a transmission line with impedance Z is given as Eq, (2.63), a7, ( Zee szotanBe mas 6) = 20 (FE) = No/ Eq = 0/3, A = ns = No/ VE; = No, where Im the present case, Zp = ‘Ae = 120n (Q). Hence, ‘s+ jptan L.2n -d)=7, (BE Tait «(Seen to (1+ (4/3)tant 2x 3 \GA)+ fran 2, =e (SEU IERLZE) — 256- = 0.43 ngZ=5. =n ( PUD a) = (0286- J0337)m0= 043m ABLE. = 0.492210". r= Fath +4 Fraction of incident power reflected by structure is [T/? = 0.24. ee Problem 8.12 Orange light of wavelength 0.61 ym in air enters a block of glass ‘with e, = 2.25. What color would it appear to a sensor embedded in the glass? The ‘wavelength ranges of colors are violet (0.39 to 0.45 jum), blue (0.45 to 0.49 sam), ‘green (0.49 t0 0.58 sm), yellow (0.58 to 0.60 pm), orange (0.60 to 0.62. jm), and red (0.62.t0 0.78 pm). Solution: In the glass, 216 CHAPTER 8 ‘The light would appear violet. Problem 8.13 A plane wave of unknown frequency is normally incident in ait upon the surface of a perfect conductor. Using an electric-field meter, it was determined ‘that the total electric field in the air medium is always zero when measured at a 0. Determine: (a) the polarization of the incident wave, (2) the angie of incidence, (©) the time-domain expressions for the reflected electric and magnetic fields, (@) the time-domain expressions for the transmitted electric and magnetic fields, and (@) the average power density cartied by the wave in the dielectric medium. Solution (a) B= 5102449 vim Since E! is along §, which is perpendicular to the plane of incidence, the wave is perpendicularly polarized. (B) From Eq, (8.482), the argument of the exponential is - occupying the — jky(xsin®, + zeos®,) = — j(3x+4z). Hae, ky sink ky cos; from which we determine that mtn? or a3687%, wit VP+8=5 (adm). Also, oa upk = ck = 3x 10°xS = 15x10" Grads). © CHAPTER 8 289 ‘aces — ni cos +* qac0s8; +71 cos c= 141, $0.59. AL, In accordance with Eq, (8-492), and sing the relation Ej = TLE}, where we used the fact that @; = @, and the z-direction has been reversed. Et = Steel] = —$4.1c05(1.5 x 10%-3x+4z) (Vim), HF = (#8.70+26.53)cos( 1.5 x 10% 3x+4z) (mA/m). (@) In medium 2, hen fZ=svi=10 jam, and a.=sin | sina eso! [Janse Ve z and the exponent of Et and Bis 11.46" iGz+9.542). — jka( asin ®, + 20089.) = ~j10(xsin17.46° + 2c08 17.46") = Hence, 10x 0.59 ¢-39-542), 100.59 -_ psr40.ste) Te (~Sc0s0,+4sind,) Et = RefB'e!] = 75.90c0s(1.5 x 10% —3x- 9.542) Bt = (00s 17.46" + 2sin17.46°) FP costs x 10% 31-9542) = (-229.86 + 29.39) cos(1.5 x 10°%—3x—9.54z) (mA/m). (vim), © (zi? _ (5.90 Su = ay Brass =009 Wm?) ——<$<$<—< 290 CHAPTER 8 Problem 827 Repeat Problem 8.26 for a wave in air with i §2x 10-72 E62) (A/mm), E incident upon the planar boundary of a dielectric medium (z > 0) with ¢; = 9. Since Ef is along #, which is perpendicular to the plane of incidence, the wave is I ‘TM polarized, or equivalently its electric field vector is parallel polarized (parallel to | ‘the plane of incidence). (B) From Eq. (8.65b), the argument of the exponential is —jky(xsin; + 20088)) = —j(8x+ 62). Hence, kysin@=8, ky cos; from which we determine tant ($) = sas, VE+E=10 (adm). | Also, =3x108x10=3x 10% (rad/s). © = Agcos@,— Ty cos; ‘T2C0s@; +71 cos®; an 14 ny) Se = 0.44, In accordance with Bgs. (8.652) to (8.654), Ej = 2x 107%) and Bi = (€c0s@;—sind;)2 x 10-7 e AEE) = (24.52 26.03) erro), CHAPTER 8 291 Bis similarto € except for reversal of z-components and multiplication of amplitude by Ty. Hence, with Ty = -0:30, BT = ef el] = —(81.36 + 21.81) 005(3 x 10°r — 8x +62) Vim, HE = 92x 10-7Ty cos(3 x 10% — 8x4 62) =90.6 x 107? cos(3 x 10°r— 8x+6z) A/m. (@) Inmedivm 2, and the exponent of Et and Bt is — jta(xsin®, +2008) = — j20(xsin15.47° + z00815.47° Hence, Ei = (Leos, —asine,) Eyer AHEM = (80.96 ~ 20.27) 2 x 107? 377 0.44 e H8478918) = (23.18 20.90) eS ESIS), feng FED -atentzso19 2 $2.64 x 10-2er AOH+28913), B= me{Bel*} (%3.18 — 20.90) cos(3 x 10° — 8x— 28.912) Vim, HE = $2.64 x 107cos(3 x 10°r ~ 8x— 28.912) A/m. © s TEBE MeBP | _ (2.64 10-7)? =i 2 = BE = = SE 105.67 = 44 Wie EE Problem 828 Natural light is randomly polarized, which means that, on average, half the light energy is polarized along any given direction (in the plane orthogonal to the direction of propagation) and the other half of the energy is polarized along the i PETA IT 292 CHAPTER 8 direction orthogonal to the first polarization direction. Hence, when treating natural light incident upon a planar boundary, we can consider half of its energy t be in she form of parallel-polatized waves and the other half as perpendicularly polarized waves, Determine the fraction of the incident power reflected by the planar surface of a piece of glass with x = 1.5 when illuminated by natural light at 70°. Sotution: Assume the incident power is 1 W. Hen Incident power with parallel polarization 05W, Incident power with perpendicular polarization = 0.5 W. £2/€1 = (ma/m)? = n? = 1.5? = 2.25. Equations (8.60) and (8.68) give _ 90870° - 2.25 — sin? 70° oe a 605 70° + 92.25 — sin? 70° Ty = 222500870" + V/2.25 = sin? 70° eee eee 2.250870" + /2.25— sin? 70° Sr) .5 (0.21)? = 22 mW, S(T)? 3(0.53)? = 151 3m. ‘Total reflected power = 22 + 151.3 = 173.3 mW, or 17.33%.. Reflected power with parallel polarization Reflected power with perpendicular polarization Problem 829 A parallel polarized plane wave is incident from air onto a dielectric medium with ¢, =9 at the Brewster angle. Whatis the refraction angle? Figure P8.29: Geomeuyy of Problem 8.29. Solution: For nonmagnetic materials, Eq. (8.72) gives = Op ten! [2=w3=7157. CHAPTER & 293 or @: = 18.44". ln Problem 830 A perpendicularly polarized wave in air is obliquely incident upon a planar glass-air interface at an incidence angle of 30°. The wave frequency is 600 THz (1 THz = 10"? Hz), which corresponds to green light, and the index of refraction of the glass is 1.6. Ifthe electric field amplitude of the incident wave is 50 ‘Vim, determine (@) the reflection ané transmission coefficients, and () the instantaneous expressions for E and H in the glass medium. Solution: (a) For nonmagnetic materials, (£2/£1, (8.60) gives re cos i — of (m/m)? sin? _ cos30° — /(1.6)?—sin*30° * 0058+ af (n/m) —sin?@ 160830°+ 4/(1.6)?—sin?30” ts 14Ty = 1-0.27= 0.73. (ng/ni)?. Using this relation in Eq. 0.27, (B) In the glass medium, sin® m sin@, 120% 220K _ 75n= 235.62 (9), 1.6 ‘anf _ 2nx 600x102 x16 _ 6 iy” an = === ae = 6.40 x 10° rad/m, Eh = (8) = 0.73 x 50 = 36.5 Vim. From Eas. (8.49¢) and (8.494), Bi = peje strc) Hi, = (-kcos@,+2sin6,) 2 HafasinGtzcor®), 294 CHAPTER & ‘and the comesponding instantaneous expressions are: EL(xzt) Bh (x2) 36.5cos( xr — kaxsin®,~kzzeos8,) (Vm), C088, — 2.058,)0.16cos( «ar — kaxsin, — kezc0s®.) (A/m), with @ = 2n x 10! rad/s and kp = 6.4% x 10° rad/m. —— eee Problem 831 Show that the reflection coefficient I. can be written in the form sin(®,— 8) TS Sn(66) Solution: From Eq, (8.582), Tacos8\~n: cos, _ (n2/ni)eos®;~cos@, Tiac0s6, +n: c0s6, ~ (m2/ni)e0s®, + cose, © ‘Using Snell's law for refraction given by Eq, (8.31), we have a _ sin my” Sin’ _ Sin@,cos@j—cos@sin@; _ sin(@,—6,) ~ Sin, cos; Feos@,sing; ~ sin(@, +6)” aa Problem 8.32 Show that for nonmagnetic media, the reflection coefficient Ty can be written in the form a tant ** an6.F8) Solution: From Eq. (8.66a), I} is given by Ty = Teges@—mc0s® _ (n2/n1)cos®,— cose 1 q2e0s6,-#n; 6058; ~ (Fz/m)c0s8, +0056,” For nonmagnetic media, 1 = 1p = yo and CHAPTER § 295 Snell's law of refraction is Hence, Ti gn8,_, sin, m9, F080, ‘To show that the expression for Ty is the same as — Han(@:— 8) Ti a(er8,)” We shall proceed with the latter and show that it is equal to the former. tan(6,—6:) _ sin(®—€)o05(&+61) tan(6,48,) ~ eos(@,—@)sin(@,+@:)” Using the identities (from Appendix C): 2sinxeosy = sin(x+y) +sin(x—y), and if we let x= @—6; and y = @, +8; in the numerator, while letting x= @, +0, and y= &— 8; in the denominator, then sin(20,) + sin(—28;) = “sin(28,)+ sin(28) ” But sin20 = 2sinOcos®, and sin(—€) —sin6, hence, tan(@,—6;) _ sin@cos®,—sin@cos tan(@, +6) ~ Sin®,cos®, + sin, cos®,” which is the intended result. Problem 833 _A parallel polarized beam of light with an electric field amplitude of 20 (V/m) is incident in air on polystyrene with 1, = 1 and e, = 2.6. If the incidence angle at the air-polystyrene planar boundary is 50°, determine (a) the reflectivity and transmissivity, and (b) the power carried by the incident, reflected, and transmitted beams if the spot on the boundary illuminated by the incident beam is 1 m? in area. PRIRRERERARCT OTT 296 CHAPTER 8 Solution: (2) From Eq. (8.68), ~(€2/€1)<056, + /(e2/e1) — sin®®, (€2/€1)cos®; + ¥/ (€2/e1)—sin? —2.6c0s50° + \/2.6— sin" 50° 2.600850? + 2.6 — sin? 50 IT yl? = (0.08) = 6.4 x 1073, Ry = 0.9936. 0.08, ®) or x 1208 x c0s50° = 0.34 W, 0.338 W. Sections 8-6 to 8-8: Geometric Optics Problem 834 A man is 2 m tall. How long should a mirror be, when placed in front of him at a distance d, in order for him to have a full-length view of his reflected image? Does the answer depend on a? Figure P8.34: For the man to be able to see his feet, the mirror must be at least half the man’s height. Solution: The mirror needs to be only half his height and positioned as illustrated in the figure. The distance d is irrelevant. CHAPTER 8 297 — Problem 8.35 A gas flame is 2 m from a screen, as shown in Fig. 8-39 (P8.35). A. concave mirror is used to produce on the screen a real image of the flame, magnified four times. Determine s and the focal length of the mirror. Concave misror Sereen Gas Flame be tn I Figure P8.35: Imaging configuration of Problem 8.35. Solution: From Table 8-4, ifthe image is real and magnified, then the object has to bbe between f and 2f from the mirror. Also, the image is inverted and at / = $+ 2. Hence, st2__, 2 Consequently, s on 267 Problem 8.36 A dentistuses 2 concave mirror with a radius of curvature of 6 cm {0 view a filling in a tooth, If the mirror is placed at 2.¢m from the tooth, by how many times will the image of the filling be magnified? Solution: =6cm, cm, 298 CHAPTER 8 which yields ’ = —6 em. Hence, M= Problem 8.37 A flower is 20 cm from a concave spherical misror. If the image of the flower is formed at a distance of 80 cm from the mirror, determine (a) the mirror’s radius of curvature, and (b) the lateral magnification. Solution: (@) s=20cmands’ = 80 cm. Hence, from Eq. (8.86), or R= 32cm. () From Eq, (8.84), Problem 8.38 A candle in air is placed at a distance of 25 cm to the left of a glass ‘medium with a concave spherical boundary characterized by a radius of curvature of 50 em. If the index of refraction of glass is 1.5, determine the location of the candle’s image. : Solution: With ny =1, m= 1.5, s=25 om, and R curvature of a concave boundary is negative), Ea. (8.94) is, 50 cm (the xadins of 1515-1 Ble which gives Problem 839 A thin bi-convex lens with index of refraction of 1.6 has radii of ‘curvature of 20em and 30 cm. Determine the location, magnification, and orientation of an object placed 20 cm from the lens. mR CHAPTER 8 299 Solution: From Table 8-5, a biconvex lens has a positive focal length. Hence, in Eq, (8.102), we set Ry = 20 em and Ry = 30 em: pao-o(h-a)=04-0(-8) which gives f = 10cm. Application of Eq, (8.103) gives + ze ‘The image is virtual and erect, and M = —s'/s = 1.25. —25 ern. Problem 8.40 Repeat Problem 8.39 for an object placed 50 cm from a planar- concave leas with a radivs of curvature of 15 cm. Solution: For a planar-concave lens (which has a negative focal length), we use 15 em in Eq, (8.102) to get o-(F-Z) 16-1)(0- 4). which gives f = —25 cm. Eq. (8.103) gives toil Pusp th oak, -16.67 om. stytcg: (= 16.670 ‘The image is virtual and erect, and M = —s'/s = 0.33. Problem 8.41 The curved face of a planar convex lens has a radius of curvature of 3 cm and the index of refraction of the lens material is 1.5. Determine the focal length of the lens when: (2) the planar surface of the lens faces the light, and (®) the convex surface of the lens faces the ght. Solution: @) ®) f=6cm. ‘Thus, f of a planar convex lens is positive regardless which side faces the incident light. es Problem 8.42 The image formed by 2 convex lens was observed to move by @ Gistance of 1 em when the object was moved from far away (essentially at infinity) 10 ory 42.cm from the lens. What is the focal Jength of the lens? Solution: When object is at infinity, image is at ssf ‘When objectis at s = 42 em, image is a sh: ryt Bans But s,=s+1em= f+1om. Hence, 1 1 Ben * Ftiea 1 1 _Uen-f_ Ben Ff Fel AF+) F+1)" which yields the solution —_— Problem 8.43 Equation (8.102) defines the focal length of a thin lens in air. (a) Derive an expression for f for the general case of a thin lens with index of refraction n; immersed in & medium with index of refraction 1m. (©) Determine the focal length of a bi-concave lens with radii of curvature of 10 em. and 15 om and index of refraction of 1.5 when placed in air and when placed in water (with index of refraction of 1.33). Solution: (a) The thin-leas equation is given by Eq. (8.100) as 2m 1 2p = (mad (Ge - 6em. CHAPTER 8 301 OE) @-a) f Tim) \Ri Ra)” (b) A bi-concave lens has a negative focal Jength (when 7) > itm). Hence, in above ‘equations, we choose Ry = 15 om and Re = 10 em: .60 cm (in air), (Ff = 235 em Gin water). Problem 8.44 A positive lens is used to image an object placed 50 cm in front of it ‘ona screen located 1 m behind the lens. What is the focal length of the lens? Solution: Bet 307 100 =4, or f=3333em. 7 i —_— Problem 8.45 An imaging system consists of two thin positive lenses with focal Tengths of 15 cm for the first lens and 5 em for the second. If the two lenses are separated by a distance of 50 cm, locate the image of an object placed 25 cm in front of the first lens, relative to the location of the second lens. Solution: Image due to first lens: 4ytlt ay which gives 54 = 37.5 em, With respect to the second lens, s2 = SDem—s = 12.5em TE TRE IT I eT I Figure P8.45: Two-lens arrangement of Problem 8.45, —_— Problem 8.46 Two thin lenses with focal lengths fr and fp are placed in contact with each other, as shown in Fig. 8-40 (P8.46). Show that F,, the focal length of the ‘combination, is given by Ah Figure P8.46: Two lenses in contact The lenses are so thin that they may be considered to be at the same location (Problem 8.46), Solution: The lens formula for the first lens is tia nT RA m2 CHAPTER 8 303 For the second lens, oi ta ‘The object for the second lens is virtual because it is the image formed by the first lens, and since the two lenses are thin and in contact, we can consider them to be at the same location. Hence, as) wm ens an and =a a) ‘Upon combining Eq. (15) and Eq. (18) and eliminating s1, we have Gd)-f ae so A 7 Ljiit nTETF where s; is the object distance and s} is the image due to the combination of both lenses. ——— Problem 8.47 Two thin lenses of focal lengths fi = 5 em and fo = —10 cm are separated by a distance of Scm, as shownin Fig. 8-41 (P8.47). Determine the location of the object. Solution: 304 CHAPTER 8 Figure P8.47: Imaging configuration of Problem 8.47. From the figure, [so] = 15— 5 = 10cm, and since itis a viral object, s» = —10 em. Lyfe, =0 7 70° geo CHAPTER 9 305 Chapter 9 Sections 9-1 and 9-2: Short Dipole and Antenna Radiation Characteris- tes Problem 9.1 A center-fed Henzian dipole is excited by a current I dipole is 3/50 in length, determine the maximum radiated power density at a distance of km, Solution: From Eq, (9.14), the maximum power density radiated by a Hertzian dipole is given by _ mok2BE _ 377 x (2n/A)? x 10? x (0/50)? “3ueR aaa = 1.9% 10-8 Wim? = 1.9 GiWim*). So Problem 92 A I-mlong dipole is excited by a 1-MHz current with an amplitude of 12 A. What is the average power density radiated by the dipole at a distance of 5 km in a direction that is 30° from the dipole axis? Solution: At 1 MHz, 2 = c/f = 3x 108/108 = 300 m. Hence 1/2 = 1/300, and therefore the antenna is a Hertzian dipole. From Eq. (9.12), sine) (BEBE) are __ 120nx (21/300)? x 122 x 12 in? 10 2 BaP Sx 30" = 754 10"! (wie). Problem 93 Determine the (a) direction of maximum radiation, (b) directivity, (¢) beam solid angle, and (d) half-power beamwidth in the xz plane for an antenna ‘whose normalized radiation intensity is given by 1, for0 <0 < 60°and 0 < 0 < 2, 0, elsewhere. ‘Suggestion: Sketch the pattern prior to calculating the desired quantities. Solution: The direction of maximum radiation is a circular cone 120° wide centered around the +Z-axis. From Eq. (9.23), F(8.0)= { an, an 4x Bie a WexF A292 ("singdode —2n(—cos0)|é 6aB, eT ET CTT LE DT eT 306 CHAPTER 9 fast _ nse y= BE Bn (x) ‘The half power beamwidth is 8 = 120°. SI EEEEEEE Ener Problem 9.4 Repeat Problem 9.3 for an antenna with re={ dia Oot for0s OS mmd —7/2< 65/2, Solution: The direction of maximum radiation is the +-axis (where @ = n/2 and = 0). From Eq. (9.23), 4a PST Fa an © (2B, Ii sim? 0coe? ¢sind doco _ a © JEG, cos ode ff sim? 00 _ an © iia Ha +00s20)d0 fF, (1-2 )ae sad 4x 4n “Fes dan20) [4-2/3 | 4A) =78B, 2 SS aE. In the xz plane, @ = 0 and the half power beamwidth is 90°, since sin?(45*) = sin? (135°) = i, ‘band at 1 MHz. The dipole is made of copper wire with a radius of 1 mm. (@) Determine the radiation efficiency of the antenna. (b) Whats the antenna gain in dB? <©) What antenna current is required so that the antenna would radiate 20 W, and ‘how much power will the generator have to supply to the antenna? Solution: CHAPTER 9 307 (a) Following Example 9-3, 2 = ¢/f = (3x 10 m/s)/(10° Hz) = 300m. As 1/2.= (2m)/(300 m) = 6.7 x 10, this antenna is a short (Hertzian) dipole. Thus, from respectively Eqs. (9.35), (9.32), and (9.31), =35 (mQ), SOUS HeiGex 10a) 5.8.x 107 Sim (m2), “wmo+esan 7% (b) From Example 9-2, a Hertzian dipole has a directivity of 1.5. The gain, from Eq, (9.29), is G = ED = 0.297 x 1.5 = 0.44 = —3.5 dB. (¢) From Eq. (9.30a), gay [POY _. 0 Ve V asma 84 = Pat - 20W _ 673 Re a 157 = 673 W. and from Eq. (9.31), Problem 9.6 Repeat Problem 9.5 for 2 20-cm-long antenna operating at 5 MHz, Solution: (a) At S MHz, 2=¢/f =3 x 10°/(5x 108) = 60m. As I/A = 0.2/60 = 3.33 x 1073, the antenna length satisfies the condition of a short dipole. From Eqs. (9.35), (9.32), and (931), 2 Rag = 80x? () "= gon? x (3.33 x 10-3) =8.76 (m2), _ | fife 02 fex5x108x4nx 10-7 Runs = ea oe Dax 10S 58x10" 187 GeO}, x g= Rot 8.76 _ 0.32, or 32%. ~ Raat Rios 8.76+18.57 (b) For Hertzian dipole, D = 1.5, and G = ED = 0.32% 1.5=0.48 = -3.20B. (©) From Eq. (9.308), p= |e 2x20 OSV Raa V876xH =676A. 308 CHAPTER 9 Problema 9.7 An antenna with a patter solid angle of 1.5 (st) radiates 30 W of power. Ata range of 1 km, what is the maximum power density radiated by the antenna? Solution: From Eq. (9.23), D = 4n/Qp, and from Eq, (9.24), D = 4nR*Smax/ Pra ‘Combining these two equations gives Seu — at 30 rman = OUR? Sx (08 — Problem 9.8 An antenna with a radiation efficiency of 90% has a directivity of 6.7 4B. What is its gain in €B? Solution: D = 6.7 dB corresponds to D = 4.68. 2x107% Wim), G=ED=0.9x 4.68 = 4.21 = 6.24 dB. Altematively, G (dB) = (4B) + D (AB) = 1010g0.9+ 6.7 = -0.46+ 6.7 = 6.24 GB. ea Problem 9.9 The radiation pattem of a circular parabolic-reflector antenna consists of a circular major lobe with a half-power beamwidth of 2° and a few minor lobes. ‘Ignoring the minor lobes, obtain an estimate for the antenna directivity in dB. Solution: A circular tobe means that Bx: = By. = 2° = 0.035 rad. Using Eq. (9.26), weave 4n an P= BB, O035F .03 x 10°. IngB, D(GB) = 1OlogD = 10log(1.03 x 10*) = 40.13 4B. i Problem 9.10 ‘The normalized radiation intensity of a certain antenna is given by F(8)=exp(-2007) for0<@ aie ettson? 5 = [tp elllanrirapeosdn/4y 2 = [1+ ftemsd45/4)2 — eos? (F(4cos0+1)). A plot of this array factor pattern is shown in Fig. P9.29(a). | 324 CHAPTER 9 Figure P9.29: (a) Array factor in the elevation plane for Problem 9.29(a). (©) Employing Eq, (2.110), 1 F(8)= [Beetanen a & [14 2ell(2a/?heos0+0) 2 — 1 4 2ei?meos/? . 5+ 4cos(2ncos6). A plot of this array factor pattem is shown in Fig. P9.29(b). 325 Figure P9.29: (b) Array factor in the elevation plane for Problem 9.29(b). (6) Employing Eq, (9.110), and setting ag = a1 = 1, y=0, Wi = n/2 and a= 1/2, we have 2 r(e=| Sacer » = h + EF qheaPV Oe = lt + eimene-as2 = 4e0s* (Fos = A plot of the array factor is shown in Fig. P9.29(6). 326 CHAPTER 9 Figure P9.29: (c) Array factor in the elevation plane for Problem 9.29(c). (@) Employing Eq. (9.110), and setting ap = 1, a1 = 2 yo= and d = 2/2, we have 2 F(8) Biaetmeraen - | 14 2ef®/4 gil2m/A}(A/2)c0s6| =p zetacosena =5+4c0s (ncoso+ 2) A plot of the array factor is shown in Fig. P9.29(d). CHAPTER 9 327 Figure P9.29: (¢) Array factor in the elevation plane for Problem 9.29(6). (© Employing Eq. (9.110), and setting a = 1, a1 =2, vo =0, Wi and d= 2/4, we have FA(8) = Saye ete es = [i 8 Delt gKn/VO14) 8? = h taeireooenye)? = 5-+4e0s (Foos0+ 5) = 5—4sin($ A plot of the array factor is shown in Fig. P9.29(e). 328 CHAPTER 9 =H Figure P9.29: (¢) Array factor in the elevation plane for Problem 9.29(e). Problem 9.30 If the antennas in part (a) of Problem 9.29 are parallel vertical Hertzian dipoles with axes along the x-direction, determine the normalized rediation intensity in the x-z plane and plotit Figure P9.30: (a) Two vertical dipoles of Problem 9.30. CHAPTER 9 329 sls g Figure P9.30: (b) Pattern factor in the elevation plane of the array in Problem 9.30(@). Solution: The power density radiated by a Hertzian dipole is given from Eq. (9.12) ‘by Se(8!) = Sosin” 0’, where 6! is the angle measured from the dipole axis, which in the present case is the a-axis (Fig. P9.30). Hence, @' = x/2-@ and S,(@) = Sosin®(}m—8) = Socos*®. Then, from Eq, (9.108), the total power density is the product of the element pattem and the array factor. From part (a) of the previous problem: 5(8) = S{0)F,(€) = 4500s? Bcos* (F(4e0s8+1)). ‘This function has a maximum value of 3.525 and it occurs at @max = £135.5°. The ‘maximum must be found by trial and error. A plot of the normalized array antenna pattem is shown in Fig. P9.30. 330 CHAPTER 9 — eee Problem 931 Consider the two-element dipole array of Fig. 9.29(a). If the ewo dipoles are excited with identical feeding coefficients (ay = a = 1 and Yo = Vi = 0), choose (d/2.) such that the array factor has a maximum at 6 = 45° Solution: With a9 = a; = 1 and yo = Vi = 0, FA(@) =| 1 + elt e080)2 — goog? (Heese). F,(@) is a maximum when the argument of the cosine function is zero or a multiple of x. Hence, for a maximum at 6 = 45°, ‘The first value of x, namely = 0, does not provide a useful solution because it requires d to be zero, which means that the two elements are at the same location. While this gives 2 maximum at @ = 45°, it also gives the sme maximum at all angles @ in the yxz plane because the two-element array will have become a single element with an azimuthally symmetic pattem. The value n = 1 leads to 414, Problem 9.32 Choose (d/7.) so that the array pattern of the array of Problem 9.31 hhas a null, rather than 2 maximum, at @= 45°, Solution: With ap = ay = 1 and yo = wr = 0, Fy(0) = |1 + elneiM ene? — segs? (Heese) . F,(@) is equal to zero when the argument of the cosine function is ((x/2) + nti}. Hence, for a null at @ = 45°, Stam, =0,1,2,.... CHAPTER 9 331 Problem 9.33 Find and plot the nonmalized array factor and determine the half- power beamwidth for a five-element linear array excited with equal phase and 2 ‘uniform amplitude distribution. The interelement spacing is 32/4. Solution: Using Eq. (2.121), sin? [(Nad/A)cos®] __ sin” [(151/4)cos6] N*sin?[(nd/2.)cos®] — 25sin® [(3n/4)cos6] and this pattem is shown in Fig. P9.33. The peak values of the pattern occur at @ = £90°. From numerical values of the pattem, the angles at which Fx(8) = 0.5 are approximately 6.75°on either side of the peaks. Hence, B = 13.5%. Fan(8) x Figure P9.33: Normalized array pattern of a S-element aray with uniform amplitude distribution in Problem 9.33. Problem 9.34 A three-element linear aray of isotropic sources aligned along the z- axis has an interelement spacing of 2/4 Fig. 9-38 (P9.34). The amplitude excitation of the center element is twice that of the bottom and top elements and the phases 332 CHAPTER 9 ‘are —1/2 for the bottom element and x/2 for the top element, relative to that of the Center element. Determine the array factor and plot it in the elevation plane. a2 7 uA + 2Lo a Liz Figure P9.34: (a) Three-element array of Froblem 9.34. Solution: From Eq. (9.110), cae 4 aye elio0 4 gaeNng dead)? = | eileW2) 5 el ghh2/AN/S)o008 forte

You might also like